You are on page 1of 88

PART 3A COST MEASUREMENT CONCEPTS

ANSWERS Answer (A) is incorrect because $38 does not


include $32 direct materials.
[1] Source: CMA 0690 5-27
Answer (A) is incorrect because committed costs Answer (B) is correct. Variable costs vary in direct
are fixed costs arising from the possession of plant proportion to production. Total unit variable costs are
and equipment and a basic organization. These costs $70 ($32 direct materials + $20 direct labor + $15
are affected primarily by long-run decisions as to a variable overhead + $3 variable selling costs).
company's desired capacity.
Answer (B) is correct. Discretionary costs are Answer (C) is incorrect because $52 does not
characterized by uncertainty about the relationship include $15 variable overhead and $3 variable selling
between input (the costs) and the value of the costs.
related output. Advertising and research are
examples. They should be contrasted with Answer (D) is incorrect because $18 does not
engineered costs, that is, costs having a clear input- include $32 direct materials and $20 direct labor.
output relationship (e.g.,
the cost of direct materials).
[5] Source: CMA 1291 3-30
Answer (C) is incorrect because opportunity cost is
the return available from the next best use of a Answer (A) is incorrect because total estimated costs
resource. are $948,000 [($73 x 12,000) + ($4 x 12,000) +
($3 x 8,000)].
Answer (D) is incorrect because differential
(incremental) costs are those that vary among Answer (B) is incorrect because $960,000 is
decision options. calculated using 12,000 units to determine variable
selling costs instead of 8,000.

[2] Source: CMA 1291 3-27 Answer (C) is correct. Manufacturing costs at a
production level of 12,000 units are $73 per unit
Answer (A) is incorrect because $35 equals direct ($32 + $20 + $15 + $6). Total estimated
labor plus variable manufacturing overhead. It does manufacturing costs are therefore $876,000 ($73 x
not include $6 fixed overhead. 12,000 units). Fixed selling costs are expected to be
$48,000 ($4 x 12,000 units). The anticipated total
Answer (B) is correct. Conversion costs are incurred variable selling costs are $24,000 ($3 x 8,000 units).
in transforming raw materials into finished Thus, the total estimated costs are $948,000
products. ($876,000 + $48,000 + $24,000).
They include direct labor and factory overhead.
Thus, Answer (D) is incorrect because $932,000 is
unit conversion costs equal $41 ($20 direct labor + calculated using 8,000 units to determine fixed selling
$15 variable overhead + $6 fixed overhead). costs instead of 12,000.

Answer (C) is incorrect because $48 includes $3 in


variable selling costs and $4 in fixed selling costs. [6] Source: CMA 0692 3-6

Answer (D) is incorrect because $67 includes $32 in Answer (A) is correct. The activity base for overhead
direct materials and excludes $6 fixed overhead. allocation should have a high correlation with the
incurrence of overhead. Thus, the activities of various
departments are usually more appropriate as activity
[3] Source: CMA 1291 3-28 bases than plant-wide activities, particularly when
products and production activities are not
Answer (A) is incorrect because $73 includes $15 homogeneous.
variable manufacturing overhead and $6 fixed
manufacturing overhead. Answer (B) is incorrect because the number of
departments is not as important as the relationship
Answer (B) is incorrect because $32 does not between the costs and activity base.
include $20 direct labor.
Answer (C) is incorrect because products require
Answer (C) is incorrect because $67 includes $15 similar manufacturing effort, they are relatively
variable manufacturing overhead. homogeneous, and a plant-wide rate might be
adequate.
Answer (D) is correct. Prime costs are the costs of
direct materials and direct labor. They are costs Answer (D) is incorrect because the degree of
that variability in costs is not as important as the
can be directly associated with the finished relationship between activity bases and costs, and the
product. degree to which manufacturing activities are similar
Hence, unit prime costs are $52 ($32 direct for all products.
materials
+ $20 direct labor).
[7] Source: CMA 0691 3-22

[4] Source: CMA 1291 3-29 Answer (A) is incorrect because external failure cost
is not related to statistical quality control. Answer (D) is incorrect because process costing is
employed when a company mass produces a
Answer (B) is incorrect because internal failure cost homogeneous product in a continuous fashion through
is not related to statistical quality control. a series of production steps.

Answer (C) is incorrect because prevention cost is


not related to statistical quality control. [10] Source: CIA 0594 III-47

Answer (D) is correct. Statistical quality control Answer (A) is incorrect because marketing and
methods attempt to identify nonrandom variations distribution costs should be allocated to specific
in products.
operating activities. The costs of maintaining a
statistical quality control system are appraisal costs Answer (B) is correct. ABC determines the activities
because they are incurred to detect products not that will serve as cost objects and then accumulates a
meeting specifications. cost pool for each activity using the appropriate
activity base (cost driver). It is a system that may be
employed with job order or process costing methods.
[8] Source: CIA 1193 IV-6 Thus, when there is only one product, the allocation
of costs to the product is trivial. All of the cost is
Answer (A) is incorrect because factory overhead assigned to the one product; the particular method
should be charged for direct materials, supplies, used to allocate the costs does not matter.
direct labor, and applied overhead incurred for
rework. Answer (C) is incorrect because ABC determines the
activities that will serve as cost objects and then
Answer (B) is incorrect because factory overhead accumulates a cost pool for each activity using the
should be charged for direct materials, supplies, appropriate activity base (cost driver).
direct labor, and applied overhead incurred for
rework. Answer (D) is incorrect because, under ABC, a
product is allocated only those costs that pertain to its
Answer (C) is incorrect because factory overhead production; that is, products are not
should be charged for direct materials, supplies, cross-subsidized.
direct labor, and applied overhead incurred for
rework.
[11] Source: CIA 0591 IV-7
Answer (D) is correct. The rework charge for direct
materials, indirect materials (supplies), direct labor, Answer (A) is incorrect because process costing is
and overhead applied on the basis of direct labor employed when manufacturing involves a
cost is $40,300 [$5,000 + $300 + $14,000 + (1.5 x homogeneous product.
$14,000)]. If an allowance for rework is included in
a Answer (B) is incorrect because relevant costing
company's manufacturing overhead budget, rework refers to expected future costs that are considered in
of defective units is spread over all jobs or batches decision making.
as
part of the predetermined overhead application Answer (C) is incorrect because direct costing
rate. includes only variable manufacturing costs in unit cost.
Hence, the debit is to overhead control.
Answer (D) is correct. The job-order cost system of
accounting is appropriate when products have varied
[9] Source: CIA 0593 IV-4 characteristics and/or when identifiable groupings are
possible, e.g., batches of certain styles or types of
Answer (A) is correct. Job-order costing is used by furniture. The unique aspect of job-order costing is
organizations whose products or services are the identification of costs to specific units or a
readily particular job.
identified by individual units or batches. The
advertising agency accumulates its costs by client.
Job-order costing is the most appropriate system [12] Source: CIA 1188 IV-5
for
this type of nonmanufacturing firm. Answer (A) is incorrect because process costing is
more appropriate for the continuous manufacture of
Answer (B) is incorrect because operation costing similar products.
would most likely be employed by a manufacturer
producing goods that have common characteristics Answer (B) is incorrect because transfer pricing is
plus some individual characteristics. This would not used when a company transfers products between
be an appropriate system for an advertising agency two decentralized divisions.
with such a diverse client base.
Answer (C) is correct. The job-order cost system of
Answer (C) is incorrect because relevant costing accounting is appropriate when producing products
refers to expected future costs that are considered with individual characteristics such as in the
in manufacturing of custom-built yachts. The unique
decision making. aspect of job-order costing is the identification of
costs to specific units or a particular job.
[15] Source: Publisher
Answer (D) is incorrect because the step-down
method allocates service department costs to the Answer (A) is incorrect because EWIP is credited
users of services. when completed units are transferred but should
never have a credit balance.

[13] Source: CIA 0589 IV-4 Answer (B) is correct. The sum of the debits to WIP
equals total production costs to be accounted for.
Answer (A) is incorrect because a process costing Ignoring possible spoilage, production consists either
system is used for continuous process of goods that have been completed or those still in
manufacturing process. Accordingly, after the account is credited
of units that are relatively homogeneous (e.g., oil for the cost of goods completed and transferred to
refining and automobile production). the FG inventory, the debit balance in the account is
EWIP.
Answer (B) is correct. The job-order cost system of
accounting is appropriate when producing products Answer (C) is incorrect because total production
with individual characteristics and/or when costs to be accounted for include finished goods as
identifiable well as EWIP.
groupings are possible, e.g., batches of certain
styles Answer (D) is incorrect because work-in-process is
or types of furniture. The unique aspect of job-order an asset (inventory) account. Asset accounts are real
costing is the identification of costs to specific units accounts that are not closed out at the end of the
or period.
a particular job. A job-order system is appropriate
in
consulting because of the substantial variation [16] Source: Publisher
between engagements.
Answer (A) is incorrect because factory depreciation
Answer (C) is incorrect because an operations and supplies are excellent examples of indirect costs
costing system is a hybrid of job-order and process that are considered overhead.
costing. It is used by companies that manufacture
goods with some common and some dissimilar Answer (B) is incorrect because service department
characteristics. costs are not directly related to the production of
specific goods; these costs are indirect although
Answer (D) is incorrect because a just-in-time associated with the manufacturing process. Thus, they
costing system is a hybrid-costing system used in can be allocated to the final output.
conjunction with just-in-time production systems. It
eliminates the stores account and detailed Answer (C) is incorrect because maintenance costs
recording are indirectly associated with manufacturing and are
of raw materials and direct labor through various allocated to products through the overhead account.
operations. It also replaces work-in-process with a
raw and in-process inventory. Answer (D) is correct. Marketing costs, for example,
salaries of sales personnel, sales commissions, and
advertising, are period costs and are expensed as
[14] Source: Publisher incurred. They cannot be allocated to the product
because these marketing costs are not associated
Answer (A) is incorrect because work-in-process is with the manufacturing process.
an inventory or real account.

Answer (B) is correct. The manufacturing account is [17] Source: CIA 0591 IV-11
an inventory account to which direct materials,
direct Answer (A) is incorrect because this amount results
labor, and factory overhead costs are charged as from treating the actual overhead and labor hours as
they are incurred in the production process. The the estimated values and vice versa.
sum
of these costs plus the cost of BWIP is the total Answer (B) is correct. Applied overhead equals the
production cost to be accounted for in any one actual labor hours (210,000) times the estimated
period. The total is allocated to goods completed application rate ($500,000 ・200,000 DLH = $2.50
during the period, i.e., to finished goods, and to per direct labor hour), or $525,000. This amount is
EWIP. The manufacturing account may also be $10,000 ($525,000 - $515,000 actual cost) higher
credited for abnormal spoilage. than the actual overhead cost incurred. Hence,
overhead was overapplied by $10,000.
Answer (C) is incorrect because work-in-process is
a real account. Answer (C) is incorrect because this amount results
from treating actual overhead as the estimated value
Answer (D) is incorrect because the factory and vice versa.
overhead account pools the indirect costs as
incurred Answer (D) is incorrect because $15,000 is the
and charges them to production systematically. excess of actual over estimated overhead.
Additionally, factory overhead is overapplied.
Answer (C) is incorrect because activity-based
[18] Source: Publisher costing tends to increase cost pools.

Answer (A) is incorrect because 10,250 would have Answer (D) is correct. Activity-based costing
been produced if overhead had been overapplied analyzes production processes and defines the
by activities of which they are composed. It then
$1,500 [($39,500 + $1,500) ・$4]. accumulates a cost pool for each activity using the
appropriate activity base (cost driver or causal factor
Answer (B) is incorrect because 10,000 is the result in the incurrence of cost). Whereas a traditional
of dividing budgeted, not applied, overhead by the system might use a single cost pool for a process,
application rate. activity-based costing may designate many activities
within that process for the separate accumulation of
Answer (C) is incorrect because 9,875 units would costs, each with a different allocation base. The result
have been produced if $39,500 had been the is greater accuracy because costs are more likely to
amount be assigned on a specific cause-and-effect basis than
of applied overhead. in less precise traditional systems.

Answer (D) is correct. Given actual overhead of


$39,500 and underapplied overhead of $1,500, [21] Source: Publisher
overhead applied was $38,000 ($39,500 - $1,500).
Overhead is applied at the rate of $4 per unit Answer (A) is correct. Transferred-in costs are the
($40,000 budgeted overhead ・10,000 budgeted costs incurred in prior stages of production. They
units). Accordingly, 9,500 units were produced pertain to the units of production that move from one
($38,000 applied overhead ・ $4 per unit process to another. Thus, they are the basic units
application being produced. Direct materials are added to the
rate). basic units during processing.

Answer (B) is incorrect because value is not an issue


[19] Source: CIA 0591 IV-6 in differentiating between transferred-in costs and
other direct materials.
Answer (A) is incorrect because job-order costing is
employed when manufacturing involves different Answer (C) is incorrect because value is not an issue
(heterogeneous) products. in differentiating between transferred-in costs and
other direct materials.
Answer (B) is incorrect because direct costing
includes only variable manufacturing costs in unit Answer (D) is incorrect because direct materials may
cost. be added at the beginning of a process and,
It may be used whether products are homogeneous conceivably, the transferred-in materials (i.e., basic
or heterogeneous and with either process or units) could be added to the process after the direct
job-order costing. materials were processed.

Answer (C) is incorrect because absorption costing


includes all manufacturing costs as part of the cost [22] Source: Publisher
of
a finished product. It may be used whether Answer (A) is incorrect because EUP must be
products calculated for conversion costs.
are homogeneous or heterogeneous and with either
process or job-order costing. Answer (B) is incorrect because transferred-in costs
are usually included at the beginning of the process.
Answer (D) is correct. Like products that are mass They consist of costs incurred in preceding
produced should be accounted for using process departments and are therefore conceptually similar to
costing techniques to assign costs to products. direct materials. Hence, they are separate from other
Costs costs.
are accumulated by departments or cost centers
rather than by jobs, work-in-process is stated in
terms of equivalent units, and unit costs are Answer (C) is incorrect because direct materials may
established on a departmental basis. Process be added at various points in the operation. Each
costing kind of material should be the basis for a separate
is an averaging process that calculates the average EUP calculation.
cost of all units.
Answer (D) is correct. Overhead is often applied on
the basis of a direct labor activity base such as hours
[20] Source: Publisher or cost. Thus, a single EUP calculation is made for
conversion costs (direct labor and overhead). There
Answer (A) is incorrect because activity-based is no basis for separating these costs if they are
costing may be used with process costing. incurred uniformly.

Answer (B) is incorrect because activity-based


costing may be used with a job-order costing [23] Source: Publisher
system.
Answer (A) is correct. Normal spoilage is the
spoilage that occurs under normal operating Answer (D) is incorrect because normal spoilage is
conditions. It is essentially uncontrollable in the allocated to good units as they pass through each
short inspection point based on their relative values (not
run. Normal spoilage arises under efficient based on units).
operations
and is treated as a product cost.
[26] Source: Publisher
Answer (B) is incorrect because, if spoilage occurs
from a special production run, it is abnormal. Answer (A) is incorrect because shrinkage should be
treated as spoilage and charged to the product if
Answer (C) is incorrect because spoilage is normal and charged as a loss if abnormal.
abnormal
if it arises under inefficient operations. Answer (B) is incorrect because shrinkage usually
does not result in scrap or waste products that can be
Answer (D) is incorrect because, if spoilage is sold and accounted for as a revenue.
controllable, it should be controlled under normal
circumstances. Answer (C) is incorrect because shrinkage usually
does not result in scrap or waste products that can be
sold and accounted for as a contra cost.
[24] Source: Publisher
Answer (D) is correct. Shrinkage consists of
Answer (A) is incorrect because the allocation is materials lost through the manufacturing process (e.g.,
solely to cost of goods manufactured when EWIP heat, compression, etc.). It is accounted for in the
has not been inspected. same manner as spoilage. If shrinkage is normal, it is
charged to the product. If it is abnormal, it is charged
Answer (B) is incorrect because normal spoilage as period cost (a loss).
costs should be allocated to all good units, i.e., all
units that have passed the inspection point.
[27] Source: CIA 0591 IV-9
Answer (C) is incorrect because, when the
inspection Answer (A) is incorrect because perfection standards
point is at the end of the process, goods in EWIP are based on perfect operating conditions, and
have not yet passed the inspection point. negative deviation from such standards is expected.

Answer (D) is correct. Normal spoilage costs should Answer (B) is incorrect because abnormal spoilage
be allocated to all good units, i.e., all units that may result from any of a variety of conditions or
have circumstances that are usually controllable by first-line
passed the inspection point. In typical accounting supervisors.
problems, the inspection point is at the end of the
process. Therefore, goods in EWIP have not passed Answer (C) is incorrect because abnormal spoilage
the inspection point. If the inspection point is prior may result from any of a variety of conditions or
to circumstances that are not necessarily related to
the end of the process, however, and goods in EWIP standards.
have passed the inspection point, a portion of the
normal spoilage costs should be allocated to EWIP. Answer (D) is correct. Abnormal spoilage is spoilage
that is not expected to occur under normal, efficient
operating conditions. The cost of abnormal spoilage
[25] Source: Publisher should be separately identified and reported to
management. Abnormal spoilage is typically treated
Answer (A) is incorrect because normal spoilage is as a period cost (a loss) because of its unusual
allocated to good units as they pass through each nature.
inspection point, not just to goods completed
during
the period. [28] Source: CIA 1185 IV-6

Answer (B) is incorrect because normal spoilage Answer (A) is incorrect because abnormal spoilage
must be allocated to all good units as they pass costs are not considered a component of the cost of
through each inspection point, not just to units in good units produced.
EWIP and completed goods based on their relative
values. Answer (B) is incorrect because abnormal spoilage
costs are not considered a component of the cost of
Answer (C) is correct. At each inspection point, the good units produced.
costs of normal spoilage should be allocated to the
good units passing through the inspection point. Answer (C) is correct. Abnormal spoilage is spoilage
Consequently, the cost of moving the good units to that is not expected to occur under normal, efficient
the inspection point includes the direct materials operating conditions. Because of its unusual nature,
and abnormal spoilage is typically treated as a loss in the
conversion costs of the normally spoiled units as period in which it is incurred.
well
as those of the good units. Answer (D) is incorrect because abnormal spoilage
costs must be taken out of the manufacturing account.
completed rather than an accounting for all of the
units that went into the process.
[29] Source: Publisher
Answer (D) is incorrect because a cost of production
Answer (A) is correct. If scrap material is sold on a report details the costs debited to and credited from
regular basis, e.g., daily, its value should be the manufacturing account.
recorded
either as a contra cost or as a revenue on a regular
basis, and income will be accounted for properly. If [32] Source: CIA 0577 IV-3
it
is not sold regularly and not recorded in inventory, Answer (A) is incorrect because items such as
income may be misstated. additional processing costs, competitive conditions in
sales markets, and the relative contribution margins of
Answer (B) is incorrect because failure to record all products derived from the common process must
significant scrap value can misstate income. be considered in setting selling prices.

Answer (C) is incorrect because the issue is timing Answer (B) is incorrect because items such as
the recognition of scrap value, not the account additional processing costs, competitive conditions in
used. sales markets, and the relative contribution margins of
all products derived from the common process must
Answer (D) is incorrect because this would be an be considered in determining whether to continue
example of when it is necessary to record the value producing an item.
of
scrap in inventory. Answer (C) is incorrect because management of one
department may have no control over joint costs.

[30] Source: CIA 1189 IV-7 Answer (D) is correct. Joint costs are useful for
inventory costing when two or more identifiable
Answer (A) is correct. Normal rework costs incurred products emerge from a common production
because of factors common to all units produced process. The joint costs of production must be
ordinarily are charged to factory overhead control allocated on some basis, such as relative sales value.
to
spread the costs over all good units.
[33] Source: CIA 1190 IV-10
Answer (B) is incorrect because, in a process-
costing Answer (A) is incorrect because a joint product has
application, normal rework is customarily charged relatively significant sales value when compared with
to the other products. A by-product is identifiable as an
overhead. In a job-order costing application, normal individual product only upon reaching the split-off
rework costs related to specific jobs are usually point, and it has relatively minor sales value when
charged to the work-in-process account for the compared to the other products.
given
job, not the control account. Answer (B) is incorrect because products that are
separately identifiable before the production process
Answer (C) is incorrect because rework costs are are not classified as joint products. Furthermore,
not charged to finished goods. physical volume has nothing to do with determining a
joint product. Some joint products with significant
Answer (D) is incorrect because rework costs are physical volume may not have significant sales value.
applied to good units or, in the case of abnormal
rework, charged to a loss account. Answer (C) is correct. Joint products are two or
more separate products generated by a common
process from a common input that are not separable
[31] Source: Publisher prior to the split-off point. Moreover, in contrast to
by-products, they have significant sales values in
Answer (A) is incorrect because the quantity of relation to each other either before or after additional
production report is only concerned with the units processing.
put
into and transferred out of the process in the Answer (D) is incorrect because products do not
current have to be salable at the split-off point to be
period. considered joint products; in fact, many joint
products have to be processed after the split-off
Answer (B) is correct. A quantity of production point before they can be sold.
report adds the units in BWIP and those entering
the
process during the period, and indicates the [34] Source: CIA 0585 IV-11
disposition of those units, i.e., the units completed,
spoiled, and in EWIP, respectively. Answer (A) is incorrect because treating the net
realizable value of a by-product as an addition to the
revenues of the other products attributes the
Answer (C) is incorrect because the cost of goods allocation characteristics of main products to
manufactured statement gives the cost of goods by-products.
makes the departmental overhead rate concept more
Answer (B) is incorrect because the NRV is complex and more accurate than the plant-wide
ordinarily recognized as a contra cost in the period overhead rate concept.
the by-product is produced.
Answer (C) is incorrect because, with respect to
Answer (C) is correct. Because of the relatively plant-wide and departmental overhead rates, a
small plant-wide overhead rate requires only one overhead
sales value, a cost-effective allocation method is control account. Conversely, using a departmental
used overhead rate, a separate overhead cost pool is
for by-products. The net realizable value of required for each production department, which
by-products is usually deducted from the cost of makes the departmental overhead rate concept more
the complex and more accurate than the plant-wide
main products. overhead rate concept.

Answer (D) is incorrect because recognition of a Answer (D) is correct. A firm with a relatively simple
separate net realizable value upon which to production and accounting system may use only one
allocate overhead control account with one plant-wide
some of the common costs attributes the allocation overhead rate. In a more complex environment, the
characteristics of main products to by-products. firm may need a separate overhead cost pool for
each department. The departmental rate is relatively
more complex than the plant-wide rate because of
[35] Source: Publisher the number of accounts as well as the number of rates
that must be established prior to implementation. A
Answer (A) is incorrect because cost objectives may departmental rate allows for a more accurate
be final, and should be logically related to the cost application of overhead to a specific job when
pool, preferably on a cause-and-effect basis. products require different methods of production;
e.g., product A may require more machine hours but
Answer (B) is incorrect because cost objectives may fewer labor hours than product B. If a plant-wide
be intermediate if the costs charged are later rate were used based on labor hours, product A
reallocated to another cost objective. Furthermore, would not be allocated its fair share of overhead.
cost objectives should be logically linked with the
cost pool.
[37] Source: CIA 1188 IV-4
Answer (C) is incorrect because cost objectives may
be intermediate or final, and should be logically Answer (A) is correct. Cost pools are accounts in
related to the cost pool, preferably on a which a variety of similar costs are accumulated prior
cause-and-effect basis. to allocation to cost objectives. The overhead
account is a cost pool into which various types of
Answer (D) is correct. Cost objectives are the overhead are accumulated prior to their allocation.
intermediate and final dispositions of cost pools. Indirect manufacturing costs are an element of
Cost overhead allocated to a cost pool. Ordinarily,
objectives may be intermediate as cost pools move different allocation methods are applied to variable
from their originating points to the final cost and fixed costs, thus requiring them to be separated.
objectives. Cost objectives may be final, e.g., a job, Establishing separate pools allows the determination
product, or process itself, and should be logically of dual overhead rates. As a result, the assessment of
related to the cost pool, preferably on a capacity costs, the charging of appropriate rates to
cause-and-effect basis. user departments, and the isolation of variances are
facilitated.

[36] Source: Publisher Answer (B) is incorrect because prime costs are
direct costs, and variable administrative costs are
Answer (A) is incorrect because, with respect to period, not manufacturing, costs. The question
plant-wide and departmental overhead rates, a inquires about indirect manufacturing costs.
plant-wide overhead rate requires only one
overhead Answer (C) is incorrect because establishing a
control account. Conversely, using a departmental separate pool for each assembly line worker to
overhead rate, a separate overhead cost pool is account for wages is not necessary under most cost
required for each production department, which allocation schemes.
makes the departmental overhead rate concept
more Answer (D) is incorrect because different allocation
complex and more accurate than the plant-wide methods are usually applied to variable costs and
overhead rate concept. fixed costs.

Answer (B) is incorrect because, with respect to


plant-wide and departmental overhead rates, a [38] Source: CIA 0581 IV-17
plant-wide overhead rate requires only one
overhead Answer (A) is incorrect because determining the
control account. Conversely, using a departmental income of a product or functional unit requires
overhead rate, a separate overhead cost pool is absorption (full-cost) data.
required for each production department, which
Answer (B) is correct. In the short run, management
decisions are made in reference to incremental the actual rate.
costs
without regard to fixed overhead costs because Answer (C) is incorrect because the capacity costs of
fixed the service department should be allocated by a fixed
overhead cannot be changed in the short run. Thus, overhead rate or lump-sum charge based upon the
the emphasis in the short run should be on capacity needs of the production department.
controllable costs. For example, service department
costs allocated as a part of overhead may not be Answer (D) is incorrect because the user department
controllable in the short run. does not control service department costs.

Answer (C) is incorrect because determining the


costs for the federal government's cost-plus [41] Source: Publisher
contracts
requires absorption (full-cost) data. Answer (A) is incorrect because fixed service
department cost allocation based on actual short-run
use transfers any efficiencies or inefficiencies of the
Answer (D) is incorrect because absorption costing service department to the production department.
(full-costing) is currently required for tax purposes.
Answer (B) is incorrect because allocating fixed
service department costs based on actual short-run
[39] Source: CIA 0590 IV-8 units or actual rates transfers any efficiencies or
inefficiencies of the service department to the
Answer (A) is incorrect because the physical units production department.
method is not a service department cost allocation
method. It is a method for allocating joint costs. Answer (C) is correct. The fixed costs of service
departments should be allocated to production
Answer (B) is incorrect because the step-down departments in lump-sum amounts on the basis of the
method gives only partial recognition to services service department's budgeted costs of long-term
rendered by service departments to other service capacity to serve. This basis allows the production
departments. Once a service department's costs department to develop (budget) a certain capacity
have needed from the service departments and to agree on
been allocated, the costs of subsequent service the assessment of costs. Analysis of actual results
departments are not reallocated to it. permits evaluation of the service departments' ability
to provide the estimated volume of service.
Answer (C) is incorrect because the Massachusetts
Formula is only used when there is no apparent Answer (D) is incorrect because allocating the
causal relationship. In this instance, the service department's actual costs based on actual use
relationship of services transfers any efficiencies or inefficiencies
between the service departments and the of the service department to the production
operating department.
departments is known.

Answer (D) is correct. The reciprocal method uses [42] Source: CIA 1190 IV-3
simultaneous equations to allocate costs by
explicitly Answer (A) is correct. The cause-and-effect criterion
recognizing the mutual services rendered among seeks a relationship between cost and the cost
all objective (for example, an operating division) such
departments. Because it acknowledges all sources that changes in total costs can be predicted based on
of activities of the cost objective. Thus, the number of
cost, it should be used when management is using employees in an operating division is likely to
the correlate with incurrence of costs by the personnel
results of allocations to make decisions on pricing department.
products.
Answer (B) is incorrect because square footage
would be more appropriate for allocating building and
[40] Source: Publisher maintenance costs than personnel costs.

Answer (C) is incorrect because total service years


Answer (A) is correct. The most appropriate method of employees in each division is not a basis for
of overhead allocation of variable service predicting changes in personnel department costs.
department
costs to production departments is to multiply the Answer (D) is incorrect because total book value of
actual usage of the production department by the identifiable division assets is not a basis for predicting
predetermined rate. This basis establishes the user changes in personnel department costs.
department's responsibility for the actual usage at
the
predetermined rate. [43] Source: CMA 1295 3-14

Answer (B) is incorrect because the actual rate may Answer (A) is incorrect because external failure
differ substantially from the estimated rate, and the occurs after the product is shipped; thus, statistical
production department usually has no control over quality control is not an external failure cost.
considers only the incremental costs of producing an
Answer (B) is incorrect because internal failure additional unit of product. In most cases marginal
costs costs are variable costs.
arise after poor quality has been found; statistical
quality control is designed to detect quality Answer (C) is incorrect because direct (variable)
problems. costing treats only variable costs as product costs.

Answer (C) is incorrect because statistical quality Answer (D) is incorrect because direct (variable)
control is not a training cost. costing treats only variable costs as product costs.

Answer (D) is correct. SMA 4-R lists four categories


of quality costs: prevention, appraisal, internal [46] Source: CMA 0692 3-5
failure,
and external failure (lost opportunity). Appraisal Answer (A) is incorrect because behavior in
costs response to volume changes is a factor only if the
include quality control programs, inspection, and cost object is a product.
testing.
Answer (B) is incorrect because the timing of an
expense is not a means of classifying a cost as direct
[44] Source: CMA 1290 3-1 or indirect.

Answer (A) is incorrect because theoretical capacity Answer (C) is correct. A direct cost can be
assumes all personnel and equipment will operate specifically associated with a single cost object in an
at economically feasible way. An indirect cost cannot be
peak efficiency and total plant capacity will be specifically associated with a single cost object. Thus,
used. the specific cost object influences whether a cost is
direct or indirect. For example, a cost might be
Answer (B) is correct. Practical capacity is the directly associated with a single plant. The same cost,
maximum level at which output is produced however, might not be directly associated with a
efficiently. It includes consideration of idle time particular department in the plant.
caused by human and equipment inefficiencies but
not Answer (D) is incorrect because both direct and
by inadequate sales demand. Practical capacity indirect costs can be either avoidable or unavoidable,
exceeds the other commonly used denominator depending upon the cost object.
levels
included in the calculation of the fixed factory
overhead rate. Because practical capacity will [47] Source: CMA 1292 3-1
almost
always exceed the actual use of capacity, it will Answer (A) is incorrect because a revenue center is
result evaluated on the basis of revenue generated, without
in an unfavorable production volume variance. regard to costs.
Moreover, this variance (the difference between
budgeted fixed overhead and the fixed overhead Answer (B) is correct. Cost allocation is the process
applied based on standard input allowed for the of assigning and reassigning costs to cost objects. It is
actual output) will be greatest given a practical used for those costs that cannot be directly
capacity measure. The unfavorable production associated with a specific cost object. Cost allocation
volume variance is charged to income summary, so is often used for purposes of measuring income and
the effect of using a larger denominator volume is assets for external reporting purposes. Cost
the allocation is less meaningful for internal purposes
more rapid write-off of fixed overhead (practical because responsibility accounting systems emphasize
capacity may be used for federal income tax controllability, a process often ignored in cost
purposes). allocation.

Answer (C) is incorrect because practical capacity Answer (C) is incorrect because cost allocation is not
ignores demand. necessary for cash budgeting and controlling
expenditures.
Answer (D) is incorrect because the production
volume to meet a given production level may be Answer (D) is incorrect because allocations are not
more needed for variable costing, which concerns direct,
or less than practical capacity. Horngren and Foster not indirect, costs.
call this volume the master-budget volume.

[48] Source: CMA 1292 3-3


[45] Source: CMA 1290 3-12
Answer (A) is incorrect because the graphic
Answer (A) is correct. Full absorption costing treats approach can be used to estimate a linear function.
fixed factory overhead costs as product costs. Thus,
inventory and cost of goods sold include (absorb) Answer (B) is incorrect because simple regression,
fixed factory overhead. which is based on one independent variable, is the
best means of expressing a linear cost function.
Answer (B) is incorrect because marginal costing
Answer (C) is incorrect because the high-low [51] Source: CMA 1293 3-9
method, although unsophisticated, can often give a
good approximation of a linear cost function. Answer (A) is incorrect because operation costing
differs from process costing in the treatment of
Answer (D) is correct. Regression analysis can be materials.
used to find an equation for the linear relationship
among variables. However, multiple regression is Answer (B) is correct. Operation costing is a hybrid
not of job-order and process costing systems wherein
used to generate an equation of the type Y = a + materials are allocated on the basis of batches of
bX production. It is used by companies that manufacture
because multiple regression has more than one goods that undergo some similar and some dissimilar
independent variable. In other words, a multiple processes. Operation costing accumulates total
regression equation would take the form: Y = a + conversion costs and determines a unit conversion
bX1 + cX2 + dX3 + . . . . cost for each operation. However, direct materials
costs are charged specifically to products or batches
as in job-order systems.
[49] Source: CMA 1292 3-4
Answer (C) is incorrect because operation costing
Answer (A) is correct. Joint products are created differs from process costing in the treatment of
from processing a common input. Common costs materials.
are
incurred prior to the split-off point and cannot be Answer (D) is incorrect because overhead allocations
identified with a particular joint product. As a are made in operation costing.
result,
common costs are irrelevant to the timing of sale.
However, separable costs incurred after the split-off [52] Source: CMA 0694 3-5
point are relevant because, if incremental revenues
exceed the separable costs, products should be Answer (A) is incorrect because rent is an example
processed further, not sold at the split-off point. of fixed factory overhead.

Answer (B) is incorrect because joint costs Answer (B) is incorrect because property taxes are
(common an example of fixed factory overhead.
costs) have no effect on the decision as to when to
sell a product. Answer (C) is incorrect because depreciation is an
example of fixed factory overhead.
Answer (C) is incorrect because sales salaries for
the Answer (D) is correct. A fixed cost is one that
production period do not affect the decision. remains unchanged within the relevant range for a
given period despite fluctuations in activity. Such
Answer (D) is incorrect because purchase costs are items as rent, property taxes, depreciation, and
joint costs. supervisory salaries are normally fixed costs because
they do not vary with changes in production. Power
costs, however, are at least partially variable because
[50] Source: CMA 0693 3-5 they increase as usage increases.

Answer (A) is incorrect because overhead costs as


well as prime costs (direct materials and labor) are [53] Source: CMA 0694 3-8
included in inventory.
Answer (A) is incorrect because committed costs
Answer (B) is incorrect because materials costs are have not been amortized.
also included.
Answer (B) is incorrect because discretionary costs
Answer (C) is incorrect because inventory costs are are those that do not have a clear cause and effect
expensed when the goods are sold, not when they relationship between inputs and outputs.
are transferred to finished goods.
Answer (C) is incorrect because engineered costs are
Answer (D) is correct. Under an absorption costing those that have a measurable relationship between
system, inventoriable (product) costs include all inputs and outputs.
costs
necessary for good production. These include direct Answer (D) is correct. Committed costs result when
materials and conversion costs (direct labor and a going concern holds fixed assets such as property,
overhead). Both fixed and variable overhead is plant, and equipment. The related committed costs
included in inventory under an absorption costing include depreciation, long-term lease payments, and
system. Inventoriable costs are treated as assets insurance. Such costs establish the present level of
until operating capacity and cannot be altered in the short
the products are sold because they represent run.
future
economic benefits. These costs are expensed at the
time of sale. [54] Source: CMA 0694 3-9

Answer (A) is incorrect because variable costs are


fixed per unit; they do not fluctuate. Fixed costs per characterized by uncertainty about the relationship
unit change as production changes. between input and the value of the related output.
Examples are advertising and R&D costs.
Answer (B) is correct. Fixed costs remain
unchanged Answer (C) is incorrect because a sunk cost is a past
within the relevant range for a given period despite cost or a cost that the entity has irrevocably
fluctuations in activity, but per unit fixed costs do committed to incur. Because it is unavoidable, it is not
change as the level of activity changes. Thus, fixed relevant to future decisions.
costs are fixed in total but vary per unit as activity
changes. Total variable costs vary directly with Answer (D) is correct. A mixed cost is a combination
activity. They are fixed per unit, but vary in total. of fixed and variable elements. Consequently, the $27
of total overhead cost is mixed because it contains
Answer (C) is incorrect because all costs are both fixed overhead and variable overhead.
variable
in the long term.
[58] Source: CMA 1294 3-4
Answer (D) is incorrect because unit variable costs
are fixed in the short term. Answer (A) is incorrect because conversion costs are
composed of direct labor and factory overhead, that
is, costs incurred to convert materials into a finished
[55] Source: CMA 1294 3-1 product.

Answer (A) is incorrect because conversion costs Answer (B) is correct. Before they are incurred,
consist of direct labor and overhead. R&D costs are often considered to be discretionary.
However, Huron's R&D costs have already been
Answer (B) is incorrect because separable costs are incurred. Thus, they are sunk costs. A sunk cost is a
incurred beyond the point at which jointly produced past cost or a cost that the entity has irrevocably
items become separately identifiable. committed to incur. Because it is unavoidable, it is not
relevant to future decisions.
Answer (C) is incorrect because committed costs
result when an entity holds fixed assets; examples Answer (C) is incorrect because relevant costs are
of expected future costs that vary with the action taken.
committed costs include long-term lease payments A cost that has already been incurred is not relevant
and depreciation. to future decisions.

Answer (D) is correct. Raw materials and direct Answer (D) is incorrect because avoidable costs may
labor (such as machining and assembly) are a be eliminated by not engaging in an activity or by
manufacturer's prime costs. performing it more efficiently.

[56] Source: CMA 1294 3-2 [59] Source: CMA 1294 3-5

Answer (A) is incorrect because contribution margin Answer (A) is correct. A discretionary cost (a
ratio is the ratio of contribution margin (sales - managed or program cost) results from a periodic
variable costs) to sales. decision about the total amount to be spent. It is also
characterized by uncertainty about the relationship
Answer (B) is correct. Gross profit is the difference between input and the value of the related output.
between sales price and the full absorption cost of Examples are advertising and R & D costs.
goods sold.
Answer (B) is incorrect because an opportunity cost
Answer (C) is incorrect because contribution is the maximum benefit forgone by using a scarce
(margin) is the difference between unit selling price resource for a given purpose. It is the benefit
and unit variable costs. Fixed costs are not provided by the next best use of a particular
considered. resource.

Answer (D) is incorrect because the gross profit


margin ratio equals gross profit divided by sales. Answer (C) is incorrect because committed costs are
those for which management has made a long-term
commitment. They typically result when a firm holds
[57] Source: CMA 1294 3-3 fixed assets. Examples include long-term lease
payments and depreciation.
Answer (A) is incorrect because a carrying cost is
the Answer (D) is incorrect because incremental costs
cost of carrying inventory; examples are insurance are the differences in costs between two decision
and rent on warehouse facilities. choices.

Answer (B) is incorrect because a discretionary cost


(a managed or program cost) results from a [60] Source: CMA 1294 3-6
periodic
decision about the total amount to be spent. It is Answer (A) is incorrect because joint (common)
also costs are incurred in the production of two or more
inseparable products up to the point at which the [63] Source: CMA 1295 3-16
products become separable.
Answer (A) is incorrect because the step method can
Answer (B) is correct. Committed costs are those be used on a single- or dual-rate basis.
for
which management has made a long-term Answer (B) is incorrect because the reciprocal
commitment. They typically result when a firm method can be used on a single- or dual-rate basis.
holds
fixed assets. Examples include long-term lease Answer (C) is incorrect because the direct method
payments and depreciation. Committed costs are can be used on a single-or dual-rate basis.
typically fixed costs.
Answer (D) is correct. The single-rate method
Answer (C) is incorrect because an opportunity cost combines fixed and variable costs. However, dual
is the maximum benefit forgone by using a scarce rates are preferable because they allow variable costs
resource for a given purpose; it is the benefit to be allocated on a different basis from fixed costs.
provided by the next best use of a particular
resource.

Answer (D) is incorrect because prime costs are [64] Source: CMA 1295 3-26
composed of raw material and direct labor costs.
Answer (A) is incorrect because direct costing is a
system that treats fixed costs as period costs; in other
[61] Source: CMA 0695 3-9 words, production costs consist only of variable
costs, while fixed costs are expensed as incurred.
Answer (A) is incorrect because residual income is
the excess of earnings over an imputed charge for Answer (B) is correct. An activity-based costing
the (ABC) system identifies the causal relationship
given investment base. between the incurrence of cost and the underlying
activities that cause those costs. Under an ABC
Answer (B) is incorrect because a marginal rate of system, costs are applied to products on the basis of
return is the return on the next investment. resources consumed (drivers).

Answer (C) is correct. The margin of safety is the Answer (C) is incorrect because cycle time is the
excess of budgeted revenues over breakeven period from the time a customer places an order to
revenues. It is considered in sensitivity analysis. the time that product is delivered.

Answer (D) is incorrect because a target or hurdle Answer (D) is incorrect because variable costing is
rate of return is the required rate of return. It is also the same as direct costing, which expenses fixed
known as the discount rate or the opportunity cost costs as incurred.
of
capital.
[65] Source: CMA 0694 3-1

[62] Source: CMA 1295 3-15 Answer (A) is incorrect because product costing is
an objective of a cost accounting system.
Answer (A) is incorrect because the direct method
does not recognize the fact that service Answer (B) is incorrect because department
departments efficiency is an objective of a cost accounting system.
might provide services to each other; all costs are
assigned directly to production departments. Answer (C) is incorrect because inventory valuation
is an objective of a cost accounting system.
Answer (B) is incorrect because the variable
method Answer (D) is correct. A cost accounting system has
is a nonsense term as used here.
numerous objectives, including product costing,
Answer (C) is correct. The three most common assessing departmental efficiency, inventory valuation,
methods of allocating service department costs are income determination, and planning, evaluating, and
the direct method, the step method, and the controlling operations. Determining sales commissions
reciprocal method (also called the simultaneous is not an objective of a cost accounting system
equations method). The reciprocal method is because such commissions are based on sales, not
theoretically the preferred method because it costs.
recognizes reciprocal services among service
departments.
[66] Source: CMA 0694 3-3
Answer (D) is incorrect because the linear method
is Answer (A) is incorrect because all factory overhead
not one of the methods used to allocate is included in conversion costs, not just indirect labor.
departmental
costs. Answer (B) is incorrect because direct materials are
not an element of conversion costs; they are a prime
cost.
Answer (C) is correct. Conversion costs consist of Answer (C) is correct. Total variable cost changes
direct labor and factory overhead. These are the when changes in the activity level occur within the
costs of converting raw materials into a finished relevant range. The cost per unit for a variable cost is
product. constant for all activity levels within the relevant
range. Thus, if the activity volume increases within the
Answer (D) is incorrect because direct labor is also relevant range, total variable costs will increase. A
an element of conversion costs. fixed cost does not change when volume changes
occur in the activity level within the relevant range. If
the activity volume increases within the relevant
[67] Source: CIA 1196 III-82 range, total fixed costs will remain unchanged.

Answer (A) is incorrect because $1.66 is the Answer (D) is incorrect because the variable cost per
average unit and the total fixed costs will remain constant if the
cost over 4 months. activity level increases within the relevant range.

Answer (B) is correct. Using the high-low method,


the variable and fixed costs for shipping can be [70] Source: CIA 0596 III-94
calculated. The difference in cost levels divided by
the difference in unit volume equals the variable Answer (A) is incorrect because a variable cost
cost would remain a constant percentage of standard
per unit of $1.40 [($114,000 - $93,000) ・(70,000 - dollars shipped.
55,000)]. The variable cost for 70,000 units is
$98,000 ($1.40 x 70,000). Subtracting the variable Answer (B) is incorrect because a fixed cost would
cost from total shipping cost results in the fixed be a lower percentage when standard dollars shipped
cost were high than when they were low.
of $16,000 ($114,000 - $98,000).
Answer (C) is incorrect because a semi-fixed cost as
Answer (C) is incorrect because $30,000 per month a percentage would move up and down with
plus $35.00 per sales order is based on orders standard dollars shipped, with a base level higher
instead of sales, and the March shipping costs are than zero percent.
incorrectly matched with the April orders.
Answer (D) is correct. There is no systematic
Answer (D) is incorrect because $58,000 per month relationship between standard dollars shipped and the
plus $23.33 per sales order is based on orders. percentage of scrap.

[68] Source: CIA 1196 III-83 [71] Source: CMA 0696 3-1

Answer (A) is incorrect because advertising cost is Answer (A) is incorrect because normal capacity is
considered fixed. the long-term average level of activity that will
approximate demand over a period that includes
Answer (B) is incorrect because sales salaries are seasonal, cyclical, and trend variations.
considered fixed in terms of dollar sales.
Answer (B) is incorrect because expected annual
Answer (C) is incorrect because advertising and activity is an approximation of actual volume levels
sales for a specific year.
salaries are considered fixed costs in terms of
dollar Answer (C) is correct. Theoretical (ideal) capacity is
sales. the maximum capacity given continuous operations
with no holidays, downtime, etc. It assumes perfect
Answer (D) is correct. Both advertising and sales efficiency at all times. Consequently, it can never be
salaries should be classified as fixed costs. The attained and is not a reasonable estimate of actual
advertising was constant for 3 of the 4 months and volume.
would be considered fixed in terms of dollar sales.
Sales salaries also did not vary with dollar sales. Answer (D) is incorrect because master-budget
capacity is the expected level of activity used for
budgeting for a given year.
[69] Source: CIA 1194 III-50

Answer (A) is incorrect because the variable cost [72] Source: CMA 0696 3-2
per
unit and the total fixed costs will remain constant if Answer (A) is correct. The choice of practical rather
the than master budget capacity as the denominator level
activity level increases within the relevant range. will result in a lower absorption costing net income.
Practical capacity is the maximum level at which
Answer (B) is incorrect because the variable cost output is produced efficiently, with an allowance for
per unavoidable interruptions, for example, for holidays
unit and the total fixed costs will remain constant if and scheduled maintenance. Because this level will be
the higher than master-budget (expected) capacity, its
activity level increases within the relevant range. use will usually result in the underapplication of fixed
factory overhead. For example, given costs of target price, which is the expected market price given
$100,000 and master-budget capacity of 800,000 the company's knowledge of its customers and
units, $.125 per unit is the application rate. If competitors. Subtracting the unit target profit margin
practical capacity is 1,250,000 units, the determines the long-term target cost. If this cost is
application lower than the full cost, the company may need to
rate is $.08 per unit. If actual production is 800,000 adopt comprehensive cost-cutting measures. For
units, fixed factory overhead will not be over- or example, in the furniture industry, certain price points
underapplied given the use of master-budget are popular with buyers: a couch might sell better at
capacity. However, there will be $36,000 (450,000 $400 than at $200 because consumers question the
units x $.08) of underapplied fixed factory quality of a $200 couch and thus will not buy the
overhead lower-priced item. The result is that furniture
if practical capacity is the denominator level. manufacturers view $400 as the target price of a
Consequently, given t hat the beginning inventory couch, and the cost must be somewhat lower.
is
zero and that production exceeded sales, less fixed Answer (B) is incorrect because all product cost
factory overhead will be inventoried at the lower categories are addressed by target costing.
practical capacity rate than at the master-budget
rate.
The effect is that master-budget net income will be Answer (C) is incorrect because all product cost
greater. categories are addressed by target costing.

Answer (B) is incorrect because a normal capacity Answer (D) is incorrect because the manner in which
rate results in a larger ending inventory and a raw materials costs are accounted for is irrelevant.
greater
net income than a theoretical or practical capacity
rate. [75] Source: CMA 0696 3-16

Answer (C) is incorrect because the master-budget Answer (A) is correct. SMA 4X states that
rate exceeds the theoretical capacity rate. It results value-chain analysis for assessing competitive
in advantage is an integral part of the strategic planning
a greater ending inventory and a greater net process. Value-chain analysis is a continuous process
income. of gathering, evaluating, and communicating
information for business decision making. A value
Answer (D) is incorrect because a practical capacity chain depicts how customer value accumulates along
rate results in a lower ending inventory and a lower a chain of activities that lead to an end product or
net income than a normal capacity rate. service. A value chain consists of the activities
required to research and develop, design, produce,
market, deliver, and support its product. Extended
[73] Source: CMA 0696 3-12 value-chain analysis expands the view of the parties
involved to include those upstream (e.g., suppliers)
Answer (A) is correct. The breakeven point is the and downstream (e.g., customers).
level of sales at which revenues equal the sum of
variable and fixed costs. Consequently, the Answer (B) is incorrect because process value
contribution margin equals fixed costs at the analysis relates to a single process.
breakeven point. Because this relationship is true,
the Answer (C) is incorrect because computer-integrated
breakeven point in units sold can be determined by manufacturing uses computers to control all aspects
dividing fixed costs by the difference between unit of manufacturing in a single location.
selling price and unit variable cost (unit
contribution Answer (D) is incorrect because ABC identifies the
margin). activities associated with cost incurrence and the
drivers of those activities. Costs are then assigned to
Answer (B) is incorrect because the profit margin is cost objects based on the demands they make on
the difference between revenues and cost of goods activities.
sold.

Answer (C) is incorrect because operating profit is [76] Source: CMA 0696 3-17
the difference between operating revenues and
expenses. Answer (A) is incorrect because excess capacity is
unused capacity.
Answer (D) is incorrect because the contribution
margin ratio equals unit selling price minus unit Answer (B) is incorrect because manufacturing lead
variable cost, divided by unit selling price. Dividing (cycle) time is the sum of setup time and
fixed costs by the CMR yields the breakeven point manufacturing time for a customer order. It is a
in component of customer response time.
dollars.
Answer (C) is correct. Practical capacity is the
maximum level at which output is produced
[74] Source: CMA 0696 3-15 efficiently, with an allowance for unavoidable
interruptions, for example, for holidays and scheduled
Answer (A) is correct. Target costing begins with a maintenance. Because this level will be higher than
expected capacity, its use will ordinarily result in Answer (D) is incorrect because factory overhead is
underapplied fixed factory overhead. an indirect cost that is an element of conversion costs.

Answer (D) is incorrect because theoretical


capacity [80] Source: CMA 1296 3-29
makes no allowance for unavoidable interruptions.
Answer (A) is correct. Life-cycle costing estimates a
product's revenues and expenses over its expected
[77] Source: CMA 0696 3-18 life cycle. This approach is especially useful when
revenues and related costs do not occur in the same
Answer (A) is incorrect because depreciation is a periods. It emphasizes the need to price products to
factory overhead cost and therefore is a conversion cover all costs, not just those for production. Hence,
cost. costs are determined for all value-chain categories:
upstream (R&D, design), manufacturing, and
Answer (B) is correct. Conversion costs are downstream (marketing, distribution, and customer
necessary to convert raw materials into finished service). The result is to highlight upstream and
products. They include all manufacturing costs, for downstream costs in the cost planning process that
example, direct labor and factory overhead, other often receive insufficient attention.
than direct materials.
Answer (B) is incorrect because the life-cycle model
Answer (C) is incorrect because indirect labor is a includes the upstream (R&D and design) and
factory overhead cost and therefore is a conversion downstream (marketing, distribution, and customer
cost. service) elements of the value chain as well as
manufacturing costs.
Answer (D) is incorrect because indirect materials
are factory overhead costs and therefore are Answer (C) is incorrect because the life-cycle model
conversion costs. includes the upstream (R&D and design) and
downstream (marketing, distribution, and customer
service) elements of the value chain as well as
[78] Source: CMA 0696 3-20 manufacturing costs.

Answer (A) is incorrect because revenues minus Answer (D) is incorrect because life-cycle costing
cost emphasizes the significance of locked-in costs, target
of goods sold is gross profit (margin). costing, and value engineering for pricing and cost
control. Thus, cost savings at all stages of the life
Answer (B) is incorrect because nonmanufacturing cycle are important.
variable costs are also part of the calculation.

Answer (C) is incorrect because a direct cost is a [81] Source: Publisher


cost that can be feasibly associated with a single
cost Answer (A) is incorrect because scrap consists of
object. raw materials left over from the production cycle but
still usable for purposes other than those for which it
Answer (D) is correct. Contribution margin is the was originally intended. Scrap may be sold to outside
excess of revenues over all variable costs customers, usually for a nominal amount, or may be
(including used for a different production process.
both manufacturing and nonmanufacturing variable
costs) that vary with an output-related cost driver. Answer (B) is correct. By-products are products of
The contribution margin equals the revenues that relatively small total value that are produced
contribute toward covering the fixed costs and simultaneously from a common manufacturing
providing a net income. process with products of greater value and quantity
(joint products).

[79] Source: CMA 1296 3-3 Answer (C) is incorrect because waste is the amount
of raw materials left over from a production process
Answer (A) is incorrect because conversion cost or production cycle for which there is no further use.
pricing does not place any emphasis on raw Waste is usually not salable at any price and must be
materials discarded.
cost.
Answer (D) is incorrect because abnormal spoilage is
Answer (B) is correct. Conversion costs consist of spoilage that is not expected to occur under normal,
direct labor and factory overhead, the costs of efficient operating conditions. The cost of abnormal
converting raw materials into finished goods. spoilage should be separately identified and reported
Normally, a company does not consider only to management. Abnormal spoilage is typically
conversion costs in making pricing decisions, but if treated as a period cost (a loss) because of its
the customer were to furnish the raw materials, unusual nature.
conversion cost pricing would be appropriate.

Answer (C) is incorrect because direct labor is an [82] Source: Publisher


element of conversion costs.
Answer (A) is incorrect because target costing
(pricing) begins with a target price, which is the best alternative use of that resource.
expected market price given the company's
knowledge of its customers and competitors. Answer (C) is correct. A marginal cost is the sum of
Subtracting the unit target profit margin the costs necessary to effect a one-unit increase in the
determines activity level.
the long-term target cost.
Answer (D) is incorrect because differential (or
Answer (B) is correct. Kaizen costing supports the incremental) cost is the difference in total cost
cost reduction process in the manufacturing phase between two decisions.
of
existing products. Kaizen is a Japanese word that
refers to continuous improvement. Kaizen makes [85] Source: Publisher
improvements through the accumulations of small
betterment activities rather than innovative Answer (A) is incorrect because activity-based
improvements. costing "identifies the causal relationship between the
incurrence of cost and activities, determines the
Answer (C) is incorrect because variable (direct) underlying driver of the activities, establishes cost
costing considers only variable manufacturing costs pools related to individual drivers, develops costing
to rates, and applies cost to product on the basis of
be product costs, i.e., inventoriable. Fixed resources consumed (drivers)" (SMA 2A).
manufacturing costs are considered period costs
and Answer (B) is incorrect because variable (direct)
are expensed as incurred. costing considers only variable manufacturing costs to
be product costs, i.e., inventoriable. Fixed
Answer (D) is incorrect because process costing is manufacturing costs are considered period costs and
used to assign costs to similar products that are are expensed as incurred.
mass
produced on a continuous basis. Costs are Answer (C) is correct. Backflush costing is often
accumulated by departments or cost centers rather used with a just-in-time (JIT) inventory system. It
than by jobs, work-in-process is stated in terms of delays costing until goods are finished. Standard
equivalent units, and unit costs are established on costs are then flushed backward through the system
a to assign costs to products. The result is that detailed
departmental basis. Process costing is an averaging tracking of costs is eliminated. The system is best
process that calculates the average cost of all suited to companies that maintain low inventories
units. because costs then flow directly to cost of goods
sold.

[83] Source: CMA 0697 3-1 Answer (D) is incorrect because absorption costing
(sometimes called full absorption costing) treats all
Answer (A) is incorrect because direct labor is also manufacturing costs as product costs. These costs
a include variable and fixed manufacturing costs
product cost. whether direct or indirect. Thus, fixed manufacturing
overhead is inventoried. Compare with variable
Answer (B) is incorrect because a period cost is costing.
expensed when incurred. Direct labor cost is
inventoriable.
[86] Source: CMA 1295 3-28
Answer (C) is incorrect because direct labor is also
a Answer (A) is incorrect because gross operating
prime cost. profit is the net result after deducting all manufacturing
costs from sales, including both fixed and variable
Answer (D) is correct. Direct labor is both a product costs.
cost and a prime cost. Product costs are incurred to
produce units of output and are deferred to future Answer (B) is incorrect because net profit is the
periods to the extent that output is not sold. Prime remainder after deducting from revenue all costs,
costs are defined as direct materials and direct both fixed and variable.
labor.
Answer (C) is incorrect because the breakeven point
is the level of sales that equals the sum of fixed and
[84] Source: Publisher variable costs.

Answer (A) is incorrect because margin of safety is Answer (D) is correct. Contribution margin is
the excess of budgeted revenues over the calculated by subtracting all variable costs from sales
breakeven revenue. It represents the portion of sales that is
point. available for covering fixed costs and profit.

Answer (B) is incorrect because opportunity cost is


the maximum benefit forgone by using a scarce
resource for a given purpose. It is the benefit, for [87] Source: Publisher
example, the contribution to income, provided by
the Answer (A) is incorrect because cost accounting is
concerned with more than just reporting to be used
in [90] Source: Publisher
making nonroutine decisions.
Answer (A) is incorrect because scrap consists of
Answer (B) is incorrect because cost accounting raw materials left over from the production cycle but
also still usable for purposes other than those for which it
provides information for internal reporting. was originally intended. Scrap may be sold to outside
customers, usually for a nominal amount, or may be
Answer (C) is correct. Cost accounting is a used for a different production process.
combination of (1) managerial accounting in the
sense Answer (B) is incorrect because abnormal spoilage is
that its purpose can be to provide internal reports spoilage that is not expected to occur under normal,
for efficient operating conditions. The cost of abnormal
use in planning and control and in making spoilage should be separately identified and reported
nonroutine to management. Abnormal spoilage is typically
decisions, and (2) financial accounting because its treated as a period cost (a loss) because of its
product-costing function satisfies external reporting unusual nature.
requirements for reporting to shareholders,
government, and various outside parties. Answer (C) is correct. Waste is the amount of raw
materials left over from a production process or
Answer (D) is incorrect because managerial production cycle for which there is no further use.
accounting entails internal reporting for use in Waste is usually not salable at any price and must be
planning discarded.
and controlling routine operations.
Answer (D) is incorrect because normal spoilage is
the spoilage that occurs under normal operating
[88] Source: CMA 1293 3-1 conditions. It is essentially uncontrollable in the short
run. Normal spoilage arises under efficient operations
Answer (A) is correct. According to SMA 2A, a and is treated as a product cost.
cost driver is "a measure of activity, such as direct
labor hours, machine hours, beds occupied,
computer time used, flight hours, miles driven, or [91] Source: Publisher
contracts, that is a causal factor in the incurrence
of Answer (A) is incorrect because gross margin (profit)
cost to an entity." It is a basis used to assign costs is the difference between sales and the absorption
to cost of goods sold. It should be contrasted with
cost objects. contribution margin (sales - variable costs) and profit
margin (income ・revenue).
Answer (B) is incorrect because cost drivers are
measures of activities that cause the incurrence of Answer (B) is incorrect because residual income is
costs. the excess of the return on an investment over a
targeted amount equal to an imputed interest charge
Answer (C) is incorrect because cost drivers are not on invested capital. The rate used is ordinarily the
accounting measurements but measures of weighted-average cost of capital. Some enterprises
activities prefer to measure managerial performance in terms of
that cause costs. the amount of residual income rather than the
percentage ROI. The principle is that the enterprise is
Answer (D) is incorrect because, although cost expected to benefit from expansion as long as
drivers may be used to assign costs, they are not residual income is earned. Using a percentage ROI
necessarily mechanical. For example, a cost driver approach, expansion might be rejected if it lowered
for pension benefits is employee salaries. ROI even though residual income would increase.

Answer (C) is incorrect because practical capacity is


[89] Source: Publisher the maximum level at which output is produced
efficiently. It allows for unavoidable delays in
Answer (A) is correct. Abnormal spoilage is spoilage production for maintenance, holidays, etc. Use of
that is not expected to occur under normal, practical capacity as a denominator value usually
efficient results in underapplied overhead because it always
operation conditions. exceeds the actual use of capacity.

Answer (B) is incorrect because actual spoilage is Answer (D) is correct. Partial productivity equals the
the quantity of output divided by the quantity of one
spoilage that occurred. input.

Answer (C) is incorrect because normal spoilage is


expected to occur. [92] Source: CIA 1195 III-79

Answer (D) is incorrect because residual spoilage is Answer (A) is incorrect because both measures
a represent the results for a single period.
nonsense term.
Answer (B) is correct. Residual income equals
earnings in excess of a minimum desired return. Answer (D) is incorrect because indirect costs cannot
Thus, be specifically associated with a given cost object in
it is measured in dollars. If performance is an economically feasible way. They are also defined
evaluated as costs that are not directly identified with one final
using ROI, a manager may reject a project that cost object but that are identified with two or more
exceeds the minimum return if the project will final cost objects or with at least one intermediate
decrease overall ROI. For example, given a target cost object.
rate of 20%, a project with an ROI of 22% might be
rejected if the current ROI is 25%.

Answer (C) is incorrect because the target rate for [95] Source: Publisher
ROI is the same as the imputed interest rate used
in Answer (A) is incorrect because periodic inventory
the residual income calculation. systems rely on physical counts to determine
quantities.
Answer (D) is incorrect because the same
investment Answer (B) is incorrect because step-down method
base should be employed in both methods. of service department cost allocation is a sequential
(but not a reciprocal) process. These costs are
allocated to other service departments as well as to
[93] Source: CIA 0593 IV-14 users. The starting point may be the service
department that rendered the greatest percentage of
Answer (A) is incorrect because budgeted, not its services to other service departments or that
actual, UCM is used to calculate this variance. incurred the greatest dollar amount of services to
other service departments.
Answer (B) is incorrect because the flexible budget
volume is the actual volume, resulting in a zero Answer (C) is correct. Perpetual inventory records
variance. provide for continuous record keeping of the
quantities of inventory (and possibly unit costs and/or
Answer (C) is incorrect because budgeted, not total costs). This method requires a journal entry
actual, UCM is used to calculate this variance. every time items are added to or taken from
inventory.
Answer (D) is correct. For a single-product
company, the sales volume variance is the Answer (D) is incorrect because reciprocal method
difference uses simultaneous equations to allocate each service
between flexible budget and master budget sales department's costs among the departments providing
quantity, times master budget unit contribution mutual services before reallocation to other users.
margin
(UCM). This amount can also be calculated for each
product in a sales mix, and the results are added to [96] Source: Publisher
determine the total sales volume variance. This
variance may be further decomposed into quantity Answer (A) is correct. Price (rate) variance equals
and mix variances. the difference between the actual and standard price
of an input, multiplied by the actual quantity.

[94] Source: Publisher Answer (B) is incorrect because controllable variance


in two-way analysis is the part of the total factory
Answer (A) is incorrect because opportunity cost is overhead variance not attributable to the volume
the maximum benefit forgone by using a scarce variance.
resource for a given purpose. It is the benefit, for
example, the contribution to income, provided by Answer (C) is incorrect because spending variance is
the an overhead variance. For variable overhead, it is the
best alternative use of that resource. difference between actual costs and the product of
the actual activity and the budgeted application rate.
Answer (B) is correct. Avoidable costs are those For fixed overhead, the spending (also known as the
that budget or flexible budget) variance is the difference
between actual and budgeted fixed costs. In
may be eliminated by not engaging in an activity or three-way analysis, the two spending variances
by isolated in four-way analysis are combined. In
performing it more efficiently. two-way analysis, the two spending variances and the
variable overhead efficiency variance are combined.
Answer (C) is incorrect because cost driver "is a
measure of activity, such as direct labor hours, Answer (D) is incorrect because quantity (usage)
machine hours, beds occupied, computer time variance is an efficiency variance for direct materials.
used,
flight hours, miles driven, or contracts, that is a
causal [97] Source: Publisher
factor in the incurrence of cost to an entity" (SMA
2A).
Answer (A) is incorrect because actual costing is
based on actual rates and quantities for indirect as
well as direct costs. subtracting the desired profit margin, the long-run
target cost is known. If current costs are too high to
Answer (B) is correct. Benchmarking (also called allow an acceptable profit, cost-cutting measures are
competitive benchmarking or best practices) implemented or the product is abandoned. The
compares one's own product, service, or practice assumption is that the target price is a constraint.
with the best known similar activity. The objective
is
to measure the key outputs of a business process [99] Source: Publisher
or
function against the best and to analyze the Answer (A) is incorrect because the controller (or
reasons
for the performance difference. Benchmarking comptroller) is a financial officer having responsibility
applies for the accounting functions (management and
to services and practices as well as to products and financial) as well as budgeting and internal control.
is
an ongoing systematic process. It entails both Answer (B) is incorrect because the chief financial
quantitative and qualitative measurements that officer (CFO) is the senior officer empowered with
allow oversight of the entire financial operations of a
both an internal and an external assessment. company.

Answer (C) is incorrect because backflush costing is Answer (C) is incorrect because the controller (or
often used with a just-in-time (JIT) inventory comptroller) is a financial officer having responsibility
system. for the accounting functions (management and
It delays costing until goods are finished. Standard financial) as well as budgeting and internal control.
costs are then flushed backward through the
system Answer (D) is correct. The treasurer has
to assign costs to products. The result is that responsibility for safeguarding financial assets
detailed (including the management of cash) and arranging
tracking of costs is eliminated. The system is best financing.
suited to companies that maintain low inventories
because costs then flow directly to cost of goods
sold. [100] Source: CMA Samp Q3-6

Answer (D) is incorrect because budgeting is the Answer (A) is incorrect because normal spoilage
formal quantification of management's plans. arises more frequently from factors that are inherent
Budgets in the manufacturing process.
are usually expressed in quantitative terms and are
used to motivate management and evaluate its Answer (B) is incorrect because abnormal spoilage
performance in achieving goals. In this sense, costs are treated as a loss, and normal spoilage costs
standards are established. are inventoried.

Answer (C) is correct. Spoiled goods are defective


items that cannot be feasibly reworked. Traditional
[98] Source: CMA 0697 3-5 cost accounting systems distinguish between normal
and abnormal spoilage because, in some operations,
Answer (A) is incorrect because target pricing is a degree of spoilage is viewed as inevitable.
used However, organizations that have adopted rigorous
on products that have not yet been developed. approaches to quality regard normal spoilage as
minimal or even nonexistent. Thus, all spoilage may
Answer (B) is incorrect because target pricing be identified as abnormal. Normal spoilage occurs
considers all costs in the value chain. under normal, efficient operating conditions. It is
spoilage that is uncontrollable in the short run and
Answer (C) is incorrect because target pricing can therefore should be expressed as a function of good
be output (treated as a product cost). Accordingly,
used in any situation, but it is most likely to normal spoilage is assigned to all good units in
succeed process costing systems, that is, all units that have
when costs can be well controlled. passed the inspection point at which the spoilage was
detected. If normal spoilage is attributable to a
Answer (D) is correct. Target pricing and costing specific job, only the disposal value of the normally
may result in a competitive advantage because it is spoiled goods is removed from work-in-process,
a thereby assigning the cost of normal spoilage to the
customer-oriented approach that focuses on what good units remaining in the specific job. Abnormal
products can be sold at what prices. It is also spoilage is not expected to occur under normal,
advantageous because it emphasizes control of efficient operating conditions. The cost of abnormal
costs spoilage should be separately identified and reported.
prior to their being locked in during the early links Abnormal spoilage is typically treated as a period
in cost (a loss) because it is unusual.
the value chain. The company sets a target price
for a Answer (D) is incorrect because the tighter the
potential product reflecting what it believes standards, the more likely that any spoilage will be
consumers will pay and competitors will do. After deemed to be abnormal.
Consequently, the NRV for CBL is $600,000
($800,000 - $200,000). CBL contributes 75%
[101] Source: CMA 0690 4-6 [$600,000 ・($200,000 + $600,000)] of the
realizable value and should bear an equal proportion
Answer (A) is incorrect because $75,000 is the of the $300,000 of joint costs, or $225,000. The
amount allocated to MSB based on the relative total cost of CBL is $425,000 ($225,000 +
sales $200,000), and the unit cost is $5.3125 ($425,000 ・
value method. 80,000).

Answer (B) is correct. Given 60,000 units of MSB


and 90,000 units of CBL (a total of 150,000 units),
CBL makes up 60% of the total quantity (90,000 ・ [104] Source: CMA 0690 4-9
150,000). Hence, CBL should be charged with 60%
of the joint costs (60% x $300,000 = $180,000). Answer (A) is incorrect because the contribution
from the joint milling process would be $80,000
Answer (C) is incorrect because $225,000 is the lower ($200,000 - $120,000).
amount allocated to CBL based on the relative
sales Answer (B) is incorrect because the contribution
value method. from the joint milling process would be $80,000
lower ($200,000 - $120,000).
Answer (D) is incorrect because $120,000 equals
the joint costs that should be allocated to MSB Answer (C) is incorrect because the contribution
under from the joint milling process would be $80,000
the physical quantities method. lower ($200,000 - $120,000).

Answer (D) is correct. MSB sells for $120,000 ($2


[102] Source: CMA 0690 4-7 x 60,000 units) at the split-off point. By processing
further, the company earns $200,000 ($300,000 in
Answer (A) is correct. At $2 each, the 60,000 units sales - $100,000 in preservative costs). Thus, the
of MSL sell for $120,000. At $4 each, the 90,000 contribution from the joint milling process would be
units of CBL sell for $360,000. Because 25% $80,000 lower ($200,000 - $120,000) if MSB is
[$120,000 ・($120,000 + $360,000)] of the total sold at the split-off point.
sales are produced by MSB, it should absorb 25%
of the joint cost. Thus, $75,000 (25% x $300,000
joint cost) is allocated to MSB. [105] Source: CMA 0689 4-21

Answer (B) is incorrect because $180,000 is the Answer (A) is incorrect because $168,000 is the
joint cost allocated to CBL based on physical joint cost allocated to Silken Skin on a
quantities. physical-quantity basis.

Answer (C) is incorrect because $225,000 is the Answer (B) is correct. Under the NRV method,
joint cost allocated to CBL based on the relative additional separable costs are deducted from sales
sales value method. revenue to determine the NRV. In this problem, no
additional costs are incurred, so the NRV equals the
Answer (D) is incorrect because $120,000 is the total sales revenue for the joint products.
joint cost allocated to MSB based on physical
quantities. Liquid Skin 180,000 gals. x $2.40 = $432,000
Silken Skin 120,000 gals. x $3.90 = 468,000
--------
[103] Source: CMA 0690 4-8 Total Sales $900,000
========
Answer (A) is incorrect because the total cost of Because sales of Liquid Skin make up 48% of total
CBL is $425,000 ($225,000 + $200,000). The unit sales ($432,000 ・$900,000), it should be allocated
cost is $5.3125 ($425,000 ・80,000). 48% of the joint cost. Thus, Liquid Skin's share is
$201,600 (48% x $420,000).
Answer (B) is incorrect because $5.625 is
calculated Answer (C) is incorrect because $218,400 is Silken
using costs of $225,000 instead of $200,000. Skin's share of joint costs under the NRV method.

Answer (C) is incorrect because $2.50 is calculated Answer (D) is incorrect because $252,000 is the
by dividing $200,000 by 80,000 units. joint cost allocated to Liquid Skin on a
physical-quantity basis.
Answer (D) is correct. The first step is to compute
the sales for each product. The 60,000 units of MSB
sell for $5 each after $100,000 of additional costs [106] Source: CMA 0689 4-22
are incurred. The NRV is therefore $200,000 [($5 x
60,000 units) - $100,000]. The 80,000 units of CBL Answer (A) is correct. The total physical quantity is
sell for $10 each, or $800,000, after further 300,000 gallons (180,000 + 120,000). Because
processing. But to determine the NRV for CBL, the Silken Skin accounts for 120,000 gallons, or 40% of
costs of this processing must be deducted from the total, it should be allocated $168,000 (40% x
sales. $420,000 joint cost).
complete. Hence, direct materials have already been
Answer (B) is incorrect because $201,600 is the added. Ending inventory has not reached the 25%
joint cost allocated to Liquid Skin on the NRV basis. stage of completion, so direct materials have not yet
been added to these units. Thus, the equivalent units
Answer (C) is incorrect because $218,400 is Silken for direct materials calculated on a FIFO basis are
Skin's share of joint costs under the NRV method. equal to the units started and completed in the current
period (85,000 units completed - 15,000 units in
Answer (D) is incorrect because $252,000 is the BWIP = 70,000 units started and completed).
joint cost allocated to Liquid Skin on a
physical-quantity basis. Answer (B) is incorrect because 80,000 total units
were transferred in from Department 1.

[107] Source: CMA 0689 4-23 Answer (C) is incorrect because 85,000 equals the
equivalent units for direct materials calculated on a
Answer (A) is incorrect because $140,000 is the weighted-average basis.
joint cost allocated to Liquid Skin under the NRV
method given additional processing costs. Answer (D) is incorrect because 90,000 units equals
the sum of units transferred in from Department 1 and
Answer (B) is incorrect because $168,000 is the ending work-in-process inventory.
joint cost allocated to Silken Skin on a
physical-quantity basis.
[110] Source: CIA 1192 IV-6
Answer (C) is incorrect because $252,000 is the
joint cost allocated to Liquid Skin on a Answer (A) is incorrect because 170,000 is the
physical-quantity basis. number of equivalent units of materials for the period.

Answer (D) is correct. If the products cannot be sold Answer (B) is correct. The number of equivalent units
at split-off, additional processing costs must be computed under this method excludes work done on
incurred. Under the NRV method, these additional BWIP in the prior period. Thus, the total of
costs are deducted from sales to determine NRV. equivalent units of conversion cost for the period is
Total sales of the joint products are $432,000 for calculated as follows:
Liquid Skin and $468,000 for Silken Skin. The costs
incurred beyond the split-off point are 10,000 units in EWIP x 50% = 5,000
180,000 completed units x 100% = 180,000
Liquid Skin 180,000 gals. x $1.40 = $252,000 -------
Silken Skin 120,000 gals. x $ .90 = 108,000 185,000
The NRVs are $180,000 ($432,000 - $252,000) for 20,000 units in BWIP x 50% = (10,000)
Liquid Skin and $360,000 ($468,000 - $108,000) -------
for Silken Skin, a total of $540,000. Consequently, 175,000
Silken Skin should be allocated $280,000 of the =======
joint
cost [$420,000 x ($360,000 ・$540,000)]. Answer (C) is incorrect because 180,000 is the total
amount of work done on the completed units.

[108] Source: CMA 0689 4-24 Answer (D) is incorrect because 185,000 is the
amount determined using the weighted-average
Answer (A) is incorrect because $140,000 is the method.
joint cost allocated to Liquid Skin under the NRV
method given additional processing costs.
[111] Source: CIA 0594 III-75
Answer (B) is incorrect because $168,000 is the
joint cost allocated to Silken Skin on a Answer (A) is incorrect because $37,500 results
physical-quantity basis. from allocating the cost based on total kilowatt hours
used by service and production departments.
Answer (C) is correct. If a physical-quantity
allocation method is used, sales, costs, and NRVs Answer (B) is incorrect because $15,625 is the
are irrelevant. The total physical quantity is amount allocated to the assembly department.
300,000
gallons (180,000 + 120,000). Because Liquid Skin Answer (C) is incorrect because $39,062.50 results
is from allocating the cost based on direct costs for
60% of the total, it should be allocated $252,000 each production department.
(60% x $420,000 joint cost).
Answer (D) is correct. Under the direct method,
Answer (D) is incorrect because $280,000 is the service department costs are allocated directly to the
joint cost allocated to Silken Skin under the NRV producing departments without recognition of
method. services provided to other service departments.
Allocation of service department costs are made only
to production departments based on their relative use
[109] Source: CIA 0593 IV-5 of services, in this case, kilowatt hours. Thus, the cost
allocated to the Machining Department is $46,875
Answer (A) is correct. Beginning inventory is 40% [$62,500 x (150,000 ・200,000)].
Answer (C) is incorrect because normal spoilage is
[112] Source: CIA 0594 III-76 4% of the units inspected, not of the units entering
production.
Answer (A) is incorrect because no cost should be
allocated. Answer (D) is correct. Normal spoilage equals 4% of
the units inspected. The equivalent units of normal
Answer (B) is incorrect because no cost should be spoilage equal 19,336 [.04 x (460,800 units passing
allocated. inspection + 22,600 units failing inspection)]. The
equivalent units of abnormal spoilage are simply the
residual of the spoiled units, or 3,264 (22,600 total
Answer (C) is incorrect because no cost should be spoiled units - 19,336 normal spoilage).
allocated.

Answer (D) is correct. Under the direct method, [115] Source: CIA 0578 IV-1
service department costs are allocated directly to
the Answer (A) is incorrect because LIFO is equally
producing departments without recognition of applicable to either job-order costing or process
services provided to other service departments. costing.
Allocation of service department costs are made
only Answer (B) is incorrect because process costing is
to production departments based on their relative equally useful for the estimation of overhead.
use
of services. Thus, none of the cost will be allocated Answer (C) is incorrect because control of costs
to does not vary between job-order and
the maintenance department. process-costing systems.

Answer (D) is correct. Job-order costs are used in


[113] Source: CIA 0594 III-77 determining the costs of a specific, clearly identifiable
job or project. In contrast, process costing averages
Answer (A) is incorrect because, under the step the costs of all production.
method, service costs are allocated to both
production and service departments.
[116] Source: CIA 1186 IV-4
Answer (B) is correct. Under the step method,
service costs are allocated to producing Answer (A) is correct. A cost system accounts for
departments the costs of manufacturing inventoriable output. The
and to other service departments. This method objective is to determine the portion of manufacturing
does cost to be expensed (because the output was sold)
not allocate any cost back to the departments and the portion to be deferred (because the output
whose was still on hand). Process costing is used for
costs have already been allocated. This company continuous process manufacturing of units that are
will relatively homogeneous (e.g., oil refining, automobile
allocate Power Department costs first because the production, etc.). Job-order costing is used to
Maintenance Department receives relatively more account for the cost of specific jobs or projects when
service from the Power Department. The amount of output is heterogeneous. The difference is often
Power Department cost allocated to the overemphasized. Job-order costing simply requires
Maintenance subsidiary ledgers (to keep track of the specific jobs)
Department is $12,500 [$62,500 x (50,000 ・ for the same work-in-process (manufacturing)
250,000)]. This allocation is based on the total account and finished goods inventory account that are
kilowatt hours used by all departments (service and basic to process costing.
production).
Answer (B) is incorrect because how orders are
Answer (C) is incorrect because $6,250 results from taken is irrelevant to whether job-order or process
allocating Power Department costs based on costing is used.
maintenance hours used.
Answer (C) is incorrect because profit is determined
Answer (D) is incorrect because $8,000 results from in the same way in both job-order and process
allocating the Maintenance Department's direct costing.
costs
to itself based on kilowatt hours used. Answer (D) is incorrect because the cost system is
not necessarily related to the manufacturing
processes.
[114] Source: CIA 1193 IV-7

Answer (A) is incorrect because 904 equals 4% of [117] Source: CIA 0590 IV-3
units failing final inspection.
Answer (A) is correct. Job-order costing is used by
Answer (B) is incorrect because normal spoilage is companies whose products or services are readily
4% of the total goods inspected, not 4% of the identified by individual units or a specific job, each of
completed units passing inspection. which receives varying amounts and types of input.
The dissimilarity of the various loan services employed, the basic materials usually vary by
provided production run, but the processes used are similar
makes job-order costing appropriate. (e.g., milling, grinding, finishing, painting, polishing,
etc.).
Answer (B) is incorrect because process costing is
used by companies whose products or services are Answer (B) is incorrect because direct materials
relatively uniform and are produced in a series of rather than direct labor vary by production run.
production steps or processes.
Answer (C) is incorrect because direct materials
Answer (C) is incorrect because differential costing rather than overhead vary by production run.
is
not a cost accumulation method. It is useful for Answer (D) is incorrect because direct materials
decision-making. rather than conversion costs vary by production run.

Answer (D) is incorrect because joint product


costing [121] Source: Publisher
is not a cost accumulation method. It is a method
of Answer (A) is incorrect because operation costing is
allocating joint costs to joint products. a hybrid form of job-order and process costing.

Answer (B) is incorrect because operation costing is


[118] Source: CIA 1187 IV-5 a hybrid form of job-order and process costing.

Answer (A) is incorrect because road building Answer (C) is incorrect because direct costing
involves unique projects which require job-order includes only variable manufacturing costs in unit cost.
costing.
Answer (D) is correct. Operation costing is a hybrid
Answer (B) is incorrect because electrical of job-order costing and process costing. As in
contracting process costing, a single average unit conversion cost
involves unique projects which require job-order is applied based on operations. Direct materials costs
costing. are applied to the individual batches in the same
manner as in job-order costing. It is used to account
Answer (C) is correct. Process costing is used for for the costs of batch processing of relatively large
continuous process manufacturing of relatively numbers of similar units in individual production runs.
homogeneous units. Newspapers are published in
long runs of identical items, hence process costing Operation costing is appropriate for the processing of
is different types of materials through the same basic
applicable. operations, such as woodworking, finishing, and
polishing for different product lines of furniture. It
Answer (D) is incorrect because the manufacture of usually has many more WIP accounts because there
clothing requires different sizes and styles to be is one for each process or operation.
produced. Operations costing is required.

[122] Source: Publisher


[119] Source: Publisher
Answer (A) is incorrect because shoes are produced
Answer (A) is incorrect because, when the products in production runs or batches, but with different styles
are unique, job-order costing is appropriate. or materials.

Answer (B) is correct. Operation costing is used Answer (B) is incorrect because clothing is produced
when goods are produced in batches or production in production runs or batches, but with different styles
runs. While the production procedures or or materials.
operations
are similar, the types of items processed are Answer (C) is incorrect because furniture is
different, produced in production runs or batches, but with
e.g., different models of furniture, etc. different styles or materials.

Answer (C) is incorrect because, when the product Answer (D) is correct. The products of the oil
is refining industry are homogeneous and are not
homogeneous, process costing is appropriate. produced in batches. Although refineries can vary the
nature of their output slightly, they essentially produce
Answer (D) is incorrect because the products of the a continuous flow of products.
food and beverage industry may be either unique
or
homogeneous, and produced in jobs, lots, or [123] Source: CIA 1188 IV-6
batches.
Answer (A) is incorrect because the EUP for
conversion costs would be 8,330 [8,780
[120] Source: Publisher weighted-average EUP - (30% x 1,500 units in
BWIP)] if the FIFO method were used.
Answer (A) is correct. When operation costing is
Answer (B) is correct. The weighted-average
method Answer (C) is incorrect because the 700 units of
of process costing commingles prior period (BWIP) normal spoilage are only 60% complete.
and current costs. It does not consider the degree
of Answer (D) is incorrect because the 700 units each
completion of BWIP when computing EUP. of both normal and abnormal spoilage are only 60%
complete.
Units % CC EUP
----- --- ------
Completed 7,400 100 7,400 [126] Source: CIA 0586 IV-6
EWIP 2,300 60 1,380
------ Answer (A) is incorrect because $33 is the normal
Equivalent units 8,780 spoilage cost [(.03 x 2,000) x $.55].
======
Answer (B) is incorrect because $20.35 is based on
Answer (C) is incorrect because 9,230 includes 30% calculating normal spoilage as 3% of the number of
of BWIP in the total. baseballs completed [(100 - (.03 x 2,100)) x $.55].

Answer (D) is incorrect because 9,700 is the sum of Answer (C) is correct. Abnormal spoilage is
the physical, not equivalent, units completed and in calculated as the total unit cost times the amount of
EWIP. spoilage in excess of expected normal spoilage. Total
unit cost is

[124] Source: CIA 0591 IV-4 Materials cost ($840 ・2,100 EUP) $0.40
Conversion cost ($315 ・2,100 EUP) 0.15
Answer (A) is incorrect because 26,000 (31,000 -----
weighted-average EUP - 5,000 EUP in BWIP) is the $0.55
number of direct materials equivalent units =====
calculated Spoilage in excess of normal spoilage is 40 units [100
using the FIFO method. spoiled units - (.03 x 2,000 good units)]. Abnormal
spoilage is thus $22 ($.55 x 40).
Answer (B) is incorrect because 28,500 is the
number of units transferred out to finished goods Answer (D) is incorrect because $1,100 is the cost
inventory. of good baseballs produced ($.55 x 2,000).

Answer (C) is correct. The weighted-average


approach averages the costs in beginning
work-in-process with those incurred during the [127] Source: CIA 0587 IV-5
period. Accordingly, the degree of completion of
the Answer (A) is incorrect because normal spoilage
BWIP is ignored in computing the equivalent units ($120), not abnormal spoilage ($80), should be
for included in the cost of good units produced.
direct materials. Direct materials equivalent units
therefore consist of units transferred to finished Answer (B) is correct. Abnormal spoilage is not
goods expected to occur under efficient operating
(28,500) and units that failed inspection (2,500), or conditions. Thus, abnormal spoilage is excluded from
31,000. Ending work-in-process inventory has not the cost of the good units. Hence, the total
reached the point at which materials are added. production cost of $2,200 is reduced by $80 [20
units x ($2,200 ・550 total units)] to arrive at the
Answer (D) is incorrect because 34,000 [5,000 units $2,120 cost of good units.
in BWIP + 32,000 units transferred in - (60% x
5,000 units in BWIP)] is the number of physical Answer (C) is incorrect because the total production
units costs should be reduced by the abnormal spoilage
minus the conversion work previously done on the ($80) to find the cost of good units produced.
BWIP.
Answer (D) is incorrect because $2,332 results from

[125] Source: Publisher basing the cost per unit on 500 good units rather than
550 total units.
Answer (A) is correct. EUP assigned to spoilage are
based upon the actual processing costs incurred. If
700 spoiled units are 60% complete, 420 [128] Source: CIA 1190 IV-9
equivalent
units have been produced. Accordingly, 420 EUP Answer (A) is incorrect because spoilage is
should be assigned to both normal and abnormal calculated using the normal input (not total units) of
spoilage. The most accurate equivalent unit 30,000 units (28,500 ・.95).
calculation includes spoilage, both abnormal and
normal. Answer (B) is correct. Normal input (input for the
good units and normal spoilage) is calculated by
Answer (B) is incorrect because the 700 units of dividing the good units produced by the proportion
abnormal spoilage are only 60% complete. left after the spoiled units are removed, or 95%
(100% - 5%). Thus, normal input is the input for Answer (A) is incorrect because $5,000 is the cost
30,000 units (28,500 ・.95), normal spoilage is to further process product X after split-off. The
1,500 units (30,000 - 28,500), and abnormal amount of joint costs allocated to product X is
spoilage is the difference between total and normal $6,000 [($12,000 sales price at split-off for product
spoilage [(31,000 - 28,500) - 1,500]. X ・$20,000 total sales price for X and Y) x
$10,000 joint processing costs].
Answer (C) is incorrect because spoilage is
calculated using the normal input (not good units) Answer (B) is correct. Under the net realizable value
of method, joint costs are allocated to products based
30,000 units (28,500 ・.95). on their relative net realizable values unless sales price
quotations are available at split-off. Split-off sales
Answer (D) is incorrect because spoilage is price quotations are available, so the amount of joint
calculated using the normal input (not total units) costs allocated to product X can be computed as
of follows:
30,000 units (28,500 ・.95). Abnormal spoilage
would then be determined by subtracting 1,500 Sales value of product X
(30,000 x .05) units of normal spoilage from 2,500 ------------------------ x Joint costs
total spoiled units. Total sales value
$12,000
= ------- X $10,000
[129] Source: CIA 0586 IV-11 $20,000
= $6,000
Answer (A) is correct. To determine the proportion
of joint costs to be allocated to F1, the net Answer (C) is incorrect because, since sales prices at
realizable split-off are known, joint costs will be allocated
values of the three products must be calculated. based on each product's proportional sales value.
Net
realizable value per unit is selling price minus Answer (D) is incorrect because $10,000 is the total
additional processing costs. joint processing cost. The total needs to be allocated
to each product based on their net realizable values.
F1: 5($30 - $28) = $10
F2: 2($35 - $30) = 10
F3: 3($35 - $25) = 30 [131] Source: CIA 1184 IV-23
---
$50 Answer (A) is incorrect because the common cost
=== allocations of $450 to cheese and $150 to whey
The amount of joint costs to be allocated to F1 is were arrived at by allocating total common costs of
20% ($10 ・$50). $600 instead of $1,000, which includes overhead of
$400.
Answer (B) is incorrect because the proportion of
joint costs allocated to F1 is calculated based on Answer (B) is incorrect because the common cost
the allocation of $500 to both cheese and whey was
proportion of net realizable value attributable to F1, arrived at by allocating the total common cost of
not on the proportion of selling price attributable to $1,000 by output in pounds instead of the sales
F1. values.

Answer (C) is incorrect because the allocation of Answer (C) is incorrect because the common costs
joint costs is based on net realizable value; joint are allocated based on net realizable values (price per
costs pound x demand in pounds), not sales prices per
are not divided equally among the three products. pound.
The amount of joint costs to be allocated to F1 is
20% ($10 net realizable value for five units of F1 ・ Answer (D) is correct. The relative sales value
$50 total net realizable value). method allocates costs in proportion to the relative
sales value of the individual products. The total
Answer (D) is incorrect because the portion of joint common costs are
costs allocated to F1 is based on net realizable
value. Milk (1,000 lb. x $.20) $ 200
The net realizable value for F1 is $2 per unit ($30 Labor (40 hr. x $10) 400
selling price - $28 additional processing costs) and O/H (1.0 x $400 DL cost) 400
each ton of material yields five units of F1, for a ------
total Common costs $1,000
net realizable value of $10 per ton. Net realizable ======
value for all three products is $50 per ton. Sales Value:
Therefore, Cheese ($2 x 450) $ 900 75%
the portion of joint costs allocated to F1 is 20% Whey ($.80 x 375) 300 25%
($10 ------
・$50). Total $1,200
======
Cost to cheese (75% x 1,000) $ 750
[130] Source: CIA 1185 IV-11 Cost to whey (25% x 1,000) 250
------ Answer (D) is incorrect because the joint
$1,000 manufacturing costs are the only irrelevant item.
======
If only 375 pounds of whey can be sold, the other
75 [134] Source: CMA 1295 3-19
pounds are worthless and are not allocated any
common cost. Answer (A) is correct. Prime costs are defined as
direct materials and direct labor. The first step is to
calculate the cost of raw materials used during the
[132] Source: CIA 0587 IV-6 month:

Answer (A) is incorrect because the net revenue Beginning materials inventory $ 67,000
from Plus purchases 163,000
the sale of the by-product is $1,700, not $2,000. Plus transportation in 4,000
The Minus purchase returns (2,000)
costs related to the packaging and selling of the Materials available for use $232,000
by-product must be deducted. Minus ending materials inventory (62,000)
Materials used $170,000
Answer (B) is incorrect because the $.10-per-pound Adding the $170,000 of materials used to the
packaging cost for the sawdust must be deducted $200,000 of direct labor results in a total of
from the by-product revenue. $370,000 for prime costs.

Answer (C) is correct. The net revenue from sale of Answer (B) is incorrect because $168,000 equals
the by-product is $1,700 [($2 price x 1,000 lb.) - purchases of raw materials adjusted for the change in
($.10 x 1,000 lb.) - (.1 x $2 x 1,000 lb.)]. Joint inventories.
processing costs to be allocated to joint products
are Answer (C) is incorrect because $363,000
therefore $48,300 ($50,000 - $1,700 net incorporates the change in finished goods inventories.
by-product revenue). Of this amount, $32,200
should be assigned to the two-by-fours [$48,300 x Answer (D) is incorrect because the $170,000 is
(200,000 board feet of two-by-fours ・300,000 total only the raw materials used.
board feet)].

Answer (D) is incorrect because the net revenue of [135] Source: CMA 1295 3-20
$1,700 [($2 sale price - $.10 packaging - $.20 sales
cost) x 1,000 lb.] from the by-product should be Answer (A) is incorrect because $502,000 is based
subtracted from the joint processing costs before on actual overhead.
the
joint processing costs are allocated. Answer (B) is incorrect because $503,000
incorporates the change in finished goods inventories.

[133] Source: CMA 1294 4-1 Answer (C) is incorrect because $363,000 excludes
overhead but includes the change in finished goods
Answer (A) is incorrect because the joint inventory.
manufacturing costs are the only irrelevant item.
Answer (D) is correct. Total manufacturing costs
Answer (B) is correct. Common, or joint, costs consist of materials, labor, and overhead. Total prime
cannot be identified with a particular joint product. costs were $370,000. Overhead applied was
By definition, joint products have common costs $140,000 (70% x $200,000 of direct labor).
until Therefore, total manufacturing cost is $510,000
the split-off point. Costs incurred after the split-off ($170,000 + $200,000 + $140,000).
point are separable costs. The decision to continue
processing beyond split-off is made separately for
each product. The costs relevant to the decision are [136] Source: CMA 1295 3-21
the separable costs because they can be avoided
by Answer (A) is incorrect because $469,000 uses
selling at the split-off point. They should be actual overhead and adjusts the figures for the change
compared in finished goods inventory.
with the incremental revenues from processing
further. Thus, items I (revenue from selling at split- Answer (B) is incorrect because $477,000 includes
off the change in finished goods inventory in the
point), II (variable costs of upgrade), III (avoidable calculation.
fixed costs of upgrade), and IV (revenue from
selling Answer (C) is incorrect because $495,000 uses
after further processing) are considered in making materials purchased rather than materials used and
the also fails to adjust properly for transportation in.
upgrade decision.
Answer (D) is correct. Total manufacturing costs
Answer (C) is incorrect because the joint consist of materials, labor, and overhead. That
manufacturing costs are the only irrelevant item. amount is then adjusted for the change in
work-in-process inventories to arrive at the cost of
goods transferred to finished goods. Total [140] Source: CMA 1286 4-15
manufacturing cost was $510,000. Thus, the cost of
goods transferred to finished goods inventory is Answer (A) is incorrect because 3,400 units consist
$484,000 ($510,000 + $145,000 - $171,000). of the units started and completed during November
plus the 20% of work-in-process complete as to
conversion costs (3,000 + 400).

Answer (B) is correct. Since BWIP (1,000 units)


[138] Source: CMA 1295 3-23 was already 60% complete, 400 equivalent units
were needed for completion. In addition, 3,000 units
Answer (A) is incorrect because an overapplication were started and completed during the period. The
of overhead would be represented by a credit in the 2,000 units in EWIP equal 400 equivalent units since
overhead control account. they are 20% complete. Total equivalent units are
3,800 (400 + 3,000 + 400).
Answer (B) is incorrect because the overhead was
overapplied for the month. Answer (C) is incorrect because 4,000 units consist
of the units started and completed in November and
Answer (C) is correct. The factory overhead control the units completed and transferred out from BI
account would have been debited for $132,000 of (3,000 + 1,000).
actual overhead. Credits would have totaled
$140,000 representing 70% of direct labor costs of Answer (D) is incorrect because 4,400 units consist
$200,000. Therefore, the $140,000 credit exceeds of the units started and completed in November, plus
the $132,000 debit, and there has been an the units completed and transferred out from BI plus
overapplication of overhead in the amount of the 20% of work-in-process complete as to
$8,000. conversion costs (3,000 + 1,000 + 400).

Answer (D) is incorrect because the overhead was


overapplied for the month. [141] Source: CMA 1286 4-16

Answer (A) is incorrect because 3,400 units equals


[139] Source: CMA 1286 4-14 units started and completed in November plus 20%
of ending work-in-process (3,000 + 400).
Answer (A) is correct. The computation of
equivalent Answer (B) is incorrect because 4,400 units equals
units for a period using the FIFO method of process units started and completed in November plus units
costing includes only the conversion costs and completed and transferred out from BI plus 20% of
material added to the product in that period and ending work-in-process (3,000 + 1,000 + 400).
excludes any work done in previous periods.
Accordingly, FIFO equivalent units include work and Answer (C) is incorrect because 5,000 units are the
material to complete BWIP, plus work and material number of units started in November.
to complete units started this period, minus work
and Answer (D) is correct. The difference between the
material needed to complete EWIP. Since all weighted-average and FIFO methods of process
materials are added at the beginning of the costing is how BWIP is handled. FIFO makes a
process, distinction between the costs in BWIP and the costs
only those units started during November would of goods started this period. Weighted average does
have not. Thus, when there is no BWIP, there is no
received materials in that month. Because 5,000 difference between the two costing methods. Since
units 6,000 units have been started (1,000 BWIP + 5,000
were started, the equivalent units for direct started this period), and all materials are added at the
materials beginning of the process, equivalent units for materials
equal 5,000. equal 6,000.

Answer (B) is incorrect because 6,000 is the total


units to account for. [142] Source: CMA 1286 4-17

Answer (C) is incorrect because 4,400 equals units Answer (A) is incorrect because 3,400 units consist
completed and transferred out from BI plus units of the units started and completed in November plus
started and completed in November plus 20% of the 20% of work-in-process complete as to
work-in-process on November 30 (1,000 + 3,000 + conversion costs (3,000 + 400).
400).
Answer (B) is incorrect because 3,800 units equal
Answer (D) is incorrect because 3,800 is not the the 400 units in BWIP needed for completion plus
equivalent units for direct materials. Only those the units started and completed in November plus the
units 20% of work-in-process complete as to conversion
started during November would have received costs (400 + 3,000 + 400).
materials in that month. Therefore, equivalent units
for Answer (C) is incorrect because 4,000 units equal
direct materials equal 5,000. the units completed and transferred out from BI plus
units started and completed during November (1,000
+ 3,000).
Answer (D) is correct. Under the weighted-average Answer (A) is incorrect because the total of the
method, work in the previous period on the service department costs allocated to the Assembly
beginning Department is $167,500.
inventories is included along with the work added
this Answer (B) is incorrect because the total of the
period. Thus, the only difference between the FIFO service department costs allocated to the Assembly
calculations and the weighted-average calculation Department is $167,500.
is
the equivalent units for the beginning inventory. Answer (C) is incorrect because the total of the
The service department costs allocated to the Assembly
4,000 completed units (1,000 BWIP + 3,000 started Department is $167,500.
this period) equal 4,000 equivalent units. The 2,000
units in EWIP are equivalent to 400 units (20% Answer (D) is correct. Under the direct method,
complete x 2,000 units). Thus, there are 4,400 service department costs are allocated directly to the
conversion cost equivalent units. production departments, with no allocation to other
service departments. The total budgeted hours of
service by the Quality Control Department to the two
[143] Source: CMA 0694 3-25 production departments is 28,000 (21,000 + 7,000).
Given that the Assembly Department is expected to
Answer (A) is correct. If direct labor hours are used use 25% (7,000 ・28,000) of the total hours
as the allocation base, the $50,000 of costs is budgeted for the production departments, it will
allocated over 400 hours of direct labor. Multiplying absorb 25% of total quality control costs (25% x
the 25 units of each product times 200 hours $350,000 = $87,500). The total budgeted hours of
results service by the Maintenance Department to the
in 5,000 labor hours for each product, or a total of production departments is 30,000 (18,000 +
10,000 hours. Dividing $50,000 by 10,000 hours 12,000). The Assembly Department is expected to
results in a cost of $5 per direct labor hour. use 40% (12,000 ・30,000) of the total maintenance
Multiplying 200 hours times $5 results in an hours budgeted for the production departments.
allocation Thus, the Assembly Department will be allocated
of $1,000 of overhead per unit of product. 40% of the $200,000 of maintenance costs, or
$80,000. The total service department costs
Answer (B) is incorrect because $500 is the allocated to the Assembly Department is $167,500
allocation based on number of material moves. ($87,500 + $80,000).

Answer (C) is incorrect because $2,000 assumes


that all the overhead is allocated to the wall [146] Source: CMA 0691 3-17
mirrors.
Answer (A) is incorrect because the overhead cost
Answer (D) is incorrect because $5,000 assumes per machine hour is $15.65.
overhead of $250,000.
Answer (B) is incorrect because the overhead cost
per machine hour is $15.65.
[144] Source: CMA 0694 3-26
Answer (C) is incorrect because the overhead cost
Answer (A) is incorrect because $1,000 uses direct per machine hour is $15.65.
labor as the allocation basis.
Answer (D) is correct. Machining uses 75% (21,000
Answer (B) is correct. An activity-based costing ・28,000) of the total quality control hours and 60%
(ABC) system allocates overhead costs on the basis (18,000 ・30,000) of the total maintenance hours
of some causal relationship between the incurrence budgeted for the production departments. Under the
of direct method, it will therefore be allocated $262,500
cost and activities. Because the moves for wall (75% x $350,000) of quality control costs and
mirrors constitute 25% (5 ・20) of total moves, the $120,000 (60% x $200,000) of maintenance costs.
mirrors should absorb 25% of the total materials In addition, Machining is expected to incur another
handling costs. Thus, $12,500 (25% x $50,000) is $400,000 of overhead costs. Thus, the total
allocated to mirrors. The remaining $37,500 is estimated Machining overhead is $782,500
allocated to specialty windows. Dividing the ($262,500 + $120,000 + $400,000), and the
$12,500 overhead cost per machine hour is $15.65
by 25 units produces a cost of $500 per unit of ($782,500 ・50,000 hours).
mirrors.

Answer (C) is incorrect because $1,500 is the [147] Source: CMA 0691 3-19
allocation per unit of specialty windows.
Answer (A) is incorrect because the Assembly
Answer (D) is incorrect because $2,500 is not Department will be allocated maintenance costs of
based $108,000.
on the number of material moves.
Answer (B) is correct. The step-down method
allocates service costs to both service and production
[145] Source: CMA 0691 3-16 departments but does not permit mutual allocations.
Accordingly, Quality Control will receive no Assembly Department is the $300,000 budgeted for
allocation of maintenance costs. The first step is to that department. Hence, the overhead cost applied
allocate quality control costs to the Machining per direct labor hour will be $12 ($300,000
Department. Maintenance is expected to use 20% budgeted overhead ・25,000 hours).
(7,000 ・35,000) of the available quality control
hours and will be allocated $70,000 (20% x Answer (D) is incorrect because the overhead cost
$350,000) of quality control costs. Thus, total applied per direct labor hour will be $12.
allocable maintenance costs equal $270,000
($70,000 + $200,000). The Assembly Department
is [150] Source: CMA 1273 4-1
estimated to use 40% (12,000 ・30,000) of the
available maintenance hours. Consequently, it will Answer (A) is incorrect because it is a characteristic
be of absorption costing systems.
allocated maintenance costs of $108,000 (40% x
$270,000). Answer (B) is correct. In a direct (variable) costing
system, only the variable manufacturing costs are
Answer (C) is incorrect because the Assembly recorded as product costs. All fixed manufacturing
Department will be allocated maintenance costs of costs are expensed in the period incurred. Because
$108,000. changes in the relationship between production levels
and sales levels do not cause changes in the amount
Answer (D) is incorrect because the Assembly of fixed manufacturing cost expensed, profits more
Department will be allocated maintenance costs of directly follow the trends in sales.
$108,000.
Answer (C) is incorrect because it is a characteristic
of absorption costing systems.
[148] Source: CMA 0691 3-20
Answer (D) is incorrect because neither variable nor
Answer (A) is incorrect because the total quality absorption costing includes administrative costs in
control costs to be allocated equal $421,053. inventory.

Answer (B) is incorrect because the total quality


control costs to be allocated equal $421,053. [151] Source: CMA 1273 4-2

Answer (C) is incorrect because the total quality Answer (A) is incorrect because, since profit is a
control costs to be allocated equal $421,053. function of both sales and production, it will not
always move in the same direction as sales.
Answer (D) is correct. The reciprocal method
permits mutual allocations of service costs among Answer (B) is correct. In an absorption costing
service departments. For this purpose, a system of system, fixed overhead costs are included in
simultaneous equations is necessary. The total inventory. When sales exceed production, more
costs overhead is expensed under absorption costing due
for the Quality Control Department consist of to fixed overhead carried over from the prior
$350,000 plus 25% (10,000 hours ・40,000 hours) inventory. If sales increase over production, more
of maintenance costs. The total costs for the than one period's factory overhead is recognized as
Maintenance Department equal $200,000 plus 20% expense. Accordingly, if the increase in factory
(7,000 hours ・35,000 hours) of quality control overhead expensed is greater than the contribution
costs. These relationships can be expressed by the margin of the increased units sold, there may be less
following equations: profit with an increased level of sales.

Q = $350,000 + .25M Answer (C) is incorrect because decreased output


M = $200,000 + .2Q will increase the unit cost of items sold since fixed
To solve for Q, the second equation can be factory overhead per unit will increase.
substituted into the first as follows:
Answer (D) is incorrect because, given that profit is a
Q = $350,000 + .25($200,000 + .2Q) function of both sales and productivity, it will not
Q = $350,000 + $50,000 + .05Q always move in the same direction as sales.
.95Q = $400,000
Q = $421,053

[149] Source: CMA 0691 3-18 [153] Source: CMA 1285 4-15

Answer (A) is incorrect because the overhead cost Answer (A) is correct. The contribution margin from
applied per direct labor hour will be $12. manufacturing (sales - variable costs) is $10 ($40 -
$30) per unit sold, or $1,200,000 (120,000 units x
Answer (B) is incorrect because the overhead cost $10). The fixed costs of manufacturing ($600,000)
applied per direct labor hour will be $12. and selling and administrative costs ($400,000) are
deducted from the contribution margin to arrive at an
Answer (C) is correct. With no allocation of service operating income of $200,000. The difference
department costs, the only overhead applicable to between the absorption income of $440,000 and the
the $200,000 of direct costing income is attributable to
capitalization of the fixed manufacturing costs the fixed and variable selling and administrative costs.
under
the absorption method. Since 40% of the goods
produced are still in inventory (80,000 ・200,000), [156] Source: CMA 1290 3-24
40% of the $600,000 in fixed costs, or $240,000,
was capitalized under the absorption method. That Answer (A) is incorrect because ending inventory
amount was expensed under the direct costing was $1,200,000.
method.
Answer (B) is correct. Under the absorption method,
Answer (B) is incorrect because $440,000 is the unit cost is $30 ($12 direct materials + $9 direct
operating income using absorption costing. labor + $4 variable overhead + $5 fixed overhead).
Given beginning inventory of 35,000 units, the ending
Answer (C) is incorrect because $800,000 equals inventory equals 40,000 units (35,000 BI + 130,000
the contribution margin minus the selling and produced - 125,000 sold). Hence, ending inventory
administrative costs ($1,200,000 - $400,000). was $1,200,000 ($30 x 40,000 units).

Answer (D) is incorrect because $600,000 equals Answer (C) is incorrect because ending inventory
the fixed manufacturing costs. was $1,200,000.

Answer (D) is incorrect because ending inventory


[154] Source: CMA 1286 4-18 was $1,200,000.

Answer (A) is incorrect because $400,000 does not


include the variable factory overhead. In addition, [157] Source: CMA 1290 3-25
this
is the amount of inventoriable costs using Answer (A) is incorrect because ending inventory
absorption was $1,000,000.
costing.
Answer (B) is incorrect because ending inventory
Answer (B) is correct. Under direct costing, the only was $1,000,000.
costs that are capitalized are the variable costs of
manufacturing. These include Answer (C) is correct. Using variable costing, the unit
cost of ending inventory is $25 ($12 direct materials
Direct materials used $300,000 + $9 direct labor + $4 variable overhead). Given
Direct labor 100,000 beginning inventory of 35,000 units, the ending
Variable factory overhead 50,000 inventory equals 40,000 units (35,000 BI + 130,000
-------- produced - 125,000 sold). Thus, ending inventory
Total inventoriable costs $450,000 was $1,000,000 ($25 x 40,000).
========
Answer (D) is incorrect because ending inventory
Answer (C) is incorrect because $490,000 includes was $1,000,000.
the variable selling and administrative costs.

Answer (D) is incorrect because $530,000 includes [158] Source: CMA 1290 3-29
the fixed factory overhead.
Answer (A) is correct. Absorption costing results in a
higher income figure than variable costing whenever
[155] Source: CMA 1286 4-19 production exceeds sales. The reason is that
absorption costing capitalizes some fixed factory
Answer (A) is incorrect because $400,000 does not overhead as part of inventory. These costs are
include the variable factory overhead or the fixed expensed during the period incurred under variable
factory overhead. costing. Consequently, variable costing recognizes
greater expenses and lower income. The reverse is
Answer (B) is incorrect because $450,000 does not true when sales exceed production. In that case, the
include the fixed factory overhead. In addition, this absorption method results in a lower income because
is some fixed costs of previous periods absorbed by the
the amount of inventoriable costs using direct beginning inventory are expensed in the current
costing. period as cost of goods sold. Variable costing
income is never burdened with fixed costs of previous
Answer (C) is correct. The absorption method is periods.
required for financial statements prepared
according Answer (B) is incorrect because an increase in
to GAAP. It charges all costs of production to inventory results in a higher income under absorption
inventories. The variable cost of materials costing.
($300,000), direct labor ($100,000), variable
factory Answer (C) is incorrect because the important
overhead ($50,000), and the fixed factory overhead relationship is that between actual production and
($80,000) are included. They total $530,000. actual sales, not that between actual and planned
production.
Answer (D) is incorrect because $590,000 includes
Answer (D) is incorrect because planned sales do not
determine actual income. Answer (C) is incorrect because the variable costing
contribution margin was $5,625,000.

[159] Source: CMA 1290 3-28 Answer (D) is correct. At $70 per unit, actual sales
revenue was $8,750,000 for 125,000 units. Actual
Answer (A) is correct. Under the absorption variable costs of manufacturing were $25 per unit
method, ($12 + $9 + $4). The unit incurred costs for a
all selling and administrative fixed costs are production level of 130,000 units were the same as
charged the unit planned costs, which were the same for all
to the current period. Accordingly, $980,000 of units, whether made in the previous or current year.
selling expenses and $425,000 of actual fixed For example, total planned direct materials cost for
administrative expenses were expensed during the 140,000 units was $1,680,000, or $12 per unit. The
year. The fixed manufacturing costs must be incurred unit cost was also $12 ($1,560,000 ・
calculated after giving consideration to the 130,000 units). Thus, total variable manufacturing
increase in cost was $3,125,000 ($25 x 125,000 units).
inventory during the period (some fixed costs were Consequently, manufacturing contribution margin was
capitalized) and to the underapplied overhead. The $5,625,000 ($8,750,000 - $3,125,000).
beginning finished goods inventory included 35,000
units, each of which had absorbed $5 of fixed
manufacturing overhead. Each unit produced [161] Source: CMA 1290 3-27
during
the year also absorbed $5 of fixed manufacturing Answer (A) is correct. The unit variable
overhead. Given that 125,000 of those units were manufacturing cost was $25 ($12 direct materials +
sold, cost of goods sold was debited for $625,000 $9 direct labor + $4 variable overhead). Other
of variable costs included selling expenses ($8 per unit)
fixed overhead (125,000 units x $5). At year-end, and administrative expenses ($2 per unit). The unit
the underapplied overhead was also added to cost selling and administrative costs actually incurred for
of sales of 125,000 units were the same as the planned
goods sold. Because production was expected to be unit costs. For example, actual unit variable selling
140,000 units, the overhead application rate for the expense was $8 ($1,000,000 ・125,000 units sold),
$700,000 of planned fixed manufacturing overhead which equaled the planned unit cost. Thus, total unit
was $5 per unit. Only 130,000 units were variable cost was $35 ($25 + $8 + $2). The total
manufactured. Hence, $650,000 ($5 x 130,000 expensed was therefore $4,375,000 ($35 x 125,000
units) of overhead was applied to units in process. units sold).
Because inventory increased from 35,000 to 40,000
units, $25,000 (5,000-unit increase x $5) of the Answer (B) is incorrect because the total variable
applied fixed manufacturing overhead for the cost expensed under the variable costing basis was
period $4,375,000.
was inventoried, not expensed. Actual overhead
was Answer (C) is incorrect because the total variable
$715,000, so the underapplied overhead was cost expensed under the variable costing basis was
$65,000 ($715,000 - $650,000). This amount was $4,375,000.
charged to cost of goods sold at year-end. The total
of the fixed costs expensed was therefore Answer (D) is incorrect because the total variable
$2,095,000 ($980,000 selling expenses + $425,000 cost expensed under the variable costing basis was
administrative expenses + $625,000 standard $4,375,000.
manufacturing overhead costs of units sold +
$65,000 underapplied overhead).
[162] Source: CMA 1290 3-30
Answer (B) is incorrect because the total fixed costs
under the absorption costing basis were Answer (A) is incorrect because the difference
$2,095,000. between absorption costing and variable costing
income was $25,000.
Answer (C) is incorrect because the total fixed costs
under the absorption costing basis were Answer (B) is correct. The difference is caused by
$2,095,000. the capitalization of some of the fixed manufacturing
overhead. When inventories increase during the
Answer (D) is incorrect because the total fixed period, the absorption method capitalizes that
costs overhead and transfers it to future periods. The
under the absorption costing basis were variable costing method deducts it in the current
$2,095,000. period. Inventories increased by 5,000 units during
the period, and each of those units would have
included $5 of fixed manufacturing overhead under
[160] Source: CMA 1290 3-26 absorption costing. Accordingly, $25,000 of fixed
manufacturing overhead would have been capitalized.
Answer (A) is incorrect because the variable costing Recognizing $25,000 of fixed costs in the balance
contribution margin was $5,625,000. sheet instead of the income statement results in a
$25,000 difference in income between the two
Answer (B) is incorrect because the variable costing costing methods.
contribution margin was $5,625,000.
Answer (C) is incorrect because the difference
between absorption costing and variable costing applies more fixed costs to inventory.
income was $25,000.
Answer (D) is correct. Under an absorption-costing
Answer (D) is incorrect because the difference system, income can be manipulated by producing
between absorption costing and variable costing more products than are sold because more fixed
income was $25,000. manufacturing overhead will be allocated to the
ending inventory. When inventory increases, some
fixed costs are capitalized rather than expensed.
[163] Source: CMA 1292 3-5 Decreasing production, however, will result in lower
income because more of the fixed manufacturing
Answer (A) is incorrect because manufacturing overhead will be expensed.
supplies are variable costs inventoried under both
methods.
[166] Source: CIA 0593 IV-10
Answer (B) is incorrect because factory insurance is
a fixed manufacturing cost inventoried under Answer (A) is incorrect because total variable cost
absorption costing but written off as a period cost would be $1,100 if 200,000 copies were made in
under variable costing. February.

Answer (C) is incorrect because direct labor cost is Answer (B) is correct. Unit variable cost can be
a determined by dividing the $1,000 cost increase for
product cost under both methods. the additional copies made in February by the
increase in copies made. Hence, total variable cost
Answer (D) is correct. Under absorption costing, all for 110,000 copies is $2,200 {[$1,000 ・(150,000 -
manufacturing costs, both fixed and variable, are 100,000)] x 110,000}.
treated as product costs. Under variable costing,
only Answer (C) is incorrect because variable cost per
variable costs of manufacturing are inventoried as unit would have to be $.05 to total $5,500.
product costs. Fixed manufacturing costs are
expensed as period costs. Packaging and shipping Answer (D) is incorrect because $7,920 results from
costs not product costs under either method calculating an average cost per copy for January and
because February and then multiplying by 110,000 copies.
they are incurred after the goods have been
manufactured. Instead, they are included in selling
and administrative expenses for the period. [167] Source: CIA 1193 IV-10

Answer (A) is incorrect because a change in a


[164] Source: CMA 1292 3-6 variable period cost will affect absorption and
variable costing in the same way.
Answer (A) is correct. Activity-based costing,
job-order costing, process costing, and standard Answer (B) is incorrect because, if the beginning
costing can all be used for both internal and inventory is less than the ending finished goods
external inventory, absorption costing assigns more fixed
purposes. Variable costing is not acceptable under overhead costs to the balance sheet and less to the
GAAP for external reporting purposes. cost of goods sold on the income statement than does
variable costing.
Answer (B) is incorrect because job costing is
acceptable for external reporting purposes. Answer (C) is correct. Absorption costing (full
costing) is the accounting method that considers all
Answer (C) is incorrect because variable costing is manufacturing costs as product costs. These costs
acceptable for internal purposes only. include variable and fixed manufacturing costs,
whether direct or indirect. However, variable costing
Answer (D) is incorrect because process costing is treats fixed factory overhead as a period cost instead
acceptable for external reporting purposes. of charging it to the product (inventory). Thus, when
sales exceed production, the absorption costing
method recognizes fixed factory overhead inventoried
[165] Source: CMA 1292 3-26 in a prior period. Direct costing does not.
Accordingly, net income under variable costing will
Answer (A) is incorrect because producing more of be greater than net income under absorption costing.
the products requiring the most direct labor will
permit more fixed overhead to be capitalized in the Answer (D) is incorrect because inflationary effects
inventory account. will usually affect both absorption and variable
costing in the same way.
Answer (B) is incorrect because deferring expenses
such as maintenance will increase income in the
current period (but may result in long-range losses [168] Source: CIA 0594 III-46
caused by excessive down-time).

Answer (C) is incorrect because increasing Answer (A) is incorrect because increasing
production without a concurrent increase in inventories increases absorption costing profit as a
demand result of capitalizing fixed factory overhead.
system.
Answer (B) is incorrect because when sales volume
exceeds production, inventories decline. Thus, fixed Answer (C) is incorrect because inventoriable costs
factory overhead expensed will be greater under are directly related to the products produced, unlike
absorption costing. fixed manufacturing overhead which is, under variable
costing, a period cost.
Answer (C) is correct. Absorption (full) costing is the
accounting method that considers all Answer (D) is correct. The distinction between
manufacturing absorption costing and direct costing is in the
costs as product costs. These costs include variable treatment of fixed manufacturing costs. Absorption
and fixed manufacturing costs whether direct or costing treats them as product costs by allocating
indirect. Variable (direct) costing considers only them to inventory and cost of goods sold. Direct
variable manufacturing costs to be product costs, costing treats all fixed costs as period costs by
i.e., expensing them as incurred.
inventoriable. Fixed manufacturing costs are
considered period costs and are expensed as
incurred. If production is increased without [171] Source: CIA 1187 IV-9
increasing
sales, inventories will rise. However, all fixed costs Answer (A) is incorrect because direct costing
associated with production will be an expense of arguably understates inventory.
the
period under variable costing. Thus, this action will Answer (B) is incorrect because variable factory
not artificially increase profits and improve the overhead is a product cost under any cost system.
manager's review.
Answer (C) is incorrect because it states an argument
Answer (D) is incorrect because, under variable against absorption costing.
costing, operating profit is a function of sales.
Under Answer (D) is correct. Direct costing treats fixed
absorption costing, it is a function of sales and manufacturing costs as period costs, whereas
production. absorption costing accumulates them as product
costs. If product costs are viewed as all
manufacturing costs incurred to produce output, fixed
[169] Source: CIA 0577 IV-18 factory overhead should be inventoried because it is
necessary for production. The counter argument in
Answer (A) is incorrect because variable favor of direct costing is that fixed factory overhead is
manufacturing costs, whether direct (direct more closely related to capacity to produce than to
materials the production of individual units. Internal reporting
and direct labor) or indirect (variable factory for cost behavior analysis is more useful if it
overhead), are accounted for as product costs, not concentrates on the latter.
period costs.

Answer (B) is incorrect because nonvariable [172] Source: CIA 1187 IV-51
indirect
costs are treated as period costs in direct costing. Answer (A) is incorrect because indirect labor is a
product cost and sales commissions and depreciation
Answer (C) is correct. Direct (variable) costing on the administration building are period costs.
considers only variable manufacturing costs to be
product costs. Variable indirect costs included in Answer (B) is incorrect because direct labor and
variable factory overhead are therefore treated as factory utilities are product costs. Sales materials and
inventoriable. Fixed costs are considered period administrative supplies are period costs.
costs and are expensed as incurred.
Answer (C) is incorrect because product costs
Answer (D) is incorrect because, in direct costing, include direct materials and indirect factory materials.
nonvariable direct costs are treated as period costs, Product costs do not include advertising costs,
not product costs. administrative labor, or demographic research.

Answer (D) is correct. Product costs are the costs of


[170] Source: CIA 1186 IV-8 producing the product and are usually directly
identifiable with it. Thus, product costs include the
Answer (A) is incorrect because administrative costs of the factors of production identifiable with the
costs product, which usually include direct materials, direct
and fixed manufacturing overhead costs are labor, and factory (not general) overhead. Factory
treated overhead includes both fixed and variable elements in
separately although both are period costs in a an absorption-costing system, e.g., factory utilities,
direct indirect labor, and indirect materials. Product costs
costing system. are inventoried until the product is sold, at which time
they are expensed as cost of goods sold.
Answer (B) is incorrect because selling and fixed
manufacturing overhead costs are treated
separately [173] Source: CIA 0585 IV-5
although both are period costs in a direct costing
Answer (A) is incorrect because the gross margins
for Years 1 and 3 are equal.
[175] Source: CIA 0584 IV-6
Answer (B) is incorrect because, with other factors
held constant, the greater sales volume in Year 2 Answer (A) is incorrect because $6.83 is computed
should have produced a greater gross margin than by assuming two-thirds of the order is produced at
in $6.50 per unit and one-third of the order is produced
Years 1 or 3. at $7.50 per unit.

Answer (C) is correct. Gross margin equals sales Answer (B) is incorrect because $7.00 is the simple
minus CGS (BI + CGM - EI). An absorption-costing average of the two variable cost amounts.
system applies fixed as well as variable overhead
to Answer (C) is correct. Variable cost is equal to the
products. Because Blue's production has always direct costs associated with a product, an order, or
been less than planned activity, overhead was other decision. In this case, one-third of the order has
underapplied each year. Hence, Blue must have a variable cost of $6.50, and two-thirds of the order
debited underapplied overhead each year to CGS, has a variable cost of $7.50. Thus, the average
WIP, and FG. Because production always equaled variable cost is $7.17 {($6.50 ・3) + [(2 x $7.50) ・
sales, however, no inventories existed at any 3]}.
year-end, and thus each annual underapplication
should have been debited entirely to CGS. Answer (D) is incorrect because the sales price per
Consequently, the gross margins for Years 1 and 3 unit of the new order is $7.25.
must be the same because the gross revenue and
CGS were identical for the two periods.
[176] Source: CIA 0584 IV-7
Answer (D) is incorrect because Year 3 includes the
same amount of fixed overhead costs as in Year 2, Answer (A) is correct. The unit contribution margin
but it has less revenue than in Year 2; therefore, of the new order is the selling price of $7.25 minus
the the average variable cost. The average variable cost
gross margin will be lower. is $7.17. Accordingly, the unit contribution margin on
the order is $.08.

[174] Source: CIA 0584 IV-1 Answer (B) is incorrect because a $.25 contribution
margin assumes a cost per unit of $7.00.
Answer (A) is incorrect because 34% gross margin
includes all of the listed expenses. The period costs Answer (C) is incorrect because the contribution
should not be included in the gross margin margin is determined by subtracting the average
calculation. variable cost ($7.17) from the unit sales price
($7.25).
Answer (B) is incorrect because sales salaries and
advertising expenses should not be included in the Answer (D) is incorrect because the contribution
determination of the gross margin percentage. margin is determined by subtracting the average
variable cost ($7.17) from the unit sales price
Answer (C) is incorrect because 44% gross margin ($7.25).
includes all of the rent and supervision expenses.
Only their respective percentages of 9/10 and 2/3
should be included in the gross margin percentage [177] Source: CIA 0591 IV-13
calculation.
Answer (A) is incorrect because operating income
Answer (D) is correct. The gross margin percentage under variable costing equals sales, minus all fixed
equals gross profit (sales - CGS) divided by sales. costs, minus the variable cost of the goods sold.
Sales are given as $40,000, and expenses included
in Answer (B) is incorrect because $700 results from
cost of goods sold are listed as follows. The gross multiplying variable manufacturing costs per unit
margin is $18,400, which is 46% of $40,000. ($5.50) by units produced (1,200) rather than units
sold (1,000).
Sales $40,000
Cost of goods sold Answer (C) is correct. Operating income under
Direct materials $9,050 variable costing equals sales, minus all fixed costs,
Direct labor 6,050 minus the variable cost of the goods sold. No fixed
Rent (9/10 x $3,000) 2,700 costs are inventoried. The unit variable cost of the
Depreciation 2,000 goods sold was $6.00 ($5.50 + $.50), so variable
Supervision (2/3 x $1,500) 1,000 CGS was $6,000 (1,000 units x $6.00). Total fixed
Insurance (2/3 x $1,200) 800 (21,600) costs equaled $2,200 ($1,200 + $1,000). Given
------ ------- sales of $10,000 (1,000 units x $10), operating
$18,400 income was $1,800 ($10,000 - $6,000 - $2,200).
=======
Office expenses are usually general and Answer (D) is incorrect because $2,300 does not
administrative include variable selling and administrative costs of
expenses, which are period rather than product $500 (10,000 units x $.50).
costs.
that work-in-process is 100% complete as to
[178] Source: CIA 0591 IV-14 conversion costs.

Answer (A) is incorrect because $1,800 is the Answer (C) is incorrect because $53,000 exceeds
operating income under variable costing. the actual costs incurred during the period. Given no
beginning inventory, the amount transferred out
Answer (B) is incorrect because fixed cannot exceed the costs incurred during the period.
manufacturing
overhead costs are allocated to CGS and inventory. Answer (D) is correct. The total equivalent units for
Fixed selling and administrative costs are not raw materials equals 10,000 because all materials for
allocated, but are treated as period costs. the ending work-in-process had already been added
to production. Hence, the materials cost per unit was
Answer (C) is correct. Absorption costing $3.30 ($33,000 ・10,000). For conversion costs,
inventories the total equivalent units equals 8,500 [8,000
(treats as a product cost) fixed manufacturing completed + (25% x 2,000 in EWIP)]. Thus, the
overhead. Variable costing treats all fixed conversion cost was $2.00 per unit ($17,000 ・
manufacturing overhead as a period cost. 8,500). The total cost transferred was therefore
Assuming a $42,400 [8,000 units x ($3.30 + $2.00)].
fixed manufacturing overhead application rate of
$1
per unit ($1,200 ・1,200 units produced), ending [181] Source: CMA 0696 3-4
inventory and operating income will be $200
greater Answer (A) is incorrect because $10,000 assumes
under absorption than variable costing. that work-in-process inventory is 100% complete as
Accordingly, to conversion costs.
absorption costing operating income is $2,000
($1,800 variable costing operating income + $200). Answer (B) is incorrect because $2,500 assumes that
work-in-process inventory is 100% complete as to
Answer (D) is incorrect because only fixed conversion costs and that 500 bats are in inventory.
manufacturing overhead costs are allocated to CGS
and inventory. Fixed selling and administrative Answer (C) is incorrect because $20,000 assumes
costs that work-in-process is 100% complete as to
are treated as period costs. conversion costs and that 6,000 units were
transferred out.

[179] Source: CIA 1190 IV-12 Answer (D) is correct. $42,400 of costs were
transferred out. Consequently, the cost of the ending
Answer (A) is correct. Absorption-costing operating work-in-process must have been $7,600 ($50,000
income will exceed variable-costing operating total costs incurred - $42,400).
income
because production exceeds sales, resulting in a
deferral of fixed manufacturing overhead in the [182] Source: CMA 0696 3-19
inventory calculated using the absorption method.
The difference of $200,000 is equal to the fixed Answer (A) is incorrect because $101,400 assumes
manufacturing overhead per unit ($2,200,000 ・ zero ending inventory.
275,000 = $8.00) times the difference between
production and sales (275,000 - 250,000 = 25,000, Answer (B) is incorrect because $127,650 results
which is the inventory change in units). from reversing the treatment of beginning and ending
inventories.
Answer (B) is incorrect because units produced, not
units sold, should be used as the denominator to Answer (C) is correct. Purchases equals usage
calculate the fixed manufacturing cost per unit. adjusted for the inventory change. Hence, purchases
equals $130,150 ($128,900 used - $27,500 BI +
Answer (C) is incorrect because fixed selling and $28,750 EI).
administrative costs are not properly inventoriable
under absorption costing. Answer (D) is incorrect because $157,650 assumes
zero beginning inventory.
Answer (D) is incorrect because variable selling and
administrative costs are period costs under both
variable- and absorption-costing systems in the [183] Source: CMA 0696 3-29
determination of operating income.
Answer (A) is incorrect because $6.50 includes
selling and administrative expenses.
[180] Source: CMA 0696 3-3
Answer (B) is incorrect because $6.30 includes
Answer (A) is incorrect because a portion of the
total selling costs.
costs is still in work-in-process.
Answer (C) is incorrect because $5.70 includes
administrative expenses.
Answer (B) is incorrect because $40,000 assumes
Answer (D) is correct. Cost of goods sold is based
on the manufacturing costs incurred in production Answer (A) is incorrect because $25,000 excludes
but the indirect materials.
does not include selling or general and
administrative Answer (B) is correct. Factory (manufacturing)
expenses. Manufacturing costs equal $38,500 overhead consists of all costs, other than direct
[$13,700 DM + $4,800 DL + (800 hours x $25) materials and direct labor, that are associated with
FOH]. Thus, per-unit cost is $5.50 ($38,500 ・ the manufacturing process. It includes both fixed and
7,000 units). variable costs. The factory overhead control account
should have the following costs:

[184] Source: CMA 0696 3-30 Indirect materials $ 5,000


Indirect labor ($45,000-$40,000) 5,000
Answer (A) is incorrect because the ABC Other factory overhead 20,000
assignment
of $4,513 is at a rate of $180.52 for each of the 25 -------
orders. Total overhead $30,000
=======
Answer (B) is incorrect because ABC yields a higher
allocation. Answer (C) is incorrect because $45,000 is the total
labor cost.
Answer (C) is incorrect because the total is $4,513
on the ABC basis. Answer (D) is incorrect because $50,000 is the
direct materials cost.
Answer (D) is correct. ABC identifies the causal
relationship between the incurrence of cost and
activities, determines the drivers of the activities, [187] Source: CMA Samp Q3-5
establishes cost pools related to the drivers and
activities, and assigns costs to ultimate cost objects Answer (A) is incorrect because $11,000 equals the
on the basis of the demands (resources or drivers difference between budgeted and actual overhead.
consumed) placed on the activities by those cost
objects. Hence, ABC assigns overhead costs based Answer (B) is correct. Pane applies overhead to
on multiple allocation bases or cost drivers. Under products on the basis of direct labor cost. The rate is
the traditional, single-base system, the amount 1.4 ($448,000 budgeted OH ・$320,000 budgeted
allocated is $3,987.50 ($27,500 x 14.5%). Under DL cost). Thus, $483,000 (1.4 x $345,000 actual
ABC, the amount allocated is $4,513 [($11.50 x 12) DL cost) of overhead was applied, of which $24,000
+ ($.14 x 17,500) + ($77 x 25)], or $525.50 more ($483,000 - $459,000 actual OH) was overapplied.
than under the traditional system.
Answer (C) is incorrect because $11,000 equals the
difference between budgeted and actual overhead.
[185] Source: CMA 1296 3-18
Answer (D) is incorrect because the overhead was
Answer (A) is incorrect because a wallpaper overapplied.
manufacturer would use a process costing system.

Answer (B) is incorrect because a public accounting [188] Source: CIA 0594 III-47
firm would use a job costing system.
Answer (A) is incorrect because marketing and
Answer (C) is incorrect because a paint distribution costs should be allocated to specific
manufacturer products.
would use a process costing system.
Answer (B) is correct. ABC determines the activities
Answer (D) is correct. A job costing system is used that will serve as cost objects and then accumulates a
when products differ from one customer to the cost pool for each activity using the appropriate
next, activity base (cost driver). It is a system that may be
that is, when products are heterogeneous. A employed with job order or process costing methods.
process Thus, when there is only one product, the allocation
costing system is used when similar products are of costs to the product is trivial. All of the cost is
mass produced on a continuous basis. A print shop, assigned to the one product; the particular method
for example, would use a job costing system used to allocate the costs does not matter.
because
each job will be unique. Each customer provides Answer (C) is incorrect because ABC determines the
the activities that will serve as cost objects and then
specifications for the product desired. A beverage accumulates a cost pool for each activity using the
manufacturer, however, would use a process appropriate activity base (cost driver).
costing
system because homogenous units are produced Answer (D) is incorrect because, under ABC, a
continuously. product is allocated only those costs that pertain to its
production; that is, products are not
cross-subsidized.
[186] Source: CIA 1193 IV-4
[189] Source: CIA 1194 III-49 Answer (B) is incorrect because, if an allocation base
uniformly assigns costs to cost objects when the cost
Answer (A) is incorrect because $19,800 equals the objects use resources in a nonuniform way, the base
sum of 40% of S1's pre-allocation costs and 50% of is smoothing or spreading the costs. Smoothing can
S2's pre-allocation costs. result in undercosting or overcosting of products, with
adverse effects on product pricing, cost management
Answer (B) is incorrect because $21,949 is the total and control, and decision making.
service cost allocated to P1.
Answer (C) is incorrect because financial measures
Answer (C) is incorrect because $22,500 equals the (e.g., sales dollars and direct labor costs) and
average of the pre-allocation costs of S1 and S2. nonfinancial measures (e.g., setups and units shipped)
can be used as allocation bases.
Answer (D) is correct. The reciprocal method
allocates service department costs to other service Answer (D) is incorrect because high correlation
departments as well as to production departments between the cost items in a pool and the allocation
by base does not necessarily mean that a
means of simultaneous equations, as shown below. cause-and-effect relationship exists. Two variables
Thus, total service cost allocated to P2 is $23,051 may move together without such a relationship. The
[(40% x $31,224) + (50% x $21,122)]. perceived relationship between the cost driver
(allocation base) and the indirect costs should have
S1 = $27,000 + .2S2 S2 = $18,000 economic plausibility and high correlation.
+ .1($31,224)
$27,000 + [.2($18,000 + .1S1)] $18,000
+ $3,122 [193] Source: CIA 1195 III-93
$27,000 + $3,600 + .02S1 S2 = $21,122
.98S1 = $30,600 Answer (A) is incorrect because $5.39 assumes that
S1 = $31,224 80 machine hours are required for the total
production of 20,000 units.

[190] Source: CIA 0595 III-89 Answer (B) is incorrect because $5.44 is based on
the machining overhead rate ($18).
Answer (A) is incorrect because the
weighted-average method tends to smooth costs Answer (C) is correct. Given that manufacturing
over overhead is applied on the basis of machine hours,
time. the overhead rate is $60 per hour ($1,800,000 ・
30,000) or $.96 per unit [($60 x 80 machine hours
Answer (B) is incorrect because the FIFO method is per batch) ・5,000 units per batch]. Accordingly, the
more precise. It is based only on the work unit full cost is $6.11 ($5.15 unit prime cost + $.96).
completed
in the current period. Answer (D) is incorrect because $6.95 is based on
the direct labor hour manufacturing overhead rate.
Answer (C) is incorrect because the two methods
will provide dissimilar results when physical
inventory [194] Source: CIA 1195 III-94
levels and the production costs (materials and
conversion costs) fluctuate greatly from period to Answer (A) is incorrect because $6.00 assumes one
period. setup per batch and that total machine hours equaled
80.
Answer (D) is correct. The calculation of the cost
per Answer (B) is incorrect because $6.08 assumes that
equivalent unit using the FIFO method keeps the only 80 machine hours were used.
costs in one period separate from the costs of prior
periods. Under the weighted-average method, the Answer (C) is incorrect because $6.21 assumes one
costs in the beginning work-in-process inventory setup per batch.
are
combined with current-period costs. Answer (D) is correct. Materials handling cost per
part is $.12 ($720,000 ・6,000,000), cost per setup
is $420 ($315,000 ・750), machining cost per hour
is $18 ($540,000 ・30,000), and quality cost per
batch is $450 ($225,000 ・500). Hence, total
[192] Source: CIA 1195 III-41 manufacturing overhead applied is $22,920 [(5 parts
per unit x 20,000 units x $.12) + (4 batches x 2
Answer (A) is correct. A cost allocation base is the setups per batch x $420) + (4 batches x 80 machine
common denominator for systematically correlating hours per batch x $18) + (4 batches x $450)]. The
indirect costs and a cost object. The cost driver of total unit cost is $6.296 [$5.15 prime cost +
the indirect costs is ordinarily the allocation base. ($22,920 ・20,000 units) overhead].
In a
homogeneous cost pool, all costs should have the
same or a similar cause-and-effect relationship with [195] Source: CIA 1196 III-84
the cost allocation base.
Answer (A) is incorrect because activity-based includes 150,000 units plus 40% of BWIP, 80% of
costing develops cost pools for activities and then EWIP, and 70% of the defective units.
allocates those costs to cost objects based on the
drivers of the activities. Answer (C) is correct. The equivalent units for direct
materials equals 195,000 units (20,000 BWIP +
Answer (B) is incorrect because job-order costing 150,000 units started and completed + 25,000
accumulates costs by job or lot, for example, when EWIP). This calculation recognizes that direct
the company produces varied items on a materials had not been added to beginning inventory
custom-order basis. (60% complete) and spoiled units (70% complete),
but that ending inventory (80% complete) includes
Answer (C) is incorrect because operation costing is direct materials.
a hybrid of job-order costing and process costing.
Answer (D) is incorrect because 200,000 units
Answer (D) is correct. Process costing is used to includes 100% of the defective units.
assign costs to similar products that are mass
produced on a continuous basis. Because the
company has a continuous flow of a single product, [198] Source: CIA 1196 III-86
process costing is most likely to be used for its
manufacturing operations. Answer (A) is incorrect because abnormal spoilage is
not a normal result of efficient operations. In this
case, the percentage of spoiled units is within the
[196] Source: CIA 0596 III-77 normal tolerance limit.

Answer (A) is correct. Job-order costing is Answer (B) is correct. The units that failed inspection
appropriate when producing products with are classified as normal scrap because they have
individual minimal value and can be sold without further
characteristics and/or when identifiable groupings reworking. The defective units are less than the 4%
are tolerance limit for normal spoilage. Scrap can be
possible, e.g., batches of certain styles or types of sold, disposed of, or reused.
furniture. The unique aspect of job-order costing is
the identification of costs to specific units or a Answer (C) is incorrect because reworked units are
particular job. Thus, job-order costing is appropriate defective units that require further processing for them
for auto repair. Operation costing is a hybrid of to be sold. This company does not rework defective
job-order and process costing systems. It is used by units.
companies that manufacture goods that undergo
some similar and some dissimilar processes. Answer (D) is incorrect because waste has no
Operation costing accumulates total conversion monetary value and may have a disposal cost
costs associated with it.
and determines a unit conversion cost for each
operation. However, direct materials costs are
charged specifically to products as in job-order [199] Source: CIA 0596 III-82
systems. Operation costing is appropriate for
clothing Answer (A) is incorrect because $140,000 is 40% of
manufacturing. Process costing should be used to other traceable costs.
assign costs to similar products that are mass
produced on a continuous basis. Costs are Answer (B) is incorrect because $160,000 assumes
accumulated by departments or cost centers rather an allocation base of 55,000 square feet.
than by jobs, work in process is stated in terms of
equivalent units, and unit costs are established on Answer (C) is incorrect because $176,000 assumes
a an allocation base of 50,000 square feet, the base
departmental basis. Process costing is an averaging that would be used under the step method if the costs
process that calculates the average cost of all of building operations are allocated first.
units.
Process costing is appropriate for oil refining. Answer (D) is correct. The direct method does not
allocate service costs to other service departments.
Answer (B) is incorrect because custom printing Hence, the allocation base is the square footage in the
requires job-order costing. two production departments. Fabricating's share is
40% (16,000 ・40,000) of the total cost incurred by
Answer (C) is incorrect because process costing Building Operations, or $220,000 (40% x
should be used for paint manufacturing. $550,000).

Answer (D) is incorrect because job-order costing is


appropriate for motion picture production. [200] Source: CIA 0596 III-83

Answer (A) is incorrect because $657,000 results


[197] Source: CIA 1196 III-85 from allocating building operations costs first. Also,
the information services costs are allocated using total
Answer (A) is incorrect because 175,000 units does computer hours.
not include 20,000 units from BWIP.
Answer (B) is incorrect because $681,600 results
Answer (B) is incorrect because 181,500 units from allocating building operations costs to
Information Services.
Answer (C) is incorrect because, while charging
Answer (C) is incorrect because $730,000 allocates abnormal spoilage to manufacturing overhead is an
the costs of both service departments according to occasional practice, it is not the ordinary practice,
the direct method rather than the step method. which is to charge it to a special loss account.

Answer (D) is correct. The step method of service Answer (D) is correct. Abnormal spoilage is usually
department cost allocation is a sequential (but not charged to a special loss account in the period in
a which detection occurs because it is not normal or
reciprocal) process. These costs are allocated to anticipated.
other service departments as well as to users. The
process usually starts with the service department
that [203] Source: CMA 0697 3-4
renders the greatest percentage of its services to
other service departments, the department that Answer (A) is incorrect because operation costing is
provides services to the greatest number of other used for goods that undergo some similar and some
departments, or the department with the greatest dissimilar processes. Direct materials are costed on a
costs of services provided to other service job-order basis, whereas a unit conversion cost is
departments. If the $1,200,000 of information determined for each operation.
services costs is allocated first, the allocation base
is Answer (B) is incorrect because ABC emphasizes
2,000 computer hours (200 + 1,200 + 600). Thus, activities as the basic cost objects. It establishes cost
$120,000 [(200 ・2,000) x $1,200,000] will be pools for the activities and then reassigns those costs
allocated to Building Operations and $360,000 to other cost objects (e.g., products) on the basis of
[(600 their consumption of the related cost drivers.
・2,000) x $1,200,000] to Finishing. The total of the
building operations costs to be allocated to Answer (C) is incorrect because job-order costing
production equals $670,000 ($550,000 + accumulates costs by job rather than by department.
$120,000). The allocation base will be 40,000
square feet because no costs are allocated back to Answer (D) is correct. Process costing is used to
Information Services. Accordingly, the total of assign costs to similar products that are mass
service costs allocated to Finishing equals produced on a continuous basis. Costs are
$762,000 accumulated by departments or cost centers rather
{$360,000 + [$670,000 x (24,000 ・40,000)]}. than by jobs. Process costing is an averaging process
that calculates the average cost of all units.

[201] Source: CIA 0596 III-99


[204] Source: CMA 0695 3-1
Answer (A) is incorrect because the number of
customer phone calls has little relation to Answer (A) is incorrect because 97,600 units omits
distribution. the 6,400 EUP added to beginning work-in-process.
It is probably more closely related to customer
service. Answer (B) is correct. Under FIFO, EUP are based
solely on work performed during the current period.
Answer (B) is correct. The number of shipments is The EUP equals the sum of the work done on the
an beginning work-in-process inventory, units started
appropriate cost driver. A cause-and-effect and completed in the current period, and the ending
relationship may exist between the number of work-in-process inventory. Given that beginning
shipments and distribution costs. work-in-process was 60% complete as to materials,
the current period is charged for 6,400 EUP (40% x
Answer (C) is incorrect because the number of 16,000 units). Because 92,000 units were completed
sales during the period, 76,000 (92,000 - 16,000 in
persons is not related to distribution. It is more BWIP) must have been started and completed during
closely related to marketing. the period. They represent 76,000 EUP. Finally, the
EUP for ending work-in-process equal 21,600 (90%
Answer (D) is incorrect because the dollar sales x 24,000 units). Thus, total EUP for May are
volume is not necessarily related to distribution. It 104,000 (6,400 + 76,000 + 21,600).
is
more likely related to marketing. Answer (C) is incorrect because 107,200 units
assumes beginning work-in-process was 40%
complete.
[202] Source: CMA 0680 4-5
Answer (D) is incorrect because 108,000 units
Answer (A) is incorrect because normal spoilage equals the sum of the physical units in beginning
costs end up in inventory, not abnormal spoilage. work-in-process and the physical units completed.

Answer (B) is incorrect because material variance


accounts are charged only for the variances in [205] Source: CMA 0695 3-2
material usage or material price, not for the
spoilage Answer (A) is incorrect because 85,600 units omits
of product. the work done on beginning work-in-process.
Answer (A) is correct. The FIFO costs per
Answer (B) is incorrect because 88,800 units omits equivalent unit for materials and conversion costs are
$4.50 and $5.83, respectively. EUP for materials in
the work done on ending work-in-process. ending work-in-process equal 21,600 (90% x
24,000). Thus, total FIFO materials cost is $97,200
Answer (C) is incorrect because 95,200 units (21,600 EUP x $4.50). EUP for conversion costs in
assumes the beginning work-in-process was 40% ending work-in-process equal 9,600 (40% x
complete as to conversion costs. 24,000). Total conversion costs are therefore
$55,968 (9,600 EUP x $5.83). Consequently, total
Answer (D) is correct. The beginning inventory was work-in-process costs are $153,168 ($97,200 +
20% complete as to conversion costs. Hence, $55,968).
12,800 EUP (80% x 16,000 units) were required for
completion. EUP for units started and completed Answer (B) is incorrect because $154,800 is based
equaled 76,000 [100% x (92,000 completed units - on a FIFO calculation for materials and a
16,000 units in BWIP)]. The work done on ending weighted-average calculation for conversion costs.
work-in-process totaled 9,600 EUP (40% x 24,000
units). Thus, total EUP for May are 98,400 (12,800 Answer (C) is incorrect because $155,328 is based
+ 76,000 + 9,600). on a weighted-average calculation for materials and a
FIFO calculation for conversion costs.

[206] Source: CMA 0695 3-3 Answer (D) is incorrect because $156,960 is the
weighted-average cost of ending work-in-process.
Answer (A) is incorrect because $4.12 is based on
EUP calculated under the weighted-average
method. [209] Source: CMA 0695 3-6

Answer (B) is correct. Under the FIFO method, Answer (A) is incorrect because $4.12 equals
EUP for materials equal 104,000 [(16,000 units in materials costs for May divided by weighted-average
BWIP x 40%) + (76,000 units started and EUP.
completed x 100%) + (24,000 units in EWIP x
90%)]. Consequently, the equivalent unit cost of Answer (B) is incorrect because $4.50 is the
materials is $4.50 ($468,000 total materials cost in equivalent unit cost based on the FIFO method.
May ・104,000 EUP).
Answer (C) is correct. The weighted-average
Answer (C) is incorrect because $4.60 is the method averages the work done in the prior period
weighted-average cost per equivalent unit. with the work done in the current period. There are
two layers of units to analyze: those completed during
Answer (D) is incorrect because $4.80 omits the the period, and those still in ending inventory. The
units completed totaled 92,000. The 24,000 ending
6,400 EUP added to beginning work-in-process. units are 90% complete as to materials, so EUP
equal 21,600. Hence, total EUP for materials are
113,600 (92,000 + 21,600). The total materials
[207] Source: CMA 0695 3-4 costs incurred during the period and accumulated in
beginning work-in-process is $522,560 ($468,000 +
Answer (A) is incorrect because $5.65 is based on $54,560). Thus, weighted-average unit cost is $4.60
EUP calculated under the weighted-average ($522,560 ・113,600 EUP).
method.
Answer (D) is incorrect because $5.02 is based on a
Answer (B) is correct. Under the FIFO method, FIFO calculation of equivalent units and a
EUP for conversion costs equal 98,400 [(16,000 weighted-average calculation of costs.
units in BWIP x 80%) + (76,000 units started and
completed x 100%) + (24,000 units in EWIP x
40%)]. Conversion costs incurred during the current [210] Source: CMA 0695 3-7
period equal $574,040 ($182,880 DL + $391,160
Answer (A) is incorrect because $5.65 omits the
FOH). Hence, the equivalent unit cost for conversion costs in beginning work-in-process.
conversion
costs is $5.83 ($574,040 ・98,400). Answer (B) is incorrect because $5.83 is the
equivalent unit conversion cost based on FIFO.
Answer (C) is incorrect because $6.00 is the cost
per equivalent unit calculated under the Answer (C) is correct. The weighted-average
weighted-average method. method does not distinguish between the work done
in the prior period and the work done in the current
Answer (D) is incorrect because $6.20 results from period. Accordingly, the 92,000 completed units
combining conversion costs for May with those in represent 92,000 weighted-average EUP. The
beginning work-in-process and dividing by 98,400 24,000 units in ending work-in-process are 40%
EUP. complete as to conversion costs, so they equal 9,600
EUP. Hence, total EUP for conversion costs are
101,600 (92,000 + 9,600). The sum of the
[208] Source: CMA 0695 3-5 conversion costs accumulated in beginning
work-in-process and incurred during the period is
$609,600 ($20,320 + $15,240 + $182,880 + quantities of that input, the result may be an illogical
$391,160). Thus, weighted-average unit cost is allocation. However, if a firm manufactures only one
$6.00 ($609,600 ・101,600 EUP). product, this allocation method may be acceptable
because all costs are to be charged to the single
Answer (D) is incorrect because $6.20 is based on a product.
FIFO calculation of equivalent units and a
weighted-average calculation of costs.
[213] Source: CMA 0696 3-21

[211] Source: CMA 0695 3-8 Answer (A) is incorrect because a material amount
should be allocated among cost of goods sold,
Answer (A) is incorrect because $99,360 is the work-in-process, and finished goods.
weighted-average cost of materials in ending
work-in-process. Answer (B) is incorrect because a material amount
should be allocated among cost of goods sold,
Answer (B) is incorrect because $153,168 is the work-in-process, and finished goods.
FIFO cost of ending work-in-process.
Answer (C) is incorrect because a material amount
Answer (C) is incorrect because $154,800 is based should be allocated among cost of goods sold,
on a FIFO calculation for materials and a work-in-process, and finished goods.
weighted-average calculation for conversion costs.
Answer (D) is correct. Overapplied or underapplied
Answer (D) is correct. The weighted-average costs factory overhead should be disposed of at the end of
per equivalent unit for materials and conversion an accounting period by transferring the balance
costs either to cost of goods sold (if the amount is not
are $4.60 and $6.00, respectively. EUP for materials material) or to cost of goods sold, finished goods
in ending work-in-process equal 21,600 (90% x inventory, and work-in-process inventory.
24,000). Thus total weighted-average materials Theoretically, the allocation is preferred, but, because
cost the amount is usually immaterial, the entire balance is
is $99,360 ($4.60 x 21,600). EUP for conversion often transferred directly to cost of goods sold. Thus,
costs in ending work-in-process equal 9,600 (40% x the entry depends upon the significance of the
24,000 units). Total conversion costs are therefore amount.
$57,600 ($6.00 x 9,600 EUP). Consequently, total
ending work-in-process costs are $156,960
($99,360 + $57,600). [214] Source: CMA 0697 3-6

Answer (A) is incorrect because $72,000 is the


[212] Source: CMA 1290 3-4 variable cost allocation.

Answer (A) is incorrect because the number of units Answer (B) is incorrect because $122,000 assumes
of production may have no logical relationship to that fixed costs are allocated equally between A and
overhead when several different products are B.
made.
Answer (C) is incorrect because $132,000 assumes
Answer (B) is incorrect because a low level of direct fixed costs are allocated at a per-page rate based on
labor costs means that fixed overhead is available capacity ($100,000 ・4,000,000 pages =
substantial, $.025 per page), not on budgeted usage ($100,000
and an appropriate cost driver should be used to ・3,600,000 pages = $.0278 per page).
make the allocation.
Answer (D) is correct. Department B is budgeted to
Answer (C) is incorrect because the allocation use 66 2/3% of total production (2,400,000 ・
should 3,600,000), so it should be allocated fixed costs of
be made on the basis of the appropriate cost $66,667 (66 2/3% x $100,000). The variable cost
drivers allocation is $72,000 (2,400,000 pages x $.03 per
without regard to the relationship between direct page), and the total allocated is therefore $138,667
materials and labor costs. ($66,667 + $72,000).

Answer (D) is correct. Allocating overhead on the


basis of the number of units produced is usually not [215] Source: CMA 0697 3-7
appropriate. Costs should be allocated on the basis
of some plausible relationship between the cost Answer (A) is incorrect because $42,000 equals the
object and the incurrence of the cost, preferably variable costs allocated to Department A.
cause and effect. Overhead costs may be incurred
regardless of the level of production. When multiple Answer (B) is incorrect because $72,000 is the
products are involved, the number of units of allocation to Department B using a single rate.
production may bear no relationship to the
incurrence Answer (C) is correct. Based on budgeted usage,
of the allocated cost. If overhead is correlated with Department A should be allocated 33-1/3%
machine hours but different products require [1,200,000 pages ・(1,200,000 pages + 2,400,000
different pages)] of fixed costs, or $33,333 (33-1/3% x
$100,000). The variable costs are allocated at $.03
per unit for 1,400,000 pages, or $42,000. The sum
of the fixed and variable elements is $75,333. Answer (D) is incorrect because this amount is far
greater than could be allocated to a service
Answer (D) is incorrect because $82,000 assumes department with one employee.
fixed costs are allocated at a per-page rate based
on
actual usage ($100,000 ・3,500,000 pages = [218] Source: Publisher
$.0286 per page).
Answer (A) is correct. The total equivalent units for
raw materials equal 10,000 units (the total units
[216] Source: CMA 0697 3-8 placed into production) because all direct materials
were added to production. The materials cost per
Answer (A) is correct. The direct method allocates equivalent unit is therefore $6.60 ($66,000 total DM
service department costs directly to the producing costs ・10,000 equivalent units). The total equivalent
departments without recognition of services units for conversion costs equal 7,000 units [6,000
provided finished units + (25% x 4,000 units in EWIP]. Hence,
among the service departments. Hence, no service conversion cost per equivalent unit is $4.86 ($34,000
cost is allocated to the Tool Department because it total CC ・7,000 equivalent units). Total product
is cost per equivalent unit is $11.46 ($6.60 + $4.86),
a service department. so total transferred-out cost is $68,760 ($11.46 x
6,000 units transferred).
Answer (B) is incorrect because the direct method
does not recognize any allocation between or Answer (B) is incorrect because $60,000 assumes
among conversion costs are added at the beginning of the
service departments; only production departments process in the same manner as raw materials.
receive cost allocations.
Answer (C) is incorrect because $39,600 assumes
Answer (C) is incorrect because the direct method that only raw materials costs are transferred out.
does not recognize any allocation between or
among Answer (D) is incorrect because $29,160 is the
service departments; only production departments amount of conversion costs transferred out.
receive cost allocations.

Answer (D) is incorrect because the direct method [219] Source: Publisher
does not recognize any allocation between or
among Answer (A) is incorrect because $11,460 assumes
service departments; only production departments that the work-in-process inventory is only 1,000
receive cost allocations. units.

Answer (B) is incorrect because $26,400 includes


[217] Source: CMA 0697 3-9 only the cost of raw materials.

Answer (A) is incorrect because, under the Answer (C) is correct. Direct materials cost and
step-down method, other service departments conversion cost per equivalent unit are $6.60 and
share $4.86, respectively. Because the ending
in the allocation of costs. work-in-process contains 4,000 equivalent units of
direct materials costs (4,000 physical units x 100%)
Answer (B) is correct. Under the step-down method, and 1,000 equivalent units of conversion costs (4,000
service costs are allocated to all departments. physical units x 25%), its recorded balance is
However, no reciprocal allocations are allowed. The $31,260 (4,000 x $6.60) + (1,000 x $4.86).
process may begin with the department that
supports Answer (D) is incorrect because $45,840 assumes
the greatest number of departments, that incurs that the units are 100% complete as to conversion
the costs as well as materials.
greatest costs, or that provides the greatest
percentage of its services to other service
departments. Thus, the Repair Department is the [220] Source: Publisher
logical starting point. Given that service costs are
allocated to each department (service or Answer (A) is incorrect because $255,300
production) transposed the ending and beginning inventory
on the basis of its proportion of employees amounts.
(excluding
employees in the allocating department), the Answer (B) is incorrect because $257,800 is the
allocation of the Repair Department's overhead to amount of materials used without regard to changes in
the inventory levels.
Tool Department is $875 {$35,000 x [1 employee
・ Answer (C) is correct. Materials used equals
(1 + 2 + 25 + 12)]}. beginning inventory, plus purchases, minus ending
inventory. Given that purchases are not known, the
Answer (C) is incorrect because this amount is far calculation is as follows:
greater than could be allocated to a service
department with one employee.
$37,000 + P - $39,500 = $257,800
P = $257,800 + $39,500 - $37,000 Beginning materials inventory $ 67,000
P = $260,300 Purchases 179,300
Transportation in 4,400
Answer (D) is incorrect because $297,300 fails to Purchase returns and allowances (2,200)
consider the beginning inventory. --------
Materials available for use $248,500
Ending materials inventory (60,000)
[221] Source: Publisher ========
Materials used $188,500
Answer (A) is incorrect because $9.25 fails to Prime costs were $408,500 ($188,500 DM +
include overhead. $220,000 DL).

Answer (B) is correct. Cost of goods sold is based Answer (B) is incorrect because $188,500 equals
on the manufacturing costs incurred in production. materials used.
It
does not include selling or general and Answer (C) is incorrect because $181,500 equals
administrative purchases adjusted for purchase returns and
expenses. Manufacturing costs consist of direct allowances and transportation.
materials, $27,400; direct labor, $9,600; and
factory Answer (D) is incorrect because $365,200 equals
overhead, $20,000 (400 direct labor hours x $50 conversion costs.
per
hour). The total of these three cost elements is
$57,000. Dividing the $57,000 of total [224] Source: Publisher
manufacturing
costs by the 4,000 units produced results in a Answer (A) is incorrect because $553,700 is based
per-unit cost of $14.25. on actual overhead.

Answer (C) is incorrect because $14.95 includes Answer (B) is incorrect because $569,500 equals
administrative costs. total manufacturing costs plus the decrease in material
inventory.
Answer (D) is incorrect because $17.75 includes
selling and administrative costs. Answer (C) is incorrect because $408,500 equals
prime costs.

[222] Source: Publisher


Answer (D) is correct. Total manufacturing costs
Answer (A) is incorrect because $150 per order consist of materials, labor, and overhead. The cost of
equals $4,500 divided by 30 orders. materials used during the month was $188,500.
Given $220,000 of direct labor and overhead applied
Answer (B) is incorrect because ABC yields an of $154,000 (70% x $220,000 of direct labor), the
allocation $404 higher than the traditional system. total production costs are $562,500 ($188,500 +
$220,000 + $154,000).
Answer (C) is incorrect because $4,500 is the
quality
control cost under the traditional system. [225] Source: Publisher

Answer (D) is correct. ABC assigns overhead costs Answer (A) is incorrect because $543,700 results
on the basis of multiple cost drivers instead of only from using actual instead of applied overhead and
one driver. Using the three cost drivers in the adding the decrease in finished goods to the cost of
question produces the following calculation: goods completed, which is not done.

Number of types Answer (B) is incorrect because $552,500 results


of materials ($12 x 12) $ 144 from adding the decrease in finished goods to the
Number of units ($.14 x 17,500) 2,450 cost of goods completed.
Number of orders ($77 x 30) 2,310
------ Answer (C) is incorrect because $528,700 results
Total $4,904 from using actual instead of applied overhead.
======
Under the old method of allocation based on direct Answer (D) is correct. Total manufacturing costs
labor, the allocated amount would have been 4,500 incurred during the period consist of materials, labor,
(15% x $30,000), or $404 lower than under ABC. and overhead. That amount is then adjusted for the
change in work-in-process inventories to arrive at the
cost of goods transferred to finished goods (cost of
[223] Source: Publisher goods manufactured). The total manufacturing costs
incurred during the month were $562,500.
Answer (A) is correct. Prime costs are incurred for Work-in-process increased during October by
direct materials and direct labor. The first step is to $25,000 ($170,000 EWIP - $145,000 BWIP).
calculate the cost of materials used during the Accordingly, the cost of goods completed (cost of
month: goods manufactured) during October was $537,500
($562,500 manufacturing costs - $25,000 increase
in Answer (D) is correct. Factory overhead is usually
EWIP). assigned to products based on a predetermined rate
or rates. The activity base for overhead allocation
should have a high correlation with the incurrence of
[226] Source: Publisher overhead. Given only one cost driver, one overhead
application rate is sufficient. If products differ in the
Answer (A) is incorrect because $537,500 is the resources consumed in individual departments,
cost of goods manufactured. multiple rates are preferable.

Answer (B) is correct. The cost of goods sold equals


cost of goods manufactured ($537,500) adjusted [229] Source: CMA 1296 3-28
for
the change in finished goods. Answer (A) is correct. ABC differs from traditional
product costing because it uses multiple allocation
Beginning finished goods $ 85,000 bases and therefore allocates overhead more
Cost of goods manufactured 537,500 accurately. The result is that ABC often charges
-------- low-volume products with more overhead than a
Goods available for sale $622,500 traditional system. For example, the cost of machine
Ending finished goods (70,000) setup may be the same for production runs of widely
-------- varying sizes. This relationship is reflected in an ABC
system that allocates setup costs on the basis of the
Cost of goods sold $552,500 number of setups. However, a traditional system
======== using an allocation base such as machine hours may
underallocate setup costs to low-volume products.
Answer (C) is incorrect because $522,500 results Many companies adopting ABC have found that they
from reversing the beginning and ending finished have been losing money on low-volume products
goods inventories. because costs were actually higher than originally
thought.
Answer (D) is incorrect because $543,700 is based
on actual overhead costs. Answer (B) is incorrect because low-volume
products are usually charged with greater unit costs
under ABC.
[227] Source: Publisher
Answer (C) is incorrect because greater setup costs
Answer (A) is incorrect because an overapplication are usually charged to low-volume products under
of overhead is represented by a credit in the ABC.
overhead control account. Actual costs are debited
and applied costs are credited. Answer (D) is incorrect because setup costs will not
be equalized unless setup time is equal for all
Answer (B) is incorrect because the overhead was products.
overapplied. Actual costs were less than those
charged to production.
[230] Source: CMA 1293 3-15
Answer (C) is correct. Factory overhead control is
debited for actual overhead and, if a single control Answer (A) is incorrect because one rate may be
account is used, credited for applied overhead. This cost beneficial when a single product proceeds
account should have been debited for $145,200 of through homogeneous processes.
actual overhead and credited for $154,000 (70% x
$220,000 DL) of applied overhead. Thus, a net Answer (B) is correct. Multiple rates are appropriate
credit of $8,800 ($154,000 - $145,200) resulted when a process differs substantially among
from an overapplication of overhead. departments or when products do not go through all
departments or all processes. The trend in cost
Answer (D) is incorrect because the $8,800 credit is accounting is toward activity-based costing, which
an overapplication of overhead. divides production into numerous activities and
identifies the cost driver(s) most relevant to each. The
result is a more accurate tracing of costs.
[228] Source: CMA 1296 3-19
Answer (C) is incorrect because, if cost drivers are
Answer (A) is incorrect because a standard cost the same for all processes, multiple rates are
system can be based on individual or multiple unnecessary.
application rates.
Answer (D) is incorrect because individual cost
Answer (B) is incorrect because whether production drivers for all relationships must be known to use
is machine intensive affects the nature but not multiple application rates.
necessarily the number of cost drivers.

Answer (C) is incorrect because a single plant-wide [231] Source: Publisher


application rate is acceptable, even with high
overhead, if all overhead is highly correlated with a Answer (A) is incorrect because $32,000 is based
single application base. on 10,000 equivalent units of conversion costs.
total costs incurred in production.
Answer (B) is correct. The equivalent units of
materials equal 10,000 because all materials are
added at the beginning of the process, and 10,000 [234] Source: Publisher
units were started. The equivalent units of
conversion Answer (A) is incorrect because $69,259 results
costs equal 9,400 [8,000 units completed + (70% x from using the equivalent units calculated under FIFO
2,000 units in ending inventory)]. The unit cost of (81,000) in determining the unit conversion cost
materials is $1.50 ($15,000 ・10,000 EU). The unit under the weighted-average method.
cost of conversion is $2.66 ($25,000 ・9,400 EU).
Thus, the cost of goods transferred was $33,280 Answer (B) is correct. For conversion costs, the
[8,000 units x ($1.50 + $2.66)]. equivalent-unit calculation under the
weighted-average method is as follows:
Answer (C) is incorrect because $36,280 assumes
transfer of 10,000 units of materials. Beginning WIP 10,000 units x 100% = 10,000
Started and completed 75,000 units x 100% =
Answer (D) is incorrect because $40,000 equals 75,000
total costs incurred in production. Ending WIP 5,000 units x 60% = 3,000
------
88,000
[232] Source: Publisher ======
The conversion costs consisted of $16,000 in
Answer (A) is correct. The cost transferred out was beginning inventory and $50,000 incurred during the
$33,280. Hence, the ending inventory must equal month, for a total of $66,000. Unit conversion cost is
the therefore $.75 ($66,000 ・$88,000 EU). Thus, the
production costs for the month (given no beginning total conversion cost transferred was $63,750 [$.75
inventories), minus costs transferred out, or $6,720 x (10,000 units in BWIP + 80,000 units started -
[($15,000 materials + $25,000 conversion cost) - 5,000 units in EWIP)].
$33,280].
Answer (C) is incorrect because $66,000 equals the
Answer (B) is incorrect because $8,000 assumes total conversion costs to be accounted for.
that
ending work-in-process contains 2,000 equivalent Answer (D) is incorrect because $64,148 is the
units of conversion costs. conversion cost transferred out under a FIFO
assumption.
Answer (C) is incorrect because $3,720 assumes
that ending work-in-process contains only
conversion [235] Source: Publisher
costs.
Answer (A) is incorrect because $88,000 is the
Answer (D) is incorrect because some units remain materials costs incurred during the month.
in
work-in-process. Answer (B) is incorrect because $93,500 results
from using a unit cost based on the FIFO method.

[233] Source: Publisher Answer (C) is correct. For materials, the


equivalent-unit calculation under the
Answer (A) is incorrect because $32,000 is based weighted-average method is
on 10,000 equivalent units of conversion costs.
Beginning WIP 10,000 units x 100% = 10,000
Answer (B) is correct. The only difference between Started and completed 75,000 units x 100% =
weighted average and FIFO relates to the beginning 75,000
inventories. Because there were no beginning Ending WIP 5,000 units x 100% = 5,000
inventories in this problem, the two valuation ------
methods 90,000
produce the same results. The equivalent units of ======
materials equal 10,000 because all materials are The materials costs consisted of $30,000 in beginning
added at the beginning of the process, and 10,000 inventory and $88,000 incurred during the month, for
units were started. The equivalent units of a total of $118,000. The equivalent unit cost of
conversion materials is therefore $1.31 ($118,000 ・90,000
costs equal 9,400 [8,000 units completed + (70% x EU). Total materials cost transferred is $111,350
2,000 units in ending inventory)]. The unit cost of (85,000 units transferred x $1.31).
materials is $1.50 ($15,000 ・10,000 EU). The unit
cost of conversion is $2.66 ($25,000 ・9,400 EU). Answer (D) is incorrect because $112,500 is the
Thus, the cost of goods transferred was $33,280 materials cost transferred out under FIFO.
[8,000 units x ($1.50 + $2.66)].

Answer (C) is incorrect because $36,280 assumes [236] Source: Publisher


transfer of 10,000 units of materials.
Answer (A) is incorrect because 75,000 units is the
Answer (D) is incorrect because $40,000 equals amount started and completed during the month; it
ignores the impact of inventories.
Answer (B) is incorrect because $111,350 is based
Answer (B) is incorrect because 80,000 units is on the weighted-average method.
based on the FIFO method.
Answer (C) is correct. For materials, the
Answer (C) is incorrect because 81,000 units is equivalent-unit calculation under the FIFO method is
based on the equivalent units for conversion costs
calculated under the FIFO method. Beginning WIP 10,000 units x 0% = 0
Started and completed 75,000 units x 100% =
Answer (D) is correct. The equivalent units for 75,000
transferred-in costs are calculated in the same way Ending WIP 5,000 units x 100% = 5,000
as ------
those for materials added at the beginning of the 80,000
process. The equivalent-unit calculation under the ======
weighted-average method is The materials cost includes $30,000 in beginning
inventory, all of which would have been transferred
Beginning WIP 10,000 units x 100% = out. The $88,000 incurred during the month is
10,000 divided by the 80,000 equivalent units to arrive at a
Started and completed 75,000 units x 100% = unit cost for the current period of $1.10. Thus, given
75,000 that 75,000 equivalent units (85,000 physical units
Ending WIP 5,000 units x 100% = 5,000 transferred out - 10,000 EU in BWIP completed in
------ the prior period) of current-period production were
90,000 completed and transferred, total materials cost
======
transferred out equals $112,500 [$30,000 BWIP +
($1.10 x 75,000 FIFO EU)].
[237] Source: Publisher
Answer (D) is incorrect because $114,615 is based
on the equivalent units for conversion costs.
Answer (A) is incorrect because $63,750 is based
on the weighted-average method.
[239] Source: Publisher
Answer (B) is correct. For conversion costs, the
equivalent-unit calculation under the FIFO method Answer (A) is correct. The FIFO unit conversion
is cost for the current period is $.62. Moreover, ending
work-in-process consists of 3,000 equivalent units of
Beginning WIP 10,000 units x 30% = conversion cost (5,000 physical units x 60%).
3,000 Accordingly, the conversion cost in the ending
Started and completed 75,000 units x 100% = work-in-process inventory consists of $1,860 ($.62
75,000 x 3,000 EU) of current-period cost. The conversion
Ending WIP 5,000 units x 60% = 3,000 cost incurred in the prior period and attached to the
------ beginning work-in-process inventory is deemed to
81,000 have been transferred out.
======
The conversion cost includes $16,000 in beginning Answer (B) is incorrect because $2,250 is based on
inventory, all of which would have been transferred the weighted-average method.
out. The $50,000 incurred during the month is
divided by the 81,000 equivalent units to arrive at Answer (C) is incorrect because $3,100 is based on
a the equivalent units for materials.
unit cost for the current period of $.62. Given that
78,000 equivalent units (85,000 physical units Answer (D) is incorrect because $5,500 is the
transferred out - 7,000 EU in BWIP completed in the amount of materials cost in the ending
prior period) of current-period production were work-in-process inventory.
completed and transferred, the total conversion
cost
transferred out was $64,360 [$16,000 BWIP + [240] Source: Publisher
($.62 x 78,000 FIFO EU)].
Answer (A) is incorrect because $1,860 is the
Answer (C) is incorrect because $66,000 equals amount of conversion costs.
total conversion costs incurred.
Answer (B) is incorrect because $3,300 assumes that
Answer (D) is incorrect because $74,500 is based materials are added proportionately throughout the
on the weighted-average unit cost per equivalent process.
unit.
Answer (C) is correct. The unit cost of materials
under FIFO is $1.10. Because the 5,000 units in
[238] Source: Publisher ending work-in-process inventory are 100%
complete as to materials, its materials cost consists of
Answer (A) is incorrect because $88,000 is the $5,500 (5,000 EU x $1.10) of current-period costs.
amount of materials costs incurred during the Materials costs incurred in the prior period and
month. attached to the beginning work-in-process inventory
are deemed to have been transferred out.
[243] Source: Publisher
Answer (D) is incorrect because $6,450 is based on
the unit cost under the weighted-average method. Answer (A) is incorrect because $2.06 results from
dividing May's costs by the equivalent units calculated
under the weighted-average method.
[241] Source: Publisher
Answer (B) is correct. The FIFO equivalent units of
Answer (A) is incorrect because 195,200 units materials equal 208,000. Accordingly, unit cost of
omits materials under FIFO is $2.25 ($468,000 materials
the 12,800 equivalent units of work on BWIP during cost in May ・208,000 EU).
the current period.
Answer (C) is incorrect because $2.30 is the
Answer (B) is correct. Under the FIFO method, weighted-average cost per equivalent unit.
equivalent units are determined based only on
work Answer (D) is incorrect because $2.51 equals total
performed during the current period. They include materials cost divided by FIFO equivalent units of
work performed to complete BWIP, work on units materials.
started and completed during the period, and work
done on EWIP. Thus, total FIFO equivalent units of
materials are [244] Source: Publisher

BWIP 32,000 units x 40% = Answer (A) is correct. The FIFO equivalent units of
12,800 conversion cost equal 196,800. Conversion cost
Started and completed incurred during May was $574,040 ($182,880 DL +
(184,000 - 32,000 in BWIP) 152,000 units x 100% $391,160 FOH). Hence, the equivalent-unit
= 152,000 conversion cost under FIFO is $2.92 ($574,040 ・
EWIP 48,000 units x 90% = 196,800).
43,200
------- Answer (B) is incorrect because $3.00 is the cost per
Total equivalent units 208,000 equivalent unit under the weighted-average method.
=======
Answer (C) is incorrect because $3.10 equals total
Answer (C) is incorrect because 214,400 units conversion cost divided by FIFO equivalent units of
assumes that BWIP was 40% complete. conversion cost.

Answer (D) is incorrect because 227,200 units is Answer (D) is incorrect because $3.23 assumes
based on the weighted-average method. EWIP is 0% complete as to conversion cost.

[242] Source: Publisher [245] Source: Publisher

Answer (A) is incorrect because 171,200 units Answer (A) is correct. Under FIFO, the
omits equivalent-unit materials cost is $2.25, and the EWIP
work on BWIP. contains 43,200 equivalent units of materials. The
equivalent-unit conversion cost is $2.92, and the
Answer (B) is incorrect because 177,600 units EWIP contains 19,200 equivalent units of conversion
omits cost. Consequently, EWIP equals $153,264 [($2.25
work on EWIP. x 43,200) + ($2.92 x 19,200)].

Answer (C) is incorrect because 184,000 equals the Answer (B) is incorrect because $154,800 is based
physical units completed. on a FIFO calculation for materials and a
weighted-average calculation for conversion cost.
Answer (D) is correct. Under FIFO, equivalent units
are determined based only on work performed Answer (C) is incorrect because $155,424 is based
during on a weighted-average calculation for materials and a
the current period. They include work performed to FIFO calculation for conversion cost.
complete BWIP, work on units started and
completed during the period, and work done on Answer (D) is incorrect because $156,960 is based
EWIP. Thus, total FIFO equivalent units of on the unit costs under the weighted-average method.
conversion cost are

BWIP 32,000 units x 80% = 25,600 [246] Source: Publisher


Started and completed
(184,000 - 32,000 BWIP) 152,000 units x 100% = Answer (A) is incorrect because $2.06 is based on
152,000 weighted-average equivalent units and FIFO costs.
EWIP 48,000 units x 40% = 19,200
------- Answer (B) is incorrect because $2.25 is the cost
Total equivalent units 196,800 based on the FIFO method.
=======
Answer (C) is correct. The weighted-average
method averages the work done in the prior period the unit conversion cost was $3.00. Furthermore, the
with the work done in the current period. The two equivalent units for materials equaled 43,200 (48,000
layers of units to analyze are those completed physical units x 90%), and the equivalent units for
during conversion cost equaled 19,200 (48,000 physical
the period and those still in EWIP. The units units x 40%). The total weighted-average cost of
completed totaled 184,000. The equivalent units of EWIP was therefore $156,960 [($2.30 x 43,200 EU
materials in EWIP equaled 43,200 (48,000 physical of materials) + ($3.00 x 19,200 EU of conversion
units x 90%). Hence, the total equivalent units of cost)].
materials equaled 227,200 (184,000 + 43,200).
The
materials cost in BWIP is combined in the [249] Source: CIA 0597 III-75
weighted-average calculation with the materials
cost Answer (A) is incorrect because any identification
incurred during the current period. The method may fail to record the movement of some
equivalent-unit materials cost is therefore $2.30 parts.
[($54,560 BWIP + $468,000 incurred in May) ・
227,200 EU]. Answer (B) is correct. Bar-code scanning is a form
of optical character recognition. Bar codes are a
Answer (D) is incorrect because $2.51 equals total series of bars of different widths that represent critical
materials costs divided by FIFO equivalent units of information about the item. They can be read and the
materials. information can be instantly recorded using a scanner.
Thus, bar coding records the movement of parts with
minimal labor costs.
[247] Source: Publisher
Answer (C) is incorrect because each vendor has its
Answer (A) is incorrect because $2.92 is the FIFO own part-numbering scheme.
unit cost.

Answer (B) is correct. The weighted-average Answer (D) is incorrect because each vendor has its
method own identification method, although vendors in the
averages the work performed in the prior period same industry often cooperate to minimize the
with number of bar-code systems they use.
the work done in the current period. The two layers
of units to analyze are those completed during the
period and those still in EWIP. The units completed [250] Source: Publisher
totaled 184,000. The 48,000 units in EWIP are 40%
complete as to conversion cost, the equivalent of Answer (A) is correct. This service organization
19,200 units. Thus, total equivalent units for produces three "products" (the three occupant
conversion cost under the weighted-average categories), and the "units produced" equal occupant
method days. According to the ABC analysis, production
equaled 203,200. Moreover, the conversion cost in involves two activities: (1) provision of residential
BWIP is combined in the weighted-average space and meals and (2) OOA. The drivers of these
calculation with the conversion cost incurred during activities are occupant days and nursing hours,
the current period. The equivalent-unit conversion respectively. Thus, the cost pool rate for the first
cost is therefore $3.00 [($20,320 DL in BWIP + activity (residential space and meals) is $120 per
$15,240 FOH in BWIP + $182,880 DL in May + occupant day ($7,200,000 ・60,000 days), and the
$391,160 FOH in May) ・203,200]. cost pool rate for the second activity (OOA) is $25
($7,500,000 ・300,000 hours). The total cost for
Answer (C) is incorrect because $3.10 is based on providing services to occupants in the low-usage
FIFO equivalent units and weighted-average cost. category is $6,570,000 [($120 x 36,000 days) +
($25 x 90,000 hours)]. The daily cost rate for these
Answer (D) is incorrect because $3.31 results from occupants is therefore $182.50 ($6,570,000 ・
omitting the equivalent units in EWIP. 36,000 occupant days).

Answer (B) is incorrect because $145.00 equals the


[248] Source: Publisher sum of the cost pool rates ($120 + $25).

Answer (A) is incorrect because $153,264 is the Answer (C) is incorrect because $245 equals the rate
FIFO EWIP. per day for medium-usage occupants.

Answer (B) is incorrect because $154,800 is based Answer (D) is incorrect because $620 is the rate per
on a FIFO calculation for materials and a day for high-usage occupants.
weighted-average calculation for conversion cost.

Answer (C) is incorrect because $155,424 is based [251] Source: CMA 0693 3-2
on a weighted-average calculation for materials
and a Answer (A) is incorrect because one effect of
FIFO calculation for conversion cost. computerization is that the amount of direct labor
relative to other costs has been decreasing. For this
Answer (D) is correct. Given a weighted-average reason, some companies have found that it is no
assumption, the unit materials cost was $2.30, and longer expedient to track direct labor costs as closely
as was once done. Thus, some companies are inventory ($500,000) + cost of goods manufactured
treating direct labor as an indirect factory overhead ($860,000) - ending finished goods inventory
cost. ($990,000) = cost of goods sold ($370,000). The
work-in-process inventories are irrelevant.
Answer (B) is incorrect because throughput time is
one of the cost drivers that is beginning to be used Answer (C) is incorrect because $490,000
more often as an overhead application base. represents the difference between beginning and
Throughput is the rate of production over a stated ending inventories.
time. This rate clearly drives (influences) costs.
Answer (D) is incorrect because $1,350,000 is the
Answer (C) is correct. With the recent automation result of reversing the treatment of beginning and
of ending finished goods inventories.
factories and the corresponding emphasis on
activity-based costing (ABC), companies are finding
new ways of allocating indirect factory overhead. [254] Source: Publisher
One change is that plant-wide application rates are
being used less often because a closer matching of Answer (A) is correct. The predetermined overhead
costs with cost drivers provides better information application rate is found by dividing the total
to budgeted overhead by the budgeted direct labor
management. ABC results in a more accurate cost. Therefore, the predetermined overhead
application of indirect costs because it provides application rate is 1.78 [$961,200 ・(36,000 x
more $15)].
refined data. Instead of a single cost goal for a
process, a department, or even an entire plant, an Answer (B) is incorrect because 1.83 results from
indirect cost pool is established for each identified dividing total budgeted overhead by the actual direct
activity. The related cost driver, the factor that labor cost.
changes the cost of the activity, is also identified.
Answer (C) is incorrect because 2.09 results from
Answer (D) is incorrect because multiple cost pools dividing total actual overhead by the budgeted direct
are preferable. They permit a better matching of labor cost.
indirect costs with cost drivers.
Answer (D) is incorrect because 2.15 results from
dividing total actual overhead by the actual direct
[252] Source: CMA 1292 3-2 labor cost.

Answer (A) is incorrect because making allocations


on the basis of units sold may not meet the [255] Source: Publisher
cause-and-effect criterion.
Answer (A) is incorrect because $195,500 is the
Answer (B) is incorrect because the salary of amount underapplied.
service
department employees is the cost allocated, not a Answer (B) is incorrect because $168,800 results
basis of allocation. from subtracting actual incurred overhead from total
budget overhead.
Answer (C) is correct. Service department costs are
considered part of factory overhead and should be Answer (C) is incorrect because $168,800 results
allocated to the production departments that use from subtracting actual incurred overhead from total
the budget overhead.
services. A basis reflecting cause and effect should
be Answer (D) is correct. The amount of factory
used to allocate service department costs. For overhead overapplied/underapplied is found by
example, the number of kilowatt hours used by subtracting the actual incurred overhead from the
each actual applied overhead. The actual applied overhead
producing department is probably the best is $934,500 [(35,000 hours x $15) x 1.78].
allocation Therefore, the amount of underapplied overhead is
base for electricity costs. $195,500 ($934,500 - $1,130,000).

Answer (D) is incorrect because making allocations


on the basis of materials usage may not meet the [256] Source: Publisher
cause-and-effect criterion.
Answer (A) is incorrect because $0 is the amount
allocated to work-in-process inventory.
[253] Source: Publisher
Answer (B) is incorrect because $39,100 is the
Answer (A) is incorrect because $360,000 results amount allocated to finished goods inventory.
from subtracting the difference between beginning
and ending work-in-process inventories from the Answer (C) is correct. Since the amount of
cost underapplied overhead is considered material, the
of goods sold. proper accounting treatment is to prorate this amount
to work-in-process, finished goods inventory, and the
Answer (B) is correct. Beginning finished goods cost of goods sold. Thus, the ending balances must
be added together to get $2,151,200 ($1,720,960 In this case, conversion costs are equal to the direct
+ manufacturing labor cost ($9,000) + indirect
$430,240). The amount of underapplied overhead manufacturing costs ($20,000), or $29,000.
is
then multiplied by .8 ($1,720,960 ・$2,151,200) to Answer (D) is incorrect because $98,000
get the amount of underapplied overhead allocated erroneously includes materials and excludes labor.
to
cost of goods sold, which is $156,400.
[260] Source: Publisher
Answer (D) is incorrect because $195,500 is the
amount of overhead underapplied. Answer (A) is incorrect because 0 units of abnormal
spoilage means all 13,000 rejected units were normal
spoilage.
[257] Source: Publisher
Answer (B) is incorrect because 1,000 units of
Answer (A) is incorrect because $10.46 per abnormal spoilage is the amount on the initial
machine production cost report.
hour results from dividing the actual overhead by
the Answer (C) is incorrect because 1,300 units of
budgeted machine hours. abnormal spoilage is found by subtracting the initial
number of abnormal spoilage units from the correct
Answer (B) is correct. The predetermined overhead amount.
rate is found by dividing total budget overhead by
budgeted machine hours. Therefore, the budgeted Answer (D) is correct. The number of units of
overhead of $961,200 is divided by the budget abnormal spoilage in this process is found by taking
machine hours of 108,000 to get a predetermined 10% of good output as normal spoilage, and then
overhead rate of $8.90 per machine hour. calculating the difference to find abnormal spoilage.
Therefore, 10% of 107,000 good output units is
Answer (C) is incorrect because $8.69 per machine 10,700, and 13,000 rejected units minus 10,700
hour results from dividing the actual overhead by normal spoilage units is equal to 2,300 units of
the abnormal spoilage.
actual machine hours.

Answer (D) is incorrect because $7.39 per machine [261] Source: Publisher
hour results from dividing budgeted overhead by
the Answer (A) is incorrect because $40,446 is the total
actual machine hours. cost of good units completed without regard to
normal spoilage.

[258] Source: Publisher Answer (B) is correct. The first step in finding the
total cost of good units produced is to calculate the
Answer (A) is incorrect because the $20,000 is unit cost of production. This number is equal to the
overhead. total cost of production divided by total input units, or
$.378 ($45,360 ・120,000 units). The next step is to
Answer (B) is incorrect because $79,000 results multiply the unit cost of production times the number
from using the cost of material purchases instead of good units completed to get a total cost of good
of units of $40,446 (107,000 x .378). This number is
materials used. then added to the total cost of normal spoilage
$4,044.60 (10,700 x .378) to get a total cost of
Answer (C) is correct. Prime costs are all direct good units of $44,490.60.
manufacturing costs--in this case, direct materials
cost ($78,000) + direct manufacturing labor cost Answer (C) is incorrect because $44,940 is the total
($9,000), or $87,000. cost of good units completed in the initial production
cost report.
Answer (D) is incorrect because $98,000 uses
overhead instead of labor. Answer (D) is incorrect because $45,360 is the total
cost of all production.

[259] Source: Publisher


[262] Source: Publisher
Answer (A) is incorrect because $4,000 is the
difference between the beginning and ending Answer (A) is incorrect because $420 is the total
work-in-process inventories. cost of abnormal spoilage in the initial production cost
report.
Answer (B) is incorrect because $20,000 excludes
direct labor. Answer (B) is correct. The total cost of abnormal
spoilage is found by multiplying the number of units of
Answer (C) is correct. Conversion costs are all abnormal spoilage times the unit cost of production.
manufacturing costs other than direct material Therefore, the total cost of abnormal spoilage is
costs. $869.40 (2,300 units x $.378).
Answer (C) is incorrect because $966 is found by completed and transferred plus 4,000 equivalent units
multiplying the number of units of abnormal from ending WIP inventory is equal to 23,000
spoilage equivalent units.
by the initial unit cost of good units.
Answer (D) is incorrect because 25,000 units does
Answer (D) is incorrect because $4,914 is the not take into consideration work previously
combined cost of normal spoilage and abnormal completed or work still needing to be completed in
spoilage. work-in-process inventories.

[263] Source: Publisher [265] Source: Publisher

Answer (A) is incorrect because 20,000 units is the Answer (A) is incorrect because $11.91 is the result
amount completed and transferred. of dividing the total cost using the weighted-average
method, by the number of equivalent units using the
Answer (B) is incorrect because 21,000 units is the FIFO method.
result of adding the number of equivalent units in
the Answer (B) is correct. The cost per equivalent unit
ending work-in-process inventory that still need to for a cost element using the weighted-average
be method is found by dividing the total cost by the
completed to the 20,000 units. equivalent units. In this case, the total cost was
$274,000 ($45,600 from beginning work-in-process
Answer (C) is correct. The amount of conversion + $228,400 from May's production). This number is
equivalent units produced using the weighted- divided by the 24,000 equivalent units to get a cost
average per equivalent unit of $11.42.
method is found by adding the number of units
completed and transferred to the equivalent Answer (C) is incorrect because $9.93 is the cost
amount per equivalent unit using the FIFO method.
of units in the ending work-in-process inventory.
Since there are 5,000 gallons in the ending Answer (D) is incorrect because $9.52 is the result of
work-in-process inventory that are 80% completed, dividing the $228,400 from May's production by the
the equivalent amount of units is 4,000 (5,000 x number of equivalent units.
80%). Therefore, 4,000 equivalent units in ending
work-in-process inventory plus 20,000 units
completed and transferred is equal to 24,000 [266] Source: Publisher
equivalent units.
Answer (A) is incorrect because $1.90 is the result of
Answer (D) is incorrect because 25,000 units is the dividing the total cost of the FIFO method by the
result of adding the total number of units in the equivalent units of the weighted-average method.
ending
work-in-process inventory to the 20,000 units. Answer (B) is correct. The cost per equivalent unit
for a cost element using the FIFO method is found by
dividing the total cost by the equivalent units. While
[264] Source: Publisher using the FIFO method, the only costs are those
incurred during the month of May, $45,500 ($35,000
Answer (A) is incorrect because 16,000 units is the + $10,500) and the equivalent units are the ones
amount completed and transferred. completed from beginning work-in-process
inventory, 3,000 ($4,000 x .75), those completed
Answer (B) is incorrect because 19,000 units does and transferred out, 16,000 units, and the amount in
not include the equivalent units from ending ending work-in-process, 4,000 (5,000 x .8).
work-in-process inventory. Therefore, the cost per equivalent unit is $1.98
($45,500 ・23,000 units).
Answer (C) is correct. The number of conversion
equivalent units produced using the FIFO method is Answer (C) is incorrect because $2.23 is the
found by adding together the equivalent units equivalent cost per unit using the weighted-average
produced from beginning work-in-process method.
inventory,
the current production transferred, and the Answer (D) is incorrect because $2.33 is the result of
equivalent dividing the total cost of the weighted-average
units produced in ending work-in-process inventory. method by the equivalent units of the FIFO method.
Since there were 4,000 units in beginning
work-in-process inventory that were 25%
completed, then 3,000 equivalent units were [267] Source: CMA Samp Q3-4
produced this month [4,000 units x (1 - .25)]. Since
there were 5,000 units in ending work-in-process Answer (A) is incorrect because 74,000 results from
inventory that were 80% completed, there were the omission of all spoiled units from the computation.
4,000 equivalent units from ending inventory
(5,000 x Answer (B) is incorrect because 74,150 results from
.8). Therefore, 3,000 equivalent units from the omission of normal spoilage from the
beginning computation.
WIP inventory plus 16,000 (21,000 - 5,000) units
Answer (C) is incorrect because 74,600 results from 250,000) increase.
the omission of abnormal spoilage from the
computation. Answer (D) is incorrect because 4% is the
percentage increase in actual unit sales over budgeted
Answer (D) is correct. Given no BWIP, the total unit sales [(260 units - 250 units) ・250 units].
equivalent units for July equal 74,750.

Physical [270] Source: CMA 0683 4-9


Equivalent
Units Units
-------- ---------- Answer (A) is incorrect because the total variance
Started and completed during July 65,000 between actual and budgeted sales is $115,000
65,000 (favorable). The sales price variance is $65,000
Normal spoilage (30% complete) 2,000 (unfavorable) and the sales quantity variance is
600 $180,000 (favorable), for a total favorable variance
Abnormal spoilage (100% complete) of $115,000. The sales price variance is unfavorable
150 150 because actual sales were less than budgeted sales,
Ending work-in-process inventory (60% complete) and the sales quantity variance is favorable because
15,000 9,000 actual quantity exceeded budgeted quantity.
------ ------
82,150 74,750 Answer (B) is incorrect because the total variance
====== between actual and budgeted sales is $115,000
====== (favorable). The sales price variance is $65,000
(unfavorable) and the sales quantity variance is
$180,000 (favorable), for a total favorable variance
[268] Source: CMA 0683 4-7 of $115,000. The sales price variance is unfavorable
because actual sales were less than budgeted sales,
Answer (A) is correct. According to the budget, and the sales quantity variance is favorable because
sales of 250 units would produce a contribution actual quantity exceeded budgeted quantity.
margin of $700, or $2.80 per unit. Dividing the
$448 Answer (C) is incorrect because the actual sales
of budgeted fixed costs by $2.80 gives a breakeven price for Xeon was less than the budgeted sales
point of 160 units. The 260 actual units sold price, creating a $65,000 unfavorable sales price
produced a contribution margin of $650, or $2.50 variance.
per unit. Dividing the $420 of fixed costs by $2.50
gives a breakeven point of 168 units. Consequently, Answer (D) is correct. First, compute what sales
the actual breakeven point is 5% (8 ・160) above would have been if all sales had been made at
the budgeted prices. The budgeted prices were $6 and
budget. $10 per unit, respectively. Actual sales in units
multiplied by the budgeted prices equals total sales of
Answer (B) is incorrect because 6.67% is the $2,080,000. Since actual sales were only
percentage difference between the actual and the $2,015,000, the variance of $65,000 is unfavorable
budgeted fixed costs [(420 - 448) ・448]. (fewer sales than budgeted).

Answer (C) is incorrect because 6.67% is the


percentage difference between the actual and the [271] Source: CMA 0683 4-10
budgeted fixed costs [(420 - 448) ・448].
Answer (A) is incorrect because the variable quantity
Answer (D) is incorrect because the percentage (not unit cost) variance is $165,000 (unfavorable).
difference between the actual and the budgeted The variable quantity variance is unfavorable because
breakeven point was 5.00% above (not below) the actual expenses are greater than budgeted expenses.
budget.
Answer (B) is incorrect because $137,000 is the
total variance for variable and fixed expenses. It is
[269] Source: CMA 0683 4-8 unfavorable because actual expenses are greater than
budgeted expenses.
Answer (A) is incorrect because only 20% of the
increased volume is due to an improved market
share, while the other 80% is due to the increased Answer (C) is incorrect because $137,000 is the
industry volume. total variance for variable and fixed expenses. It is
unfavorable because actual expenses are greater than
Answer (B) is incorrect because 80% of the budgeted expenses.
increased volume was due to the increased
industry Answer (D) is correct. The proper procedure is to
volume. calculate variable costs by multiplying actual units
sold by the budgeted costs per unit. The budgeted
Answer (C) is correct. Based on the revised industry costs were $3 and $7.50, respectively. The actual
volume, Xerbert should have sold 258,000 units costs ($3 x 130 = $390 and $7.50 x 130 = $975)
(10% x 2,580,000). Since 260,000 were actually were identical to budgeted variable costs at the actual
sold, the company increased its market share by volume, i.e., zero variance.
2,000 which is 20% of the 10,000 (260,000 -
$20X = $1,200,000
[272] Source: CMA 1285 4-4 X = 60,000 units
At a selling price of $60 each, the total revenue is
Answer (A) is incorrect because $80,000 is the $3,600,000 ($60 x 60,000 units).
income before the tax expense of $(24,000)
($80,000 x 30%). Answer (B) is incorrect because $3,312,000 is the
annual sales revenue that results when the $96,000 of
Answer (B) is incorrect because ($800,000) is the income tax is ignored.
loss that would result if sales volume per year
(instead Answer (C) is incorrect because $1,656,000 is the
of per month) were 4,000 units. annual sales revenue when the $96,000 of income tax
is ignored and the sum of the fixed costs and net
Answer (C) is correct. The income statement for a income ($1,104,000 = $880,000 fixed costs +
volume of 48,000 units (4,000 per month x 12 $224,000 net income) is divided by the variable unit
months) would appear as follows: cost of $40 (instead of the contribution margin of
$20).
Sales ($60/unit) $2,880,000
Variable manufacturing ($35/unit) Answer (D) is incorrect because $3,110,400 is the
(1,680,000) annual sales revenue when the $96,000 of income is
Variable selling ($5/unit) (240,000) subtracted from (instead of added to) the $224,000.
----------
Contribution margin $ 960,000
Fixed costs (880,000) [275] Source: CMA 1285 4-7
----------
Income before tax $ 80,000 Answer (A) is incorrect because 30% is the
Tax expense (30%) (24,000) contribution margin percentage that results from
---------- including manufacturing overhead as a prime cost.
Net income $ 56,000
========== Answer (B) is incorrect because 76% is the
contribution margin percentage that results from
Answer (D) is incorrect because a $(560,000) loss dividing total variable costs by the sales price
would result if the $(800,000) loss was reduced for ($45.60 ・60).
the tax savings of $240,000 ($800,000 x 30%).
Answer (C) is incorrect because 20% is the
contribution margin percentage that results from
[273] Source: CMA 1285 4-5 treating all variable costs as prime costs.

Answer (A) is incorrect because 22,000 units is Answer (D) is correct. Prime costs are direct
fixed materials and direct labor. Since these two elements
cost ($880,000) divided by variable costs ($40). totaled $28 ($16 + $12) before the increase, the new
total would be $33.60 ($28 x 1.2). In other words,
Answer (B) is correct. The breakeven point in units prime costs would increase by $5.60, and total
equals fixed costs divided by the contribution variable costs would increase to $45.60. Subtracting
margin $45.60 from the $60 selling price leaves a
per unit. At a selling price of $60 per unit and with contribution margin of $14.40. The contribution
variable costs of $40 per unit, the unit contribution margin percentage thus becomes 24% ($14.40 ・
margin is $20. Thus, the breakeven point is 44,000 $60).
units ($880,000 ・$20).

Answer (C) is incorrect because the contribution [276] Source: CMA 0686 4-28
margin should reflect selling expenses.
Answer (A) is incorrect because fixed costs should
Answer (D) is incorrect because there are no be divided by the contribution margin, not unit
income variable costs.
taxes at the breakeven point.
Answer (B) is incorrect because 91,000 units is total
cost ・selling price.
[274] Source: CMA 1285 4-6
Answer (C) is incorrect because selling and
Answer (A) is correct. The formula for solving this administrative costs are included in the contribution
problem is Sales - Variable costs - Fixed costs - margin.
Taxes = $224,000. The $224,000 is equal to 70%
(the complement of the tax rate) of before-tax Answer (D) is correct. The breakeven point in units is
income. Thus, before-tax income must equal found by dividing the fixed costs ($210,000) by
$320,000 ($224,000 ・.7) and taxes would be contribution margin per unit ($3). Variable costs are
$96,000. If sales equal $60 times X units and $700,000 at 100,000 units, or $7 per unit. Selling
variable costs are $40 times X units, the following price is $10 per unit. Dividing $210,000 by $3 per
is unit results in a breakeven point of 70,000 units.
the equation to solve for units sold (X):

$60X - $40X - $880,000 - $96,000 = 224,000 units [277] Source: CMA 0686 4-29
last year's sales price of $7.50 and last year's variable
Answer (A) is incorrect because selling 100,000 cost of $2.25.
units
results from ignoring incomes.
[280] Source: CMA 0687 4-11
Answer (B) is correct. Since the tax rate is 40%, the
$90,000 would consist of 60% of before-tax Answer (A) is incorrect because 17,500 is based on
income.
Thus, to earn an income of $90,000 after tax, the the preceding year's fixed costs.
before-tax income must be $150,000 ($90,000 ・
.6). Dividing the fixed costs of $210,000 plus the Answer (B) is correct. The breakeven point (BEP) in
desired before-tax income of $150,000 by the $3 units equals fixed cost divided by UCM. Fixed cost
contribution margin per unit gives a breakeven for the previous year was $105,000 (20,000 units at
point breakeven x $5.25 UCM). Fixed cost for the current
of 120,000 units. year is $115,500 (110% x $105,000). The new
UCM is $6 ($9 selling price - $3 variable cost).
Answer (C) is incorrect because 102,858 units Accordingly, the BEP is 19,250 units ($115,500 ・
disregards the $50,000 of selling and $6).
administrative
costs in computing the contribution margin. Answer (C) is incorrect because 20,000 is the
preceding year's BEP.
Answer (D) is incorrect because 145,000 units
results from using a 60% tax rate rather than a Answer (D) is incorrect because 22,000 is the
40% breakeven point if the current year's fixed costs of
tax rate. $115,500 (110% x $105,000) is divided by last
year's contribution margin of $5.25.

[278] Source: CMA 0686 4-30


[281] Source: CMA 0687 4-12
Answer (A) is incorrect because fixed costs should
be divided by the contribution margin, not the unit Answer (A) is correct. Since last year's after-tax
variable cost. profit was $5,040, pretax net income must have been
$8,400 [$5,040 ・(1 - 40% tax rate)]. Because fixed
Answer (B) is correct. The breakeven point is fixed cost has been fully recovered at the BEP, all of the
costs divided by the contribution margin per unit. UCM beyond that sales level is included in pre-tax
With the increase, the new total for fixed costs net income. The UCM was $5.25, so the units sold in
would excess of the 20,000-unit BEP equaled 1,600
be $241,500; the contribution margin per unit ($8,400 ・$5.25). If 21,600 total units were sold last
would year, an increase of 1,000 units results in sales of
still be $3. The breakeven point is 80,500 units. 22,600 units.

Answer (C) is incorrect because selling and


administrative costs are reflected in the Answer (B) is incorrect because 21,960 units
contribution assumes the last year's after-tax profit was the pretax
margin. net income.

Answer (D) is incorrect because 94,150 units is Answer (C) is incorrect because sales volume is
total estimated to be 22,600 units [20,000 ・($8,400
costs of $941,500 divided by selling price of $10. pretax NI ・$5.25 UCM) ・1,000].

Answer (D) is incorrect because sales volume is


[279] Source: CMA 0687 4-10 estimated to be 22,600 units [20,000 ・($8,400
pretax NI ・$5.25 UCM) ・1,000].
Answer (A) is incorrect because $9.00 is the
expected price, not the price with the same CMR.
[282] Source: CMA 0687 4-13
Answer (B) is incorrect because $8.25 is the sum of
the old UCM and the new unit variable cost. Answer (A) is incorrect because $213,750 is the
sales revenue when fixed costs are not increased by
Answer (C) is correct. Last year, unit variable cost 10%.
was $2.25, so the unit contribution margin (UCM)
was $5.25 ($7.50 price - $2.25), and the Answer (B) is incorrect because $257,625 results
contribution margin rate (CMR) was 70% ($5.25 ・ from using a 60% tax rate rather than a 40% tax rate.
$7.50). If variable costs increase by one-third, the
new variable cost will be $3 [$2.25 x (4 ・3)]. If a Answer (C) is incorrect because $207,000 results
70% CMR is desired, the $3 variable cost will be from failing to convert the after-tax income of
30% of sales, and the unit sales price will be $10 $22,500 to pre-tax income of $37,500 [$22,500 ・
($3 (1 - 40% tax rate)].
・30%).
Answer (D) is correct. An after-tax net income of
Answer (D) is incorrect because $9.75 is the sum of $22,500 equals a pretax income of $37,500
[$22,500 ・(1 - 40% tax rate)]. With a UCM of $6 only costs that are capitalized are the variable costs
contributing toward the $153,000 total of fixed cost of manufacturing. These include
($115,500) and desired profit ($37,500), 25,500
units ($153,000 ・$6) must be sold. At $9 per unit, Direct materials used $300,000
sales revenue is $229,500. Direct labor 100,000
Variable factory overhead 50,000
--------
[283] Source: CMA 0687 4-14 Total inventoriable costs $450,000
========
Answer (A) is incorrect because the sales quantity
(volume) variance focuses on the firm's aggregate Answer (C) is incorrect because the $40,000 of
results. It assumes a constant product mix and an variable selling and administrative costs should not be
average contribution margin for the composite unit. included in the inventoriable costs.
The sales volume variance equals the budgeted
Answer (D) is incorrect because $530,000 is the
average UCM calculated for the composite unit inventoriable cost under absorption (full) costing.
multiplied by the difference between the actual and
budgeted unit sales.
[286] Source: CMA 1286 4-19
Answer (B) is incorrect because it is the rate of
return Answer (A) is incorrect because $400,000 equals
a potential investment must earn before it is prime costs.
acceptable to management.
Answer (B) is incorrect because $450,000 is the
Answer (C) is incorrect because it is a nonsense inventoriable cost under variable costing.
term.
Answer (C) is correct. The absorption method is
Answer (D) is correct. The margin of safety required for financial statements prepared according
measures to GAAP. It charges all costs of production to
the amount by which sales may decline before inventories. The variable cost of materials
losses ($300,000), direct labor ($100,000), variable factory
occur. It equals budgeted or actual sales minus overhead ($50,000), and the fixed factory overhead
sales ($80,000) are included. They total $530,000.
at the BEP. It may be stated in either units sold or
sales revenue. Answer (D) is incorrect because $590,000 includes
the fixed and variable selling and administrative costs.

[284] Source: CMA 1296 3-25


[287] Source: CMA 1290 3-24
Answer (A) is incorrect because, given excess
capacity, any price less than $47 is advantageous. Answer (A) is incorrect because ending inventory
was $1,200,000.
Answer (B) is incorrect because, given excess
capacity, any price greater than $47 is Answer (B) is correct. Under the absorption method,
uneconomical. unit cost is $30 ($12 direct materials + $9 direct
labor + $4 variable overhead + $5 fixed overhead).
Answer (C) is correct. The total unit cost includes Given beginning inventory of 35,000 units, the ending
$20 of fixed costs ($17 + $3) that is not avoidable inventory equals 40,000 units (35,000 BI + 130,000
if produced - 125,000 sold). Hence, ending inventory
the units are purchased. Moreover, the company was $1,200,000 ($30 x 40,000 units).
has
excess capacity. The opportunity cost of making the
table tops is zero because no production will be Answer (C) is incorrect because ending inventory
displaced. Consequently, the relevant unit cost of was $1,200,000.
making the table tops is the $47 unit variable cost,
and the supplier's price must be less to justify the Answer (D) is incorrect because ending inventory
buy was $1,200,000.
decision.

Answer (D) is incorrect because the decision to [288] Source: CMA 1290 3-25
reject a price of $50 or more may depend on
whether the firm must displace other production to Answer (A) is incorrect because ending inventory
make the table tops. was $1,000,000.

Answer (B) is incorrect because ending inventory


[285] Source: CMA 1286 4-18 was $1,000,000.

Answer (A) is incorrect because $400,000 does not Answer (C) is correct. Using variable costing, the unit
include $50,000 of variable factory overhead. cost of ending inventory is $25 ($12 direct materials
+ $9 direct labor + $4 variable overhead). Given
Answer (B) is correct. Under variable costing, the beginning inventory of 35,000 units, the ending
inventory equals 40,000 units (35,000 BI + 130,000 fixed overhead (125,000 units x $5). At year-end,
produced - 125,000 sold). Thus, ending inventory the underapplied overhead was also added to cost of
was $1,000,000 ($25 x 40,000). goods sold. Because production was expected to be
140,000 units, the overhead application rate for the
Answer (D) is incorrect because ending inventory $700,000 of planned fixed manufacturing overhead
was $1,000,000. was $5 per unit. Only 130,000 units were
manufactured. Hence, $650,000 ($5 x 130,000
units) of overhead was applied to units in process.
[289] Source: CMA 1290 3-29 Because inventory increased from 35,000 to 40,000
units, $25,000 (5,000-unit increase x $5) of the
Answer (A) is correct. Absorption costing results in applied fixed manufacturing overhead for the period
a was inventoried, not expensed. Actual overhead was
higher income figure than variable costing $715,000, so the underapplied overhead was
whenever $65,000 ($715,000 - $650,000). This amount was
production exceeds sales because absorption charged to cost of goods sold at year-end. The total
costing of the fixed costs expensed was therefore
capitalizes some fixed factory overhead as part of $2,095,000 ($980,000 selling expenses + $425,000
inventory. These costs are expensed during the administrative expenses + $625,000 standard
period incurred under variable costing. manufacturing overhead costs of units sold +
Consequently, $65,000 underapplied overhead).
variable costing recognizes greater expenses and
lower income. The reverse is true when sales Answer (B) is incorrect because the total fixed costs
exceed on the absorption costing basis were $2,095,000.

production. In that case, the absorption method Answer (C) is incorrect because the total fixed costs
results in a lower income because some fixed costs on the absorption costing basis were $2,095,000.
of
previous periods absorbed by the beginning Answer (D) is incorrect because the total fixed costs
inventory on the absorption costing basis were $2,095,000.
are expensed in the current period as cost of goods
sold. Variable costing income is never burdened
with [291] Source: CMA 1290 3-27
fixed costs of previous periods.
Answer (A) is correct. The unit variable
Answer (B) is incorrect because an increase in manufacturing cost was $25 ($12 direct materials +
inventory results in a higher income under $9 direct labor + $4 variable overhead). Other
absorption variable costs included selling expenses ($8 per unit)
costing. and administrative expenses ($2 per unit). The unit
selling and administrative costs actually incurred for
Answer (C) is incorrect because the important
relationship is between actual production and sales of 125,000 units were the same as the planned
actual unit costs. For example, actual unit variable selling
sales, not between actual and planned production. expense was $8 ($1,000,000 ・125,000 units sold),
which equaled the planned unit cost. Thus, total unit
Answer (D) is incorrect because planned sales do variable cost was $35 ($25 + $8 + $2). The total
not expensed was $4,375,000 ($35 x 125,000 units
determine actual income. sold).

Answer (B) is incorrect because the total variable


[290] Source: CMA 1290 3-28 cost expensed on the variable costing basis was
$4,375,000.
Answer (A) is correct. Under the absorption
method, Answer (C) is incorrect because the total variable
all selling and administrative fixed costs are cost expensed on the variable costing basis was
charged $4,375,000.
to the current period. Accordingly, $980,000 of
selling expenses and $425,000 of actual fixed Answer (D) is incorrect because the total variable
administrative expenses were expensed during the cost expensed on the variable costing basis was
year. The fixed manufacturing costs must be $4,375,000.
calculated after giving consideration to the
increase in
inventory during the period (some fixed costs were [292] Source: CMA 1290 3-30
capitalized) and to the underapplied overhead. The
beginning finished goods inventory included 35,000 Answer (A) is incorrect because the difference
units, each of which had absorbed $5 of fixed between absorption costing and variable costing
manufacturing overhead. Each unit produced income was $25,000.
during
the year also absorbed $5 of fixed manufacturing Answer (B) is correct. The difference is caused by
overhead. Given that 125,000 of those units were the capitalization of some of the fixed manufacturing
sold, cost of goods sold was debited for $625,000 overhead. When inventories increase during the
of period, the absorption method capitalizes that
overhead and transfers it to future periods. The
variable costing method expenses it in the current Answer (D) is correct. Under absorption costing, all
period. Inventories increased by 5,000 units during manufacturing costs, both fixed and variable, are
the period, and each of those units would have treated as product costs. Under variable costing, only
included $5 of fixed manufacturing overhead under variable costs of manufacturing are inventoried as
absorption costing. Accordingly, $25,000 of fixed product costs. Fixed manufacturing costs are
manufacturing overhead would have been expensed as period costs. Packaging and shipping
capitalized. costs are not product costs under either method
Recognizing $25,000 of fixed costs in the balance because they are incurred after the goods have been
sheet instead of the income statement results in a manufactured. Instead, they are included in selling
$25,000 difference in income between the two and administrative expenses for the period.
costing methods.

Answer (C) is incorrect because the difference [295] Source: CMA 1292 3-6
between absorption costing and variable costing
Answer (A) is incorrect because ABC is appropriate
income was $25,000. for external as well as internal purposes.

Answer (D) is incorrect because the difference Answer (B) is incorrect because job-order costing is
between absorption costing and variable costing acceptable for external reporting purposes.
income was $25,000.
Answer (C) is correct. Activity-based costing,
job-order costing, process costing, and standard
[293] Source: CMA 1290 3-26 costing can all be used for both internal and external
purposes. Variable costing is not acceptable under
Answer (A) is incorrect because the variable costing GAAP for external reporting purposes.
contribution margin was $5,625,000.
Answer (D) is incorrect because process costing is
Answer (B) is incorrect because the variable costing acceptable for external reporting purposes.
contribution margin was $5,625,000.

Answer (C) is incorrect because the variable costing [296] Source: CMA 1292 3-26
contribution margin was $5,625,000.
Answer (A) is incorrect because producing more of
Answer (D) is correct. At $70 per unit, actual sales the products requiring the most direct labor will
revenue was $8,750,000 for 125,000 units. Actual permit more fixed overhead to be capitalized in the
variable costs of manufacturing were $25 per unit inventory account.
($12 + $9 + $4). The unit costs incurred for the
actual production level of 130,000 units were the Answer (B) is incorrect because deferring expenses
same as the unit costs for a planned production such as maintenance will increase income in the
level current period (but may result in long-range losses
of 140,000 units. These unit costs were the same caused by excessive down-time).
for
units manufactured in both the current and Answer (C) is incorrect because increasing
previous production without a concurrent increase in demand
year. For example, total planned direct materials applies more fixed costs to inventory.
cost
for 140,000 units was $1,680,000, or $12 per unit. Answer (D) is correct. Under an absorption costing
The incurred unit cost was also $12 ($1,560,000 ・ system, income can be manipulated by producing
130,000 units). Thus, total variable manufacturing more products than are sold because more fixed
cost was $3,125,000 ($25 x 125,000 units). manufacturing overhead will be allocated to the
Consequently, manufacturing contribution margin ending inventory. When inventory increases, some
was fixed costs are capitalized rather than expensed.
$5,625,000 ($8,750,000 - $3,125,000). Decreasing production, however, will result in lower
income because more of the fixed manufacturing
overhead will be expensed.
[294] Source: CMA 1292 3-5

Answer (A) is incorrect because manufacturing [297] Source: CMA 1273 4-1
supplies are variable costs inventoried under both
methods. Answer (A) is incorrect because the cost of a unit of
product changing because of a change in number of
Answer (B) is incorrect because factory insurance is units manufactured is a characteristic of absorption
costing systems.
a fixed manufacturing cost inventoried under
absorption costing but written off as a period cost Answer (B) is correct. In a variable costing system,
under variable costing. only the variable costs are recorded as product costs.
All fixed costs are expensed in the period incurred.
Answer (C) is incorrect because direct labor cost is Because changes in the relationship between
a production levels and sales levels do not cause
product cost under both methods. changes in the amount of fixed manufacturing cost
expensed, profits more directly follow the trends in Selling and administrative expenses
sales. (400,000)
----------
Answer (C) is incorrect because idle facility Operating income $ 440,000
variation ==========
is a characteristic of absorption costing systems.
Answer (C) is incorrect because $600,000 is the
Answer (D) is incorrect because neither variable nor operating income that results from capitalizing
absorption costing includes administrative costs in $240,000 fixed manufacturing costs and $160,000 of
inventory. selling and administrative costs (the $160,000 is
incorrect as all selling and administrative costs should
be expensed).
[298] Source: CMA 1273 4-2
Answer (D) is incorrect because $840,000 is the
Answer (A) is incorrect because profit is a function gross profit under absorption costing, i.e., before
of selling and administrative expenses.
both sales and production, so it will not always
move
in the same direction as sales. [300] Source: CMA 1285 4-15

Answer (B) is incorrect because profit is a function Answer (A) is correct. The contribution margin from
of manufacturing (sales - variable costs) is $10 ($40 -
both sales and production, so it will not always $30) per unit sold, or $1,200,000 (120,000 units x
move $10). The fixed costs of manufacturing ($600,000)
in the same direction as sales. and selling and administrative costs ($400,000) are
deducted from the contribution margin to arrive at an
Answer (C) is correct. In an absorption costing operating income of $200,000. The difference
system, fixed overhead costs are included in between the absorption income of $440,000 and the
inventory. When sales exceed production, more $200,000 of variable costing income is attributable to
overhead is expensed under absorption costing due capitalization of the fixed manufacturing costs under
to fixed overhead carried over from the prior the absorption method. Since 40% of the goods
inventory. If sales increase over production, more produced are still in inventory (80,000 ・200,000),
than one period's factory overhead is recognized as 40% of the $600,000 in fixed costs, or $240,000,
expense. Accordingly, if the increase in factory was capitalized under the absorption method. That
overhead expensed is greater than the contribution amount was expensed under the variable costing
margin of the increased units sold, there may be method.
less
profit with an increased level of sales. Answer (B) is incorrect because $440,000 is the
operating income under absorption costing.
Answer (D) is incorrect because decreased output
will increase the unit cost of items sold. Fixed Answer (C) is incorrect because $800,000 is the
factory operating income if fixed costs of manufacturing are
overhead per unit will increase. not deducted.

Answer (D) is incorrect because $600,000 is the


[299] Source: CMA 1285 4-14 operating income that results from capitalizing 40% of
both fixed manufacturing costs and selling and
Answer (A) is incorrect because $200,000 is the administrative costs.
operating income under variable costing.

Answer (B) is correct. Absorption costing net [301] Source: CMA 0694 3-4
income
is computed as follows: Answer (A) is correct. The breakeven point in units is
calculated by dividing the fixed costs by the
Sales (120,000 x $40) contribution margin per unit. If selling price is constant
$4,800,000 and costs increase, the unit contribution margin will
---------- decline, resulting in an increase of the breakeven
Variable production costs ($30 x 200,000 units) point.
$6,000,000
Fixed production costs Answer (B) is incorrect because a decrease in costs
600,000 will lower the breakeven point. The unit contribution
---------- margin will increase.
Total production costs (200,000 units)
$6,600,000 Answer (C) is incorrect because an increase in the
Ending inventory (80,000 x $33) selling price will also increase the unit contribution
(2,640,000) margin, resulting in a lower breakeven point.
----------
Cost of goods sold Answer (D) is incorrect because both a cost
$3,960,000 decrease and a sales price increase will increase the
---------- unit contribution margin, resulting in a lower
Gross profit $ 840,000 breakeven point.
Answer (C) is incorrect because variable costs are
[302] Source: CMA 0697 3-2 the same under either method.

Answer (A) is incorrect because variable costs will Answer (D) is incorrect because gross margins will
change in total, but unit variable costs will be be different. Fixed factory overhead is expensed
constant. under variable costing and capitalized under the
absorption method.
Answer (B) is correct. The relevant range is the
range
of activity over which unit variable costs and total [305] Source: CMA adap
fixed costs are constant. The incremental cost of
one Answer (A) is incorrect because $3,000 is the value
additional unit of production will be equal to the of the by-product.
variable cost.
Answer (B) is correct. The NRV at split-off for each
Answer (C) is incorrect because actual fixed costs of the joint products must be determined. Given that
should not vary greatly from budgeted fixed costs Alfa has a $4 selling price and an additional $2 of
for processing costs, the value at the split-off is $2 per
the relevant range. pound. the total value at split-off for 10,000 pounds
is $20,000. Betters has a $10 selling price and an
Answer (D) is incorrect because the relevant range additional $2 of processing costs. Thus, the value at
can change whenever production activity changes; split-off is $8 per pound. The total value of 5,000
the pounds of Betters is therefore $40,000. The 1,000
relevant range is merely an assumption used for pounds of Morefeed has a split-off value of $3 per
budgeting and control purposes. pound, or $3,000. Assuming that Morefeed (a
by-product) is inventoried (recognized in the
accounts when produced) and treated as a reduction
[303] Source: CMA 0697 3-3 of joint costs, the allocable joint cost is $90,000
($93,000 - $3,000). (NOTE: Several other methods
Answer (A) is incorrect because variable (direct) of accounting for by-products are possible.) The total
costing does not inventory fixed factory overhead. net realizable value of the main products is $60,000
($20,000 Alfa + $40,000 Betters). The allocation to
Answer (B) is incorrect because variable (direct) Alfa is $30,000 [($20,000 ・$60,000) x $90,000].
costing does not inventory fixed factory overhead.
Answer (C) is incorrect because $31,000 fails to
Answer (C) is correct. Absorption (full) costing adjust the joint processing cost for the value of the
considers all manufacturing costs to be by-product.
inventoriable
as product costs. These costs include variable and Answer (D) is incorrect because $60,000 is the
fixed manufacturing costs, whether direct or amount allocated to Betters.
indirect.
The alternative to absorption is known as variable
(direct) costing. [306] Source: CMA 1293 3-7

Answer (D) is incorrect because conversion costs Answer (A) is incorrect because $30,000 is the
include direct labor and factory overhead but not amount allocated to Alfa when the by-product is
direct materials. inventoried.

Answer (B) is incorrect because $31,000 is the


[304] Source: CMA 0697 3-10 amount allocated to Alfa when the by-product is not
inventoried.
Answer (A) is incorrect because fixed factory
overhead is treated differently under the two Answer (C) is incorrect because $52,080 assumes
methods. that a weighting method using caloric value is used.

Answer (B) is correct. Under variable costing, Answer (D) is correct. The NRV of Alfa is $20,000,
inventories are charged only with the variable costs and the NRV of Betters is $40,000. If the joint cost
of is not adjusted for the value of the by-production, the
production. Fixed manufacturing costs are amount allocated to Betters is $62,000 {[$40,000 ・
expensed ($20,000 + $40,000)] x $93,000}.
as period costs. Absorption costing charges to
inventory all costs of production. If finished goods
inventory increases, absorption costing results in [307] Source: CMA 1293 3-4
higher income because it capitalizes some fixed
costs Answer (A) is incorrect because $3,000 is the value
that would have been expensed under variable of the by-product.
costing. When inventory declines, variable costing
results in higher income because some fixed costs Answer (B) is incorrect because $30,000 is based on
capitalized under the absorption method in prior the net realizable value method.
periods are expensed in the current period.
Answer (C) is incorrect because $31,000 is based
on the net realizable value method and fails to Answer (A) is correct. The total production costs
adjust incurred are $10,000,000, consisting of crude oil of
the joint processing cost for the value of the $5,000,000, direct labor of $2,000,000, and factory
by-product. overhead of $3,000,000. The total physical output
was 660,000 barrels, consisting of 300,000 barrels
Answer (D) is correct. Joint cost is $93,000 and of Two Oil, 240,000 barrels of Six Oil, and 120,000
Morefeed has a split-off value of $3,000 (see barrels of distillates. Thus, the allocation (rounded) is
preceding question). Assuming the latter amount is $3,636,000 {[240,000 ・(300,000 + 240,000 +
treated as a reduction in joint cost, the allocable 120,000)] x $10,000,000}.
joint
cost is $90,000. The total physical quantity Answer (B) is incorrect because $3,750,000 is
(volume) based on the physical quantity of units sold, not units
of the two joint products is 15,000 pounds (10,000 produced.
Alfa + 5,000 Betters). Hence, $60,000 of the net
joint costs [(10,000 ・15,000) x $90,000] should be Answer (C) is incorrect because $1,818,000 is the
allocated to Alfa. amount that would be assigned to distillates.

Answer (D) is incorrect because Six Oil does not


[308] Source: CMA 1293 3-5 compose 75% of the total output in barrels.

Answer (A) is incorrect because $39,208 is the


amount allocated to Alfa if the 1,000,000 calories [311] Source: CMA 1295 3-30
attributable to Morefeed is included in the
computation. Answer (A) is incorrect because $4,800,000 is the
amount that would be assigned to Six Oil.
Answer (B) is incorrect because $39,600 is the
allocation to Alfa. Answer (B) is correct. The total production costs
incurred are $10,000,000, consisting of crude oil of
Answer (C) is incorrect because $40,920 is the $5,000,000, direct labor of $2,000,000, and factory
allocation to Alfa if the sales value of the by- overhead of $3,000,000. The total value of the
product output is as follows:
is not treated as a reduction of joint cost.
Two Oil (300,000 x $20) $ 6,000,000
Answer (D) is correct. The net allocable joint cost is Six Oil (240,000 x $30) 7,200,000
$90,000, assuming the value of Morefeed is Distillates (120,000 x $15) 1,800,000
inventoried and treated as a reduction in joint -----------
costs. Total sales value $15,000,000
The caloric value of Alfa is 44,000,000 (4,400 x ===========
10,000 pounds), the caloric value of Betters is Because Two Oil composes 40% of the total sales
56,000,000 (11,200 x 5,000 pounds), and the total value ($6,000,000 ・$15,000,000), it will be
is 100,000,000. Of this total volume, Alfa makes up assigned 40% of the $10,000,000 of joint costs, or
44% and Betters 56%. Thus, $50,400 (56% x $4,000,000.
$90,000) should be allocated to Betters.
Answer (C) is incorrect because $2,286,000 is
based on the relative sales value of units sold.
[309] Source: CMA 1293 3-6
Answer (D) is incorrect because $2,500,000 is
Answer (A) is incorrect because $36,000 is based based on the physical quantity of barrels sold.
on 40%, not 4/9.

Answer (B) is correct. The gross market value of [312] Source: CIA 1194 III-47
Alfa is $40,000 ($4 x 10,000 pounds), Betters has a
total gross value of $50,000 ($10 x 5,000 pounds), Answer (A) is incorrect because $375,000 is the
and Morefeed has a split-off value of $3,000. If the total cost of R.
value of Morefeed is inventoried and treated as a
reduction in joint cost, the allocable joint cost is Answer (B) is incorrect because $390,000 is based
$90,000 ($93,000 - $3,000). The total gross value on the physical units method of allocating the joint
of the two main products is $90,000 ($40,000 + costs.
$50,000). Of this total value, $40,000 should be
Answer (C) is correct. Total sales value at split-off is
allocated to Alfa [($40,000 ・$90,000) x $90,000]. $800,000 [(2,500 x $100) + (5,000 x $80) + (7,500
x $20)]. Product S accounts for 50% (5,000 x $80 =
Answer (C) is incorrect because $41,333 fails to $400,000) of the sales value and therefore $360,000
adjust the joint cost by the value of the by-product. (50% x $720,000) of the joint costs. The total cost
of Product S is $510,000 ($360,000 allocated costs
Answer (D) is incorrect because $50,000 is the joint + $150,000 differential costs).
cost allocated to Betters.
Answer (D) is incorrect because $571,463 uses the
sales value at split-off based on actual sales.
[310] Source: CMA 1295 3-29
assumes that the by-product is charged with a portion
[313] Source: CIA 1194 III-48 of the gross joint cost.

Answer (A) is correct. The net realizable value Answer (C) is correct. The joint cost to be allocated
(NRV) method is an appropriate method of is $2,400,000 [$2,520,000 total joint cost - ($2 x
allocation when products cannot be sold at split-off. 60,000 pounds of the by-product)]. Accordingly, the
Further processing of R, which is salable at split-off, joint cost to be allocated to the Second Main
is not economical because the cost ($150,000) Product on a physical-volume basis is $1,500,000
exceeds the benefit [($150 - $100) x 2,500 units = {$2,400,000 x [150,000 pounds ・(90,000 pounds
$125,000]. Thus, R's NRV is $250,000 ($100 price + 150,000 pounds)]}.
at split-off x 2,500 units). However, S and T must
be Answer (D) is incorrect because $1,575,000 does
processed further. S's NRV is $425,000 [($115 x not deduct by-product NRV from the joint cost.
5,000 units) - $150,000], and T's NRV is $125,000
[($30 x 7,500 units) - $100,000]. Given that the
NRV of T is a reduction of joint cost, the total joint [316] Source: Publisher
cost to be allocated is therefore $595,000
($720,000 Answer (A) is correct. Total joint production costs
- $125,000 NRV of T). Accordingly, based on the incurred were $9,000,000 ($4,000,000 +
NRV method, the joint cost allocated to R is $2,000,000 + $3,000,000). The total physical output
$220,370 {[$250,000 R's NRV ・($250,000 R's was 660,000 barrels (300,000 barrels of Grade One
NRV + $425,000 S's NRV)] x $595,000 allocable + 240,000 barrels of Grade Two + 120,000 barrels
joint cost}. Because further processing of R is of Grade Three). Thus, on a physical output basis,
uneconomical, the total cost of R is $220,370. Grade Two should be allocated $3,273,000
[$9,000,000 x (240,000 ・660,000)].
Answer (B) is incorrect because $370,370 includes
additional processing costs. Answer (B) is incorrect because $3,375,000 is
based on the physical quantity of units sold, not units
Answer (C) is incorrect because $374,630 is the produced.
joint cost allocated to S.
Answer (C) is incorrect because $1,636,000 is the
Answer (D) is incorrect because $595,000 is the amount assigned to Grade Three.
allocable joint cost.
Answer (D) is incorrect because $3,512,000 is the
amount assigned to Grade Two if the relative sales
[314] Source: CMA 1293 3-8 value method is used.

Answer (A) is correct. Joint costs are most often


assigned on the basis of relative sales values or net [317] Source: Publisher
realizable values. Basing allocations on physical
quantities, such as pounds, gallons, etc., is usually Answer (A) is incorrect because $3,512,000 is the
not amount assigned to Grade Two.
desirable because the costs assigned may have no
relationship to value. When large items have low Answer (B) is correct. Total joint production costs
selling prices and small items have high selling incurred were $9,000,000, ($4,000,000 +
prices, $2,000,000 + $3,000,000). The sales values of the
the large items might always sell at a loss when three products are as follows:
physical quantities are used to allocate joint costs.
Grade One (300,000 x $30) $ 9,000,000
Answer (B) is incorrect because physical quantities Grade Two (240,000 x $40) 9,600,000
are usually easy to measure. Grade Three (120,000 x $50) 6,000,000
------------
Answer (C) is incorrect because additional Total sales value $ 24,600,000
processing costs will have no more effect on the ============
allocation of joint costs based on physical Consequently, Grade One should be assigned joint
quantities costs of $3,293,000 [$9,000,000 x ($9,000,000 ・
than any other base. $24,600,000)].

Answer (D) is incorrect because the purpose of Answer (C) is incorrect because $1,636,000 is
allocating joint costs, under any method, is to based on the relative sales values of units sold, not
separate such costs on a unit basis. units produced.

Answer (D) is incorrect because $4,091,000 is


[315] Source: CMA 1296 3-30 based on the physical quantity of barrels produced.

Answer (A) is incorrect because $1,200,000


assumes that the by-product is charged with a [318] Source: Publisher
portion
of the net joint cost. Answer (A) is incorrect because $3,512,000 is the
total joint cost assigned to the output of Grade Two.
Answer (B) is incorrect because $1,260,000
Answer (B) is correct. The total sales value of the cost, respectively, that for the next year result from a
oil 10% increase in the appropriation of the current year
produced was determined in the preceding instead of a 10% decrease in the next year's
question appropriation.
to be $24,600,000. The total sales value of Grade
Two was determined to be $9,600,000. Answer (C) is correct. This question applies CVP
Accordingly, costs assigned to Grade Two on a analysis in a not-for-profit context in which the
relative sales value basis (rounded) equal agency wishes to assist as many people as possible.
$3,512,000 Thus, a breakeven point must be calculated. Total
[($4,000,000 + $3,000,000 + $2,000,000) x revenue (the appropriation) equals fixed cost plus the
($9,600,000 ・ $24,600,000)]. Thus, the value of product of unit variable cost (per-patient annual cost)
the and the number of patients who can be assisted given
ending inventory of Grade Two should be the available resources. The following are
$1,756,000 [($3,512,000 ・240,000 barrels simultaneous equations stated in the two unknowns:
produced) x 120,000 barrels in EI].
X - 5,000 Y = $5,000,000
Answer (C) is incorrect because $1,636,000 is .9X - 4,000 Y = $5,000,000
based on the relative sales values of units sold. Because X must equal 5,000Y + $5,000,000, the
second equation may be solved as follows for the
Answer (D) is incorrect because $3,375,000 is the per-patient annual cost (Y):
total joint cost assigned to the output of Grade Two
based on the relative physical volume of units sold. .9(5,000 Y + $5,000,000) - 4,000Y =
$5,000,000
4,500y + $4,500,000 - 4,000Y =
[319] Source: Publisher $5,000,000
500Y = $500,000
Answer (A) is correct. The absorption-costing Y = $1,000
breakeven point in units sold equals the sum of (1) Accordingly, the budgeted appropriation (X) must be
the total fixed costs and (2) the product of the fixed $10,000,000 [(5,000 x $1,000) VC + $5,000,000
manufacturing cost application rate and the FC], and the reduced appropriation must be
difference $9,000,000 (90% x $10,000,000).
between the BEP in units sold (X) and units
produced, with the sum divided by the UCM. Thus, Answer (D) is incorrect because $10,000,000 is the
the absorption-costing breakeven point in units sold budgeted appropriation and 5,000 is the number of
is people it would serve.
425,000 units:

($2 x 1,000,000 denominator [321] Source: Publisher


capacity)
+ $1,500,000 + $2(X - Answer (A) is incorrect because $.05 results from
900,000) inverting the numerator and denominator in the
X = ------------------------------------- calculation.
$6
$3,500,000 + $2X - $1,800,000 Answer (B) is correct. The breakeven point in units is
X = ----------------------------- equal to the fixed costs divided by the contribution
$6 margin per unit. Thus, 44,000 = $880,000 ・CM, or
$6X = $1,700,000 + $2X CM = $20.
X = 425,000
Answer (C) is incorrect because $44.00 results from
Answer (B) is incorrect because 583,333 units is the using variable cost as part of the calculation.
variable-costing BEP.
Answer (D) is incorrect because $88.00 results from
Answer (C) is incorrect because 900,000 units is dividing by an erroneous denominator.
the
actual production level.
[322] Source: Publisher
Answer (D) is incorrect because 1,000,000 units is
the capacity used to calculate the fixed overhead Answer (A) is incorrect because 250 results from
application rate. using an erroneous contribution margin.

Answer (B) is correct. The breakeven point is where


[320] Source: Publisher profit is zero and sales = fixed costs + variable costs,
so 10x = 4,000 + 2x. Thus, 8x = 4,000, or x = 500
Answer (A) is incorrect because $5,000,000 is the units. Alternatively, dividing the $4,000 of fixed costs
fixed cost and 4,000 is the number served given a by the $8 per unit contribution margin gives the same
reduced appropriation. result.

Answer (B) is incorrect because $8,333,333 and Answer (C) is incorrect because 800 results from
$833 are the appropriation amount per patient using the inverse of the contribution margin.
annual
Answer (D) is incorrect because 2,000 results from
applying the 20% to fixed costs instead of sales.

[326] Source: CMA 1286 4-19


[323] Source: Publisher
Answer (A) is incorrect because $800,000 equals
Answer (A) is incorrect because 5,514 results from prime costs.
adding the profit margin to the contribution margin.
Answer (B) is incorrect because $900,000 equals
Answer (B) is incorrect because 9,273 is the inventoriable costs under variable costing.
breakeven point.
Answer (C) is correct. The absorption method is
Answer (C) is incorrect because 13,600 results from required for financial statements prepared according
using profit as the contribution margin. to GAAP. It charges all costs of production to
inventories. The prime costs ($800,000), variable
Answer (D) is correct. Sales = Fixed Costs + manufacturing overhead ($100,000), and the fixed
Variable Costs + Profit. The profit is the selling price manufacturing overhead ($160,000) are included.
of the good multiplied by the percent profit that is They total $1,060,000.
desired/stated. Thus,
Answer (D) is incorrect because $1,060,000
$15x = $51,000 + $9.50x + .25($15x), or includes the fixed and variable selling and other
$15x = $51,000 + $13.25x, or expenses.
$1.75x = $51,000, or
x = 29,143 bears
A trial-and-error approach could also be used to [327] Source: Publisher
solve this problem by preparing an income
statement Answer (A) is correct. The contribution margin
for each of the four alternative production levels. equals sales minus variable costs. Direct costing
considers only variable costs as product costs, so the
contribution margin appears in a direct costing
[324] Source: Publisher income statement. Absorption costing treats both
variable and fixed costs as product costs. Thus,
Answer (A) is incorrect because .67 is the inverse of variable costs are not stated separately, and the
the degree of operating leverage. contribution margin would not appear in the income
statement. Accordingly, the contribution margin for
Answer (B) is incorrect because .75 results from the whole company is $480,000 ($1,000,000 net
adding fixed costs in the denominator. revenue - $400,000 variable CGS - $120,000
variable selling and administrative costs), and the
Answer (C) is correct. The degree of operating contribution margin for Division 1 is $120,000
leverage (DOL) can be calculated from the formula ($300,000 net revenues - $140,000 variable CGS -
[Q(P - V)] ・[Q(P - V) - F], if Q is the number of $40,000 variable selling and administrative costs).
units sold, P is the unit selling price, V is the unit Thus, the ratio of the contribution margin of Division
variable cost, and F is the fixed cost. Thus, (4,000 x 1 to the contribution margin of the whole company is
75) ・[(4,000 x 75) - 100,000], or $300,000 ・ 0.25.
200,000 = 1.5.
Answer (B) is incorrect because 0.2083 is the ratio
Answer (D) is incorrect because 3.0 results from of the contribution margin of Division 2 to the
failing to use contribution margin in the contribution margin of the whole company.
denominator.
Answer (C) is incorrect because it is the ratio of
division net income to total net income.
[325] Source: CMA 1286 4-18
Answer (D) is incorrect because it is the ratio of
Answer (A) is incorrect because $800,000 equals division net income to total net income, with the
the prime costs. exception of president's salary.

Answer (B) is correct. The only costs capitalized are


the variable costs of manufacturing. Prime costs [328] Source: Publisher
(direct materials and direct labor) are variable.
Answer (A) is incorrect because the contribution of
Prime costs, direct materials, Division 1 is $50,000, which is less than $105,000.
and direct labor $800,000
Variable manufacturing overhead 100,000 Answer (B) is incorrect because the contribution of
-------- Division 2 is $50,000, which is less than $105,000.
Total inventoriable costs $900,000
======== Answer (C) is correct. The contribution margin for
Division 3 is $150,000 ($250,000 net revenue -
Answer (C) is incorrect because $980,000 includes $100,000 total variable costs). The contribution
the variable selling and other expenses. controllable by Division 3's manager is $130,000
($150,000 contribution margin - $20,000
Answer (D) is incorrect because $1,060,000 equals controllable fixed cost). The total contribution by
inventoriable costs under absorption costing. Division 3 equals its net revenue minus all costs
traceable to it. Accordingly, the total contribution is
$105,000 ($130,000 controllable contribution - Answer (A) is incorrect because $157,200 results
$25,000 allocated but controllable by others). from a failure to add in the first 5 months' revenue.
Unallocated costs are excluded from the
calculation. Answer (B) is incorrect because $160,800 is the
If separate amounts are determined for the result of multiplying operating profit by .4 instead of
division's .6.
contribution and the controllable contribution, the
difference between the division's and the Answer (C) is correct. The after-tax profit of a selling
manager's price alternative is found by initially finding the year's
performance may be ascertained (assuming combined revenue. Therefore, the first 5 months'
controllability of fixed costs can be assigned). revenue of $140,000 ($400 x 350) is added to the
final 7 months' revenue of $972,000 ($360 x 2,700)
Answer (D) is incorrect because the contribution of to get a yearly revenue of $1,112,000. From this
Division 4 is $65,000, which is less than $105,000. number, the variable costs of $610,000 ($200 x
3,050) and the fixed costs of $100,000 are
subtracted to get an operating profit of $402,000.
[329] Source: Publisher The 40% tax rate is taken out of this amount by
multiplying the $402,000 by .6 (1 - .4) to get an
Answer (A) is incorrect because 167 units is the after-tax profit of $241,200.
result of adding variable cost to the sales price.
Answer (D) is incorrect because $301,200 results
Answer (B) is incorrect because 250 units results from a failure to subtract fixed costs from the
from a failure to subtract the variable cost from the revenue.
sales price.

Answer (C) is correct. At breakeven, the profit is [332] Source: Publisher


zero, therefore sales must be equal to fixed cost
and Answer (A) is incorrect because $1,327,700 results
variable cost. In this case, the formula is from a failure to include fringe benefit expense.

$400x = $200x + $100,000 Answer (B) is incorrect because $1,484,480 results


$200x = $100,000 from a failure to include overtime wages expense.
x = 500 units
Answer (C) is correct. In this problem, the fixed
Answer (D) is incorrect because 1,700 units results expenses incurred are from advertising, rent, property
from adding the after-tax profit objective to the insurance, utilities, malpractice insurance,
fixed depreciation, and wages and fringe benefits. The first
cost. step is to add all of these expenses except for wages
and fringe benefits. Therefore, you must add
$500,000 from advertising, rent is $168,000 ($28 x
[330] Source: Publisher 6,000), $22,000 for property insurance, $32,000 for
utilities, $180,000 for malpractice insurance, and
Answer (A) is incorrect because 1,250 units results depreciation is $15,000 ($60,000 ・4). This total is
from a failure to subtract variable costs from the $917,000. The next step is to determine regular
sales wages of $403,200 [($25 + $20 + $15 + $10) x 16
price. hours x 360 days] plus overtime wages of $7,500
[(200 x $15 x 1.5) + (200 x $10 x 1.5)] to get total
Answer (B) is incorrect because 1,700 units results wages of $410,700. This number multiplied by 40%
from using the after-tax profit, not the pretax profit gives a fringe benefit expense of $164,280.
objective. Therefore, total fixed expenses is equal to
$1,491,980 ($917,000 + $410,700 + $164,280).
Answer (C) is incorrect because 2,000 units results
from a failure to include fixed cost in the equation. Answer (D) is incorrect because $1,536,980 results
from depreciating all of the office equipment in the
Answer (D) is correct. The number of units to be first year.
sold for Almo to achieve its after-tax profit objective
of $240,000 is the point where revenue equals total
costs plus the pretax profit objective. The pretax [333] Source: Publisher
profit objective is found by dividing the after-tax
profit objective by (1 - tax rate) or .6. Hence, the Answer (A) is incorrect because 9,947 clients results
formula to find the desired point is from a failure to subtract variable costs.

Answer (B) is correct. In a breakeven analysis, the


$400x = $200x + $100,000 + ($240,000 ・.6) profit is equal to zero and sales must be equal to the
$400x = $200x + $100,000 + $400,000 sum of fixed costs and variable costs. In the analysis,
$200x = $500,000 there are two forms of revenue, $30 for the initial
x = 2,500 units consultation, and the inflow from favorable
settlements. Therefore, the formula is

[331] Source: Publisher $30x + ($2,000 x .2x x .3) = $4x + $1,491,980


$30x + $120x = $4x + $1,491,980 per hour) to get manufacturing costs of $650,600.
$146x = $1,491,980 The application rates for direct labor and variable
x = $10,219 clients overhead are found by dividing $150,000 and
$155,000 by 25,000 hours, respectively. Therefore,
Answer (C) is incorrect because 12,862 clients the cost of goods available for sale is equal to
results from a failure to add the $30 from the initial $713,950 ($16,950 + $46,400 + $650,600).
consultation.
Answer (D) is incorrect because $716,600 results
Answer (D) is incorrect because 57,384 clients from a failure to reduce the beginning balances by the
results from a failure to add the inflow from the fixed overhead.
favorable settlements.

[336] Source: Publisher


[334] Source: Publisher
Answer (A) is incorrect because $635,950 results
Answer (A) is incorrect because 47.5 results from from a failure to decrease the inventory balances by
averaging the possible outcomes without respect to the fixed overhead.
its probability.
Answer (B) is correct. The variable manufacturing
Answer (B) is correct. The expected value in a cost of goods sold is found by subtracting the ending
probability distribution is found by adding the balances of work-in-process and finished goods
values inventories from cost of goods available for sale. The
created by multiplying each possible outcome by ending balances of the inventories are found by
its subtracting fixed overhead from the absorption
respective probability. Therefore, the expected inventories. Therefore, the work-in-process ending
value inventory is found by subtracting $2,100 (2,100
is (20 x .10) + (30 x .30) + (55 x .40) + (85 x .20), hours x $1) from $64,000 to get $61,900. The
or 2 + 9 + 22 + 17, giving an expected value of 50 ending finished goods balance is found by subtracting
clients per day. $820 (820 hours x $1) from $14,000 to get
$13,180. Thus, the variable manufacturing cost of
Answer (C) is incorrect because 52.5 results from goods sold is equal to $638,870 ($713,950 -
adding the high and low outcomes and dividing by $61,900 - $13,180).
2.
Answer (C) is incorrect because $652,050 results
Answer (D) is incorrect because 55 is the most from a failure to subtract the finished goods inventory.
probable outcome, not the expected value.
Answer (D) is incorrect because $700,770 results
from a failure to subtract the work-in-process
[335] Source: Publisher inventory.

Answer (A) is incorrect because $667,550 results


from a failure to add the beginning balance of the [337] Source: Publisher
work-in-process inventory.
Answer (A) is incorrect because $281,130 is the
Answer (B) is incorrect because $675,600 is the result of subtracting the entire selling expense.
cost
of goods available for sale under absorption Answer (B) is correct. The contribution margin is
costing. found by subtracting total variable costs from sales
revenue. In this example, variable costs are equal to
Answer (C) is correct. The cost of goods available variable cost of goods sold plus variable selling
for sale under the variable costing method is found expenses. Since variable cost of goods sold has been
by found to be $638,870, the only issue remaining is
adding the beginning balances of finished goods selling expenses. Because the variable portion of
and selling expenses is 5% of sales revenue, variable
work-in-process inventories to the manufacturing selling expense is equal to $50,750 ($1,015,000 x
costs incurred. To find the beginning balances of .05). Therefore, the contribution margin is equal to
the $325,380 ($1,015,000 - $638,870 - $50,750).
variable inventories, fixed overhead must be
subtracted from the absorption inventory. The fixed Answer (C) is incorrect because $331,880 is the
overhead is applied at $1 per direct labor hour result of subtracting the fixed portion of selling
($25,000 of fixed overhead ・25,000 direct labor expense.
hours). Therefore, the variable inventory balance
for Answer (D) is incorrect because $363,130 is the
finished goods is $16,950 [$18,000 - ($1,050 hours result of subtracting the increase in selling expense
x $1)], and the variable inventory balance for this year.
work-in-process is $46,400 [$48,000 - (1,600
hours x $1)]. The manufacturing costs incurred are
found by adding the cost of direct materials, [338] Source: Publisher
$370,000, plus the cost of direct labor, $138,000
(23,000 hours x $6 per hour), plus the cost of Answer (A) is correct. The variable operating income
variable overhead, $142,600 (23,000 hours x $6.20 for a corporation is found by subtracting total fixed
costs from the contribution margin. In this example, Answer (A) is incorrect because it uses the variable
total fixed costs are comprised of factory overhead, cost percentage in the denominator instead of the
selling expenses, and administrative costs. With contribution margin percentage.
fixed
selling expenses being $44,250 ($95,000 - Answer (B) is incorrect because it uses the same
$50,750), total fixed costs are equal to $156,650 contribution margin percentage (20%) in both
($37,400 + $44,250 + $75,000). Therefore, with a calculations.
contribution margin of $325,380, variable operating
income is equal to $168,730 ($325,380 - Answer (C) is incorrect because it reverses the two
$156,650). contribution margin percentages.

Answer (B) is incorrect because $206,130 results Answer (D) is correct. The original breakeven level
from a failure to subtract fixed factory overhead was:
from
the contribution margin. $800,000
-------- = $2,500,000
Answer (C) is incorrect because $212,980 results .32
from a failure to subtract fixed selling expenses The new level is:
from
the contribution margin. $700,000
-------- = $3,500,000
Answer (D) is incorrect because $243,730 results
from a failure to subtract fixed administrative .20
expenses from the contribution margin. Thus, there is an increase of $1,000,000.

[339] Source: CMA Samp Q3-3 [341] Source: Publisher

Answer (A) is incorrect because unit sales Answer (A) is correct. The $4 million breakeven
multiplied level was calculated as follows:
by the sales price equals sales revenue, but this
amount does not necessarily correlate with $1,200,000
operating ---------- = $4,000,000
income. A change in unit variable costs may cause CM%
revenue and operating income to move in different Solving for CM% results in CM% = 30%. Thus,
directions. 30% of every dollar, or $0.30, contributes towards
profits.
Answer (B) is incorrect because operating income is
not necessarily correlated positively or negatively Answer (B) is incorrect because, when substituted
with into the original breakeven formula, they would not
finished goods inventory, however valued. have produced a breakeven level of $4 million.

Answer (C) is correct. In a variable costing system, Answer (C) is incorrect because, when substituted
only the variable costs are recorded as product into the original breakeven formula, they would not
costs. have produced a breakeven level of $4 million.
All fixed costs are expensed in the period incurred.
Because changes in the relationship between Answer (D) is incorrect because, when substituted
production levels and sales levels do not cause into the original breakeven formula, they would not
changes in the amount of fixed manufacturing cost have produced a breakeven level of $4 million.
expensed, profits more directly follow the trends in
sales, especially when the UCM (selling price per
unit [342] Source: Publisher
- variable costs per unit) is constant. Unit sales
times Answer (A) is correct. Assume that sales were
the UCM equals the total CM, and operating income previously $1,000, with profits of $150 (15% x
(a pretax amount) equals the CM minus fixed costs $1,000). If sales increase to $1,250, profits will
of increase to $187.50 (15% x $1,250). The $37.50
operations. If the UCM is constant and fixed costs increase is 25% of the previous $150.
are stable, the change in operating income will
approximate the change in the CM (UCM x unit Answer (B) is incorrect because profit margin
sales). increases by 25%.

Answer (D) is incorrect because operating income Answer (C) is incorrect because profit grows at the
is same rate as sales.
not necessarily correlated positively or negatively
with Answer (D) is incorrect because 15% of sales
finished goods inventory, however valued. translates to net income.

[340] Source: Publisher [343] Source: Publisher


Answer (A) is incorrect because a progressive tax is have been recognized under the straight-line method.
a tax in which individuals with higher (lower)
incomes Answer (C) is incorrect because the effect is to
pay a higher (lower) percentage of their income in increase gains and decrease losses.
tax. For example, income taxes are progressive.
Answer (D) is incorrect because the effect is to
Answer (B) is correct. With a regressive tax, the increase gains and decrease losses.
percentage paid in taxes decreases as income
increases. For example, excise taxes and payroll
taxes are both regressive taxes. An excise tax is [346] Source: Publisher
regressive because its burden falls
disproportionally Answer (A) is incorrect because $17,700 is the gross
on lower-income persons. As personal income tax liability.
increases, the percentage of income paid declines
because an excise tax is a flat amount per quality Answer (B) is incorrect because $15,300 is the net
of tax liability found when incorrectly treating the tax
the good or service purchased. credit as a direct reduction in taxable income.

Answer (C) is incorrect because a proportional tax Answer (C) is correct. The first step in calculating
is HCC's net tax liability is to subtract the exclusion for
a tax in which the individual pays a constant tax-exempt interest from income, for a gross income
percentage in taxes, regardless of income level. A amount of $69,000 ($80,000 - $11,000). Next, the
sales tax is a proportional tax. deprecation deduction is subtracted from gross
income, for a taxable income of $59,000 ($69,000 -
Answer (D) is incorrect because a progressive tax is $10,000). Then, the taxable income is multiplied by
a tax in which individuals with higher (lower) the tax rate, for a gross tax liability of $17,700
incomes ($59,000 x 30%). Finally, net tax liability is
pay a higher (lower) percentage of their income in computed by subtracting the tax credits from the
tax. For example, income taxes are progressive. gross tax liability. Therefore, HCC's net tax liability is
$9,700 ($17,700 - $8,000).

[344] Source: CMA 0686 1-20 Answer (D) is incorrect because $2,000 is the net tax
liability found when incorrectly treating the exclusion
Answer (A) is incorrect because personal income as a credit.
taxes and Social Security taxes levied against the
employee are direct taxes.
[347] Source: Publisher
Answer (B) is incorrect because personal income
taxes and Social Security taxes levied against the Answer (A) is incorrect because an exclusion or
employee are direct taxes. deduction reduces gross tax liability by the amount of
the exclusion or deduction multiplied by the tax rate.
Answer (C) is correct. Indirect taxes are those Therefore, the exclusion will reduce HCC's gross tax
levied liability by $3,300 ($11,000 x .30), and the
against someone other than individual taxpayers deduction will reduce HCC's gross tax liability by
and $3,000 ($10,000 x .30).
thus only indirectly affect the individual. Sales
taxes Answer (B) is incorrect because an exclusion or
are levied against businesses and are then passed deduction reduces gross tax liability by the amount of
along to the individual purchaser. Social Security the exclusion or deduction multiplied by the tax rate.
taxes are levied against both the employer and the Therefore, the exclusion will reduce HCC's gross tax
employee. Those levied against the employee are liability by $3,300 ($11,000 x .30), and the
direct taxes; those levied against the employer are deduction will reduce HCC's gross tax liability by
indirect. $3,000 ($10,000 x .30).

Answer (D) is incorrect because personal income Answer (C) is incorrect because an exclusion or
taxes and Social Security taxes levied against the deduction reduces gross tax liability by the amount of
employee are direct taxes. the exclusion or deduction multiplied by the tax rate.
Therefore, the exclusion will reduce HCC's gross tax
liability by $3,300 ($11,000 x .30), and the
[345] Source: CMA 0690 4-27 deduction will reduce HCC's gross tax liability by
$3,000 ($10,000 x .30).
Answer (A) is incorrect because the effect is to
increase gains and decrease losses. Answer (D) is correct. Credits directly reduce taxes,
whereas exclusions and deductions reduce income
Answer (B) is correct. An accelerated method prior to the computation of the gross tax liability.
reduces the book value of the asset more rapidly in Thus, the credit reduces the gross tax liability on a
the early years of the useful life than does the dollar-for-dollar basis, or $8,000. An exclusion or
straight-line method. Hence, the effect of an early deduction reduces gross tax liability by the amount of
sale the exclusion or deduction multiplied by the tax rate.
is to increase the gain or decrease the loss that Accordingly, the exclusion will reduce HCC's gross
would tax liability by $3,300 ($11,000 x 30%), and the
deduction will reduce HCC's gross tax liability by on private activity bonds issued after August 7, 1986.
$3,000 ($10,000 x 30%). Gross income does not A sales commission accrued in the current year but
reduce the gross tax liability; rather, it increases paid in the following year is not an example of an
the AMT adjustment.
gross tax liability by $24,000 ($80,000 x 30%).

[350] Source: CMA 1294 1-29


[348] Source: CMA 1291 2-11
Answer (A) is correct. An increase in the corporate
Answer (A) is incorrect because warranty expenses income tax rate might encourage a company to
are not deductible until paid. borrow because interest on debt is tax deductible,
whereas dividends are not. Accordingly, an increase
Answer (B) is incorrect because dividends on in the tax rate means that the after-tax cost of debt
common stock are never deductible by a capital will decrease. Given equal interest rates, a firm
corporation; they are distributions of after-tax with a high tax rate will have a lower after-tax cost of
income. debt capital than a firm with a low tax rate.

Answer (C) is incorrect because amounts accrued Answer (B) is incorrect because increased
by uncertainty encourages equity financing. Dividends do
an accrual-basis taxpayer to be paid to a related not have to be paid in bad years, but interest on debt
cash-basis taxpayer in a subsequent period are not is a fixed charge.
deductible until the latter taxpayer includes the
items Answer (C) is incorrect because an increase in
in income. This rule effectively puts related interest rates discourages debt financing.
taxpayers
on the cash basis. Answer (D) is incorrect because an increase in the
price-earnings ratio means that the return to
Answer (D) is correct. Sec. 162(a) states that a shareholders (equity investors) is declining; therefore,
deduction is allowed for the ordinary and necessary equity capital is a more attractive financing alternative.
expenses incurred during the year in any trade or
business. A corporation may therefore deduct a
reasonable amount for compensation. Accrued [351] Source: Publisher
vacation pay is a form of compensation that results
in Answer (A) is incorrect because a reorganization that
an allowable deduction for federal income tax is a mere change in the form of investment is
purposes. nontaxable.

Answer (B) is incorrect because a like-kind exchange


[349] Source: CMA 1291 2-12 allows for the deferral of gain.

Answer (A) is incorrect because the gain on an Answer (C) is correct. Like-kind exchanges,
installment sale of real property in excess of involuntary conversions, and tax-free reorganizations
$150,000 is an adjustment to taxable income for are examples of transactions that result in the deferral
purposes of computing alternative minimum or nonrecognition of gain. A reorganization is
taxable nontaxable when it is considered a mere change in
income. investment, not a disposition of assets.

Answer (B) is incorrect because mining exploration Answer (D) is incorrect because an involuntary
and development costs are adjustments to taxable conversion allows for the deferral of gain.
income for purposes of computing alternative
minimum taxable income.
[352] Source: CMA 0689 4-9
Answer (C) is incorrect because a charitable
contribution of appreciated property is an Answer (A) is incorrect because the unit contribution
adjustment margin ratio is the unit contribution margin divided by
to taxable income for purposes of computing the unit selling price.
alternative minimum taxable income.
Answer (B) is correct. The $44.00 unit cost includes
Answer (D) is correct. Taxable income is adjusted to fixed overhead as well as variable costs. Thus, it is
arrive at alternative minimum taxable income. determined in accordance with full absorption
Some costing. The difference between selling price and the
of the common adjustments include gains or losses full absorption (inventory) cost is gross profit (gross
from long-term contracts, gains on installment margin).
sales of
real property, mining exploration and development Answer (C) is incorrect because the unit contribution
costs, charitable contributions of appreciated margin is the difference between unit selling price and
property, accelerated depreciation, the unit variable cost.
accumulated
current earnings adjustment, and tax-exempt Answer (D) is incorrect because the unit gross profit
interest margin ratio is the unit gross profit divided by the unit
selling price.
Answer (C) is incorrect because committed costs
[353] Source: CMA 0689 4-10 result when a going concern holds fixed assets.
Examples are insurance, long-term lease payments,
Answer (A) is incorrect because carrying cost is the and depreciation.
cost of holding inventory, e.g., insurance expense,
storage costs, and the opportunity cost of the Answer (D) is incorrect because mixed costs are
investment. those that include both variable and fixed elements.

Answer (B) is incorrect because a sunk


(unavoidable) cost is irrelevant to a decision [356] Source: CMA 0689 4-13
because
it has been or must be incurred. Answer (A) is incorrect because sunk costs are fixed
or variable costs that are unavoidable, for example,
Answer (C) is correct. A mixed (semivariable) cost because they have already been incurred.
has fixed and variable components. An example is
the Answer (B) is incorrect because discretionary costs
electric utility charge. It includes a minimum (fixed are ordinarily not inventoriable. Examples of
cost) and a variable amount for usage above a discretionary costs include advertising and R&D
certain expenditures.
level.
Answer (C) is correct. Committed costs result when
Answer (D) is incorrect because committed costs a going concern holds fixed assets. Examples are
result when a going concern holds fixed assets. insurance, long-term lease payments, and
Examples are insurance, long-term lease payments, depreciation. Committed costs tend not to be
and depreciation. variable in the short-run because they are the results
of capital budgeting decisions, e.g., the construction
of a plant and the purchase and installation of
[354] Source: CMA 0689 4-11 equipment. For this reason, they are classified as
fixed overhead.
Answer (A) is incorrect because conversion costs
are Answer (D) is incorrect because prime costs are the
incurred to transform raw materials into a finished variable costs of direct materials and direct labor.
product. They include direct labor and factory
overhead.
[357] Source: CMA 1290 3-3
Answer (B) is correct. The R&D costs have already
been incurred. Thus, they are costs resulting from a Answer (A) is incorrect because the direct method
past irrevocable decision. These sunk costs are does not make allocations to other service
irrelevant to the decision because they are departments.
unavoidable.
Answer (B) is incorrect because the term variable
Answer (C) is incorrect because relevant costs are method is nonsensical.
those that must be weighed in making decisions
because they will vary among the options Answer (C) is incorrect because the reciprocal
considered. method recognizes reciprocal interdepartmental
service.
Answer (D) is incorrect because opportunity costs
are the returns from the alternative uses of Answer (D) is correct. The three major methods of
resources. allocating service department costs, in order of
These returns are forgone by using resources in a increasing sophistication, are the direct method, the
specific way. step-down method, and the reciprocal (or
simultaneous-equations) method. The direct method
is the simplest. It involves allocating all service
[355] Source: CMA 0689 4-12 department costs to production departments without
recognizing any service provided by one service
Answer (A) is correct. Advertising costs are department to another. The step-down method is a
discretionary costs. Unlike engineered costs (such sequential process that allocates service costs among
as service as well as production departments. However,
direct materials), a specific, measurable once a department's costs have been allocated, no
relationship additional allocations are made back to that
between a discretionary cost and its output is not department. The reciprocal method uses simultaneous
possible. Other examples are training costs, public equations to recognize mutual services. The latter
relations, and R&D. method is the most complex.

Answer (B) is incorrect because opportunity costs


are the returns from the alternative uses of [358] Source: CMA 1291 3-5
resources.
These returns are forgone when resources are used Answer (A) is incorrect because ROI does not have
in to be maximized under the residual income approach.
a specific way.
Answer (B) is incorrect because maximizing the transferred from the molding department are simply
imputed interest rate charge would diminish the the total units transferred from the molding
residual return. department--42,000 units.

Answer (C) is incorrect because the residual income


method is based on net income rather than cash [361] Source: CMA 0692 3-3
flows.
Answer (A) is incorrect because 30,000 units ignores
Answer (D) is correct. Residual income is the excess the 8,000 units in process at the beginning of the
of the return on an investment over the targeted period.
amount. This amount may be defined as the
imputed Answer (B) is correct. Direct materials are added
interest on invested capital. Some firms prefer to when the units are 60% complete as to conversion
measure managerial performance in terms of the costs. The beginning inventory of 8,000 units was
amount of residual income rather than the only 25% complete at the start of the period, and
percentage 42,000 units were transferred in. Given that the
ROI. The principle is that the firm is expected to ending inventory of 12,000 units was only 40%
benefit from expansion as long as residual income complete, neither beginning nor ending inventory had
is received direct materials in the assembly department.
earned. Using a percentage ROI approach, Accordingly, the equivalent units in the assembly
expansion might be rejected if it lowered ROI even department for direct materials must have been
though residual income would increase. 38,000 units (8,000 units BI + 42,000 units
transferred in - 12,000 units EI).

[359] Source: CMA 1291 3-6 Answer (C) is incorrect because 40,800 equals the
equivalent units for conversion costs, not direct
Answer (A) is correct. The purpose of the residual materials.
income approach is to maximize income in excess
of Answer (D) is incorrect because the 42,000 units
an imputed interest charge on invested capital. were transferred in during the month. Not all received
Thus, an input of direct materials.
any returns in excess of the cost of capital are
beneficial to the firm. The cost of capital is a
weighted [362] Source: CMA 0692 3-4
average of the various debt and equity components
of Answer (A) is incorrect because 34,800 units
the capital structure. assumes the beginning inventory was 100%
complete.
Answer (B) is incorrect because to use only the cost
of new capital would ignore the contribution of old Answer (B) is correct. The equivalent units for
capital to a department. The purpose of the conversion costs equal total units to account for,
residual minus work done on beginning inventory, minus work
income approach is to maximize income in relation not done on ending inventory. Hence, the equivalent
to units for conversion costs equal 40,800 units [50,000
capital employed. units - (25% x 8,000 units) - (60% x 12,000 units)].

Answer (C) is incorrect because use of the average Answer (C) is incorrect because 42,800 units is the
ROI might result in rejection of projects with returns answer derived using the weighted-average
in excess of the cost of capital. assumption.

Answer (D) is incorrect because the objective is to Answer (D) is incorrect because the ending inventory
maximize income in relation to costs; the was only 40% complete (not 60%), resulting in
comparison equivalent units for those 12,000 items of 4,800 units.
is not to other years or averages.

[363] Source: CMA 1277 5-5


[360] Source: CMA 0692 3-2
Answer (A) is incorrect because the difference in
Answer (A) is incorrect because 30,000 units were total costs that results from selecting one alternative
started and completed during the period. instead of another is an incremental cost.

Answer (B) is incorrect because 38,000 units were Answer (B) is incorrect because a cost that cannot
be avoided because it has already been incurred is a
transferred out, not to, the assembly department. sunk cost.

Answer (C) is incorrect because 40,800 equals the Answer (C) is correct. An imputed cost does not
equivalent units for conversion costs. entail any dollar outlay but is relevant to the
decision-making process.
Answer (D) is correct. This problem seemingly asks
a technical question, but in reality was designed to Answer (D) is incorrect because a cost that continues
test the candidate's alertness. The equivalent units to be incurred even though there is no activity is a
fixed cost. Answer (A) is incorrect because costs incurred in a
current period to achieve objectives other than the
filling of orders by customers are known as
[364] Source: CMA 0678 4-6 discretionary costs.

Answer (A) is incorrect because manufacturing Answer (B) is incorrect because costs likely to
costs respond to the amount of attention devoted to them
incurred to produce units of output are by a specified manager are controllable costs.
inventoriable
(product) costs. Answer (C) is correct. Committed costs are required
as a result of past decisions. They are the fixed costs
Answer (B) is incorrect because all costs associated of plant, equipment, and other items that are basic to
with manufacturing other than direct labor costs the entity. Because they result from long-term
and decisions about capacity, they are unlikely to change
raw materials costs are overhead costs. Conversion in the short run.
costs consist of both direct labor and overhead.
Answer (D) is incorrect because costs that fluctuate
Answer (C) is correct. Conversion costs are the with small changes in volume are variable costs.
direct labor, indirect materials, and factory
overhead
incurred to convert raw materials and transferred-in [368] Source: CMA 0678 4-11
goods in a cost center to finished goods.
Answer (A) is correct. Discretionary costs are fixed
Answer (D) is incorrect because raw materials costs in the short-run because they arise from periodic
and direct labor costs are prime costs. (e.g., annual) appropriation decisions by management
about the total to be incurred. Discretionary costs
have no well-defined input-output relationship; that is,
[365] Source: CMA 0678 4-7 they are only indirectly related to filling customer
orders. Advertising and research are examples.
Answer (A) is incorrect because all costs associated
with manufacturing other than direct labor costs Answer (B) is incorrect because costs likely to
and respond to the amount of attention devoted to them
raw materials costs are overhead costs. by a specified manager are controllable costs.

Answer (B) is incorrect because predetermined Answer (C) is incorrect because costs required as a
costs result of past decisions are committed costs.
are standard costs.
Answer (D) is incorrect because costs unaffected by
Answer (C) is incorrect because the sum of direct managerial decisions are costs such as committed
labor and overhead is conversion cost. costs and depreciation that were determined by
decisions of previous periods.
Answer (D) is correct. Prime costs are raw materials
costs and direct labor costs.
[369] Source: CMA 0678 4-12

[366] Source: CMA 0678 4-9 Answer (A) is incorrect because costs incurred in a
current period to achieve objectives other than the
Answer (A) is incorrect because marketing, filling of orders by customers are discretionary costs.
shipping,
etc., are selling and administrative costs. Answer (B) is correct. Controllable costs are directly
regulated by management at a given level of
Answer (B) is incorrect because costs that do not production within a given time span. For example,
change in total for a given period and relevant fixed costs are not controllable.
range
are fixed costs. Fixed costs also become Answer (C) is incorrect because costs required as a
progressively smaller on a per unit basis as volume result of past decisions are committed costs.
increases.
Answer (D) is incorrect because costs unaffected by
Answer (C) is incorrect because manufacturing managerial decisions are costs such as committed
costs costs and depreciation that were determined by
incurred to produce output are inventoriable decisions of previous periods.
(product) costs.

Answer (D) is correct. Variable costs fluctuate [370] Source: CMA 0680 4-5
directly and proportionately with sales or
production Answer (A) is incorrect because normal spoilage
volume, facility usage, or other activity measure. costs end up in inventory, not abnormal spoilage.

Answer (B) is incorrect because material variance


[367] Source: CMA 0678 4-10 accounts are only charged for the variances in
material usage or material price, not the spoilage of
product. Answer (D) is correct. A cost incurred for the benefit
of more than one cost objective is known as a
Answer (C) is incorrect because, while charging common cost. Allocation of common costs is a
abnormal spoilage to manufacturing overhead is an
occasional practice, it is not the ordinary practice, persistent problem in responsibility accounting. For
which is to charge it to a special loss account. example, how should the costs of corporate
headquarters be allocated to the segments of a
Answer (D) is correct. Abnormal spoilage is usually conglomerate? Common cost is also a synonym for
charged to a special loss account because it is not joint cost. In this sense, common costs are incurred in
expected to occur under normal, efficient operating the production of two or more inseparable products
conditions. Because it is unusual, it should be (e.g., costs of refining petroleum into gasoline, diesel
separately reported as a period cost. fuel, kerosene, lubricating oils, etc.) up to the point at
which the products become separable (the split-off
point).
[371] Source: CMA 1282 4-101

Answer (A) is incorrect because property taxes and [374] Source: CMA 1285 4-25
interest charges are independent of production
levels. Answer (A) is incorrect because total direct labor
cost is calculated as follows: total manufacturing cost
Answer (B) is incorrect because the president's of $2,500,000 equals raw materials, plus direct
salary usually does not vary with production levels. labor, plus factory overhead. Factory overhead is
They are therefore fixed costs and are treated as 30% of total manufacturing costs, or $750,000. If
elements of overhead. factory overhead is 80% of direct labor, direct labor
cost is $937,500 ($750,000 ・80%).
Answer (C) is incorrect because property taxes and
interest charges are independent of production Answer (B) is incorrect because total direct labor
levels. cost is calculated as follows: total manufacturing cost
of $2,500,000 equals raw materials, plus direct
Answer (D) is correct. Variable costs vary directly labor, plus factory overhead. Factory overhead is
with the level of production. As production 30% of total manufacturing costs, or $750,000. If
increases factory overhead is 80% of direct labor, direct labor
or decreases, materials costs increase or decrease, cost is $937,500 ($750,000 ・80%).
usually in direct proportion to production, sales, or
other activity measure. Answer (C) is incorrect because total direct labor
cost is calculated as follows: total manufacturing cost
of $2,500,000 equals raw materials, plus direct
[372] Source: CMA 0685 5-1 labor, plus factory overhead. Factory overhead is
30% of total manufacturing costs, or $750,000. If
Answer (A) is incorrect because some overhead factory overhead is 80% of direct labor, direct labor
costs are variable but cannot be directly traced to a cost is $937,500 ($750,000 ・80%).
particular product.
Answer (D) is correct. Total manufacturing cost of
Answer (B) is incorrect because it includes costs $2,500,000 is composed of raw materials, direct
that labor, and factory overhead. Factory overhead is
cannot be directly traced. 30% of total manufacturing costs, or $750,000. If
factory overhead is 80% of direct labor cost, direct
Answer (C) is incorrect because it includes costs labor cost is $937,500 ($750,000 ・80%).
that
cannot be directly traced.
[375] Source: CMA 1285 4-26
Answer (D) is correct. Prime costs are direct
materials and direct labor. They are directly Answer (A) is incorrect because $750,000 is factory
identifiable elements of production costs and are overhead (30% x $2,500,000).
directly traceable to the product.
Answer (B) is correct. Factory overhead is 30% of
total manufacturing costs, or $750,000. Direct labor
[373] Source: CMA 0685 5-6 is $937,500. Thus, raw materials must account for
the remaining $812,500 ($2,500,000 - $750,000 -
Answer (A) is incorrect because a variable cost is $937,500).
one that varies directly with production activity.
Answer (C) is incorrect because direct material is
Answer (B) is incorrect because conversion cost is $812,500 ($2,500,000 manufacturing costs -
the cost of labor and overhead incurred to convert $750,000 factory overhead - $937,500 direct labor
raw materials into a finished product. costs).

Answer (C) is incorrect because prime costs are the


costs of materials and labor that are directly Answer (D) is incorrect because direct material is
traceable $812,500 ($2,500,000 manufacturing costs -
to a cost objective. $750,000 factory overhead - $937,500 direct labor
costs).
parts used, the more handling is involved.

[376] Source: CMA 1285 4-27


[378] Source: CIA 1193 IV-1
Answer (A) is correct. Cost of goods manufactured
($2,425,000) equals total manufacturing costs Answer (A) is correct. The cost of small tools used in
($2,500,000) plus beginning work-in-process (75% mounting tires cannot be identified solely with the
of EWIP) minus ending work-in-process. The ending manufacture of a specific automobile. This cost
work-in-process is $300,000. should be treated as factory overhead because it is
identifiable with the production process.
$2,500,000 + .75 EWIP - EWIP = $2,425,000
$2,500,000 - .25 EWIP = $2,425,000 Answer (B) is incorrect because tire costs are readily
EWIP = $75,000 ・.25 and directly identifiable with each automobile and,
EWIP = $300,000 thus, are direct materials costs.

Answer (B) is incorrect because the carrying value Answer (C) is incorrect because the cost of the
of laborers who place tires on each automobile is readily
the work-in-process inventory is calculated as and directly identifiable with each automobile. Hence,
follows: costs of goods manufactured ($2,425,000) it is a direct labor cost.
equals total manufacturing costs ($2,500,000), plus
beginning work-in-process (75% of EWIP), minus Answer (D) is incorrect because delivery costs are
ending work-in-process. The ending readily and directly identifiable with the tires
work-in-process is $300,000. delivered. Thus, they are direct materials costs.

Answer (C) is incorrect because the carrying value


of [379] Source: CIA 1193 IV-4
the work-in-process inventory is calculated as
follows: costs of goods manufactured ($2,425,000) Answer (A) is incorrect because $20,000 only
equals total manufacturing costs ($2,500,000), plus includes other factory overhead. Indirect materials
beginning work-in-process (75% of EWIP), minus and indirect labor should also be included.
ending work-in-process. The ending
work-in-process is $300,000. Answer (B) is incorrect because $25,000 excludes
the indirect materials.
Answer (D) is incorrect because the carrying value
of Answer (C) is correct. Factory (manufacturing)
the work-in-process inventory is calculated as overhead consists of all costs, other than direct
follows: costs of goods manufactured ($2,425,000) materials and direct labor, that are associated with
equals total manufacturing costs ($2,500,000), plus the manufacturing process. It includes both fixed and
beginning work-in-process (75% of EWIP), minus variable costs. The factory overhead control account
ending work-in-process. The ending should have the following costs:
work-in-process is $300,000.
Indirect materials $ 5,000
Indirect labor ($45,000-$40,000) 5,000
[377] Source: CIA 0593 IV-3 Other factory overhead 20,000
-------
Answer (A) is incorrect because direct labor hours is Total overhead $30,000
a traditional base for assigning overhead costs to =======
production. However, it is not necessarily an
appropriate basis for assigning overhead costs Answer (D) is incorrect because $45,000 is the total
because direct labor is a small percentage of the labor cost.
total
cost of most products.

Answer (B) is incorrect because number of units [380] Source: CIA 1193 IV-5
produced is an output related measure. Materials
handling costs should be related to an input Answer (A) is incorrect because the straight-time
measure. wages times the overtime hours should still be treated
as direct labor.
Answer (C) is incorrect because the number of
vendors might be appropriate for receiving and Answer (B) is incorrect because only the overtime
inspection, but materials handling in this situation premium times the overtime hours is charged to
encompasses more than costs related to the overhead.
number
of vendors. Answer (C) is incorrect because labor costs are not
related to repairs and maintenance expense.
Answer (D) is correct. Cost drivers should be
related Answer (D) is correct. Direct labor costs are wages
to the costs accumulated in cost pools. The number paid to labor that can feasibly be specifically
of parts used has a direct cause-and-effect identified with the production of finished goods.
relationship with materials handling costs. The Factory overhead consists of all costs, other than
more direct materials and direct labor, that are associated
with the manufacturing process. Thus, straight-time characteristics and/or when identifiable groupings are
wages would be treated as direct labor; however, possible. Process costing should be used to assign
because the overtime premium cost is a cost that costs to similar products that are mass produced on a
should be borne by all production, the overtime continuous basis. Operations costing is a hybrid of
hours job order and process costing systems. It is used by
times the overtime premium should be charged to companies that manufacture goods that undergo
manufacturing overhead. some similar and some dissimilar processes. Thus,
job order costing would be appropriate for auto
repair, operations costing for clothing manufacturing,
[381] Source: CIA 1193 IV-3 and process costing for oil refining.

Answer (A) is incorrect because the direct Answer (B) is incorrect because custom printing
allocation would not use process costing.
method ignores any services that are rendered by
one Answer (C) is incorrect because paint manufacturing
service department to another service department. would not use operations costing.

Answer (B) is incorrect because the step-down Answer (D) is incorrect because motion picture
allocation method allows for limited recognition of production would not use process costing.
services rendered by service departments to other
service departments.
[384] Source: Publisher
Answer (C) is incorrect because the linear allocation
method is nonsensical. Answer (A) is correct. The correct entry to record a
purchase of materials on account is to increase the
Answer (D) is correct. The reciprocal method uses appropriate asset and liability accounts. Materials are
simultaneous equations to allocate each service charged to an inventory; the corresponding liability is
department's costs. It allocates costs by explicitly accounts payable. The asset account(s) could be
including the mutual services rendered among all stores control and/or supplies or a number of other
departments. When service departments render accounts. Also, subsidiary ledgers may be used to
services to each other, the use of the direct method account for various individual items (a perpetual
or the step-down method would not be theoretically inventory system). The term control implies that a
accurate. Accordingly, in such situations, the subsidiary ledger is being used.
reciprocal method would result in the most
accurate Answer (B) is incorrect because the entry to record
allocation. the return of materials to suppliers debits accounts
payable and credits raw materials inventory.

[382] Source: CIA 0592 IV-6 Answer (C) is incorrect because this entry reclassifies
credit balances in accounts receivable as liabilities or
Answer (A) is incorrect because a quantity variance debit balances in accounts payable as assets.
is not recorded for scrap that is anticipated.
Furthermore, work-in-process inventory is credited Answer (D) is incorrect because this entry would
only when scrap is unique to a job. record the purchase of materials for cash.

Answer (B) is incorrect because an accounting


entry [385] Source: Publisher
is not needed. The amount is not material.
Answer (A) is incorrect because overhead must be
Answer (C) is correct. Making a memorandum entry budgeted before a rate can be calculated.
at the time of recovery is appropriate. The value of
the scrap is then recognized at the time of sale. Answer (B) is correct. Annual overhead application
The rates smooth seasonal variability of overhead costs
factory overhead control account is credited and activity levels. If overhead were applied to the
because product as incurred, the overhead rate per unit in
scrap is inevitable to the company's production most cases would vary considerably from week to
operations and not attributable to a specific job. week or month to month. The purpose of an annual
This overhead application rate is to simulate constant
accounting method has the effect of spreading the overhead throughout the year.
revenue from scrap sales over all jobs or products.
Answer (C) is incorrect because overhead
Answer (D) is incorrect because normal scrap is not application rates are used to smooth seasonal
the basis for recording a variance. variability of overhead costs.

Answer (D) is incorrect because an overhead rate


[383] Source: CIA 0594 III-70 applies overhead to the product.

Answer (A) is correct. Job order costing is


appropriate when producing products with [386] Source: Publisher
individual
Answer (A) is correct. The overhead application rate
is established at the beginning of each year to should minimize expected over- or underapplied
determine how much overhead to accumulate for overhead.
each job throughout the period. The estimated
annual Answer (C) is incorrect because normal volume is an
overhead costs are divided by the annual activity average expected volume over a series of years. It
level will vary from the expected volume on a
or capacity in terms of units to arrive at the desired year-by-year basis.
rate.
Answer (D) is incorrect because practical capacity is
Answer (B) is incorrect because actual overhead is theoretical capacity adjusted downward for holidays,
not known at the beginning of the period; the maintenance time, etc. It is very difficult to attain.
overhead rate is predetermined.

Answer (C) is incorrect because the estimated [389] Source: Publisher


activity level is the rate's denominator.
Answer (A) is correct. The larger the denominator in
Answer (D) is incorrect because the actual activity the overhead application rate, the smaller the rate and
level is not known until year-end. Also, activity is a the lower the cost assigned to the product.
denominator value. Theoretical capacity, which is the absolute capacity
during continuous operations, ignoring holidays,
maintenance time, etc., provides the largest
[387] Source: Publisher denominator in the ratio.

Answer (A) is incorrect because direct labor hours is Answer (B) is incorrect because normal volume is
appropriate when overhead is incurred uniformly by less than theoretical capacity.
all types of employees.
Answer (C) is incorrect because practical capacity is
Answer (B) is incorrect because direct materials theoretical capacity adjusted downward for holidays,
cost maintenance time, etc.
would be inappropriate for a labor intensive
industry. Answer (D) is incorrect because minimum volume
relates to the lowest production level that would be
Answer (C) is incorrect because machine hours is required to operate a particular function and would
an result in the largest amount of applied overhead.
appropriate activity base when overhead varies
with
machine time used. [390] Source: CIA 1185 IV-10

Answer (D) is correct. In labor intensive industries, Answer (A) is incorrect because cost of goods sold
overhead is usually allocated based on a labor should be credited (not debited) for its share of
activity overapplied overhead.
base. If more overhead is incurred by the more
highly Answer (B) is incorrect because cost of goods sold,
skilled and paid employees, the overhead rate finished goods inventory, and work-in-process
should inventory should be credited (not debited).
be based upon direct labor cost rather than direct
labor hours. Answer (C) is incorrect because, although commonly
used, the immediate write-off method is not as
conceptually sound as the allocation among cost of
[388] Source: Publisher goods sold, finished goods inventory, and
work-in-process inventory.
Answer (A) is incorrect because theoretical
(maximum or ideal) capacity is the absolute Answer (D) is correct. Under a normal costing
capacity system, overhead is applied to all jobs worked on
assuming continuous operations, i.e., on Sundays, during the period at a predetermined rate. Because
holidays, etc., and can never be attained. cost of goods sold, finished goods inventory, and
work-in-process inventory all relate to these jobs,
Answer (B) is correct. Overhead is applied each should be adjusted by its proportionate share of
according to a rate found by dividing budgeted over- or underapplied overhead. This apportionment
overhead for a period by an estimated activity may be based on either the percentage of total
level. overhead (theoretically preferable) or the percentage
If actual activity differs from the denominator value of total cost. The entry to close overapplied overhead
(the predetermined activity level), a volume requires credits to these three accounts.
variance
will occur. This variance equals the amount of over-
or underapplied overhead attributable solely to the [391] Source: Publisher
difference between budgeted and actual activity.
The Answer (A) is incorrect because a debit to cost of
expected volume is that predicted for the period. goods sold and a credit to finished goods expenses
Thus, the use of expected volume as a inventoried costs related to items sold.
denominator
Answer (B) is incorrect because the entry to close Answer (D) is incorrect because, under FIFO, the
the overhead accounts credits CGS when overhead goods that were started last period and completed
has been overapplied. this period are deemed to be completed first and
transferred first.
Answer (C) is incorrect because debiting CGS and
crediting overhead applied does not close the
overhead accounts. [393] Source: CMA 0694 3-6

Answer (D) is correct. Although not theoretically Answer (A) is incorrect because fixed indirect
sound, total under- or overapplied overhead is manufacturing costs are included in the calculation of
often gross margin.
debited (credited) to CGS. The correct entry to
close Answer (B) is incorrect because fixed costs are also
the overhead accounts and to charge underapplied included in the calculation of gross margin.
overhead to CGS is to debit the factory overhead
applied account for the amount of overhead Answer (C) is correct. Gross margin or gross profit is
applied the excess of sales over cost of goods sold,
for the period and to credit factory overhead calculated on a full absorption basis. Cost of goods
control sold would include all manufacturing costs, both fixed
for the amount of overhead actually incurred for and variable.
the
period. The amount actually incurred exceeds the Answer (D) is incorrect because fixed costs are also
amount of overhead applied because overhead is included in the calculation of gross margin.
underapplied. The difference is the amount charged
to CGS.

[392] Source: Publisher [395] Source: CMA 0690 4-2

Answer (A) is incorrect because, if the cost of goods Answer (A) is correct. The sum of direct materials
started last period and completed this period and used, direct labor, and factory overhead applied
the (60% of direct labor) is $681,000.
cost of goods started and completed this period are
kept separate, separate layers will continue to Answer (B) is incorrect because $665,000 equals the
multiply as the units of product are passed through cost of goods manufactured.
additional WIP accounts. Thus, these costs are
combined before transfer to the next department. Answer (C) is incorrect because $489,000 equals
the direct materials purchased plus direct labor.
Answer (B) is incorrect because, if the cost of goods
started last period and completed this period and Answer (D) is incorrect because $673,000 equals
the the cost of goods sold.
cost of goods started and completed this period are
kept separate, separate layers will continue to
multiply as the units of product are passed through [396] Source: CMA 0690 4-3
additional WIP accounts. Thus, these costs are
combined before transfer to the next department. Answer (A) is correct. The cost of the goods
manufactured is the cost of goods completed during
Answer (C) is correct. Under FIFO, goods started the year. For a retailer, the equivalent is purchases.
last period and completed this period are The CGM for Alex is $665,000 [501,000 + (60% x
differentiated from goods started and completed $300,000) + $235,000 - $251,000].
this
period. The goods started last period but Answer (B) is incorrect because $681,000 equals
completed total manufacturing costs.
this period include the costs from last period as
well Answer (C) is incorrect because $673,000 is the
as this period's costs to complete, whereas goods cost of goods sold.
started and completed this period only include
current Answer (D) is incorrect because $657,000 results
costs. In the weighted-average method, the costs from adding ending inventory and subtracting
of beginning inventory in the CGS calculation.
the prior and current periods are averaged.

When the goods are transferred to the next [397] Source: CMA 0690 4-4
department or to finished goods under FIFO,
however, they are considered transferred out at Answer (A) is incorrect because $697,000 equals
one prime cost, plus overhead applied, plus (instead of
average cost so that a multitude of layers of minus) the change in the work-in-process.
inventory
is not created. This procedure is consistent with the Answer (B) is incorrect because $681,000 equals
basic concept of process costing. prime cost plus overhead applied.
Answer (C) is correct. The calculation of the cost of allocation. However, allocation encourages cost
goods sold requires the preparation of a partial control by the production departments. If the costs
income statement: are allocated, managers have an incentive not to use
services indiscriminately.
Beginning finished goods inventory $125,000
Plus cost of goods manufactured 665,000 Answer (B) is incorrect because allocation does not
-------- affect the coordination of production activity.
Goods available for sale $790,000
Minus ending finished goods inventory (117,000) Answer (C) is correct. Service department costs are
-------- indirect costs allocated to production departments to
Cost of goods sold $673,000 better determine overhead rates when the
======== measurement of full (absorption) costs is desired.
Overhead should be charged to production on some
Answer (D) is incorrect because $657,000 results equitable basis to provide information useful for such
from adding ending inventory and subtracting purposes as allocation of resources, pricing,
beginning inventory in the CGS calculation. measurement of profits, and cost reimbursement.

Answer (D) is incorrect because allocation of costs


[398] Source: CMA 0690 4-5 has no effect on the efficiency of the provision of
services when the department that receives the
Answer (A) is incorrect because overapplied allocation has no control over the costs being
overhead results in a credit to overhead control. controlled.

Answer (B) is incorrect because overhead is


overapplied. [401] Source: CMA 0693 3-1

Answer (C) is incorrect because overhead is Answer (A) is incorrect because cost allocation
overapplied. permits a company to determine the profitability of a
department and to make decisions relative to
Answer (D) is correct. The factory overhead control expanding or contracting its operations.
account should have a debit of $175,000 for the
actual costs incurred and a credit for the $180,000 Answer (B) is correct. According to SMA 2A,
(60% of direct labor) applied to production. Thus, allocation of costs is a distribution of costs that
the net effect is a $5,000 credit balance resulting cannot be directly assigned to the cost objects that
from are assumed to have caused them. The process
the overapplication of overhead. entails choosing a cost object, determining the direct
and indirect costs that should be traced to the cost
object, deciding how costs are to be aggregated
[399] Source: CMA 0690 4-10 (accumulated in cost pools) prior to allocation, and
selecting the allocation base. Cost allocation is
Answer (A) is incorrect because $50,000 is necessary for, among other things, product costing,
abnormal spoilage. pricing, investment and disinvestment decisions,
managerial performance measurement, make-or-buy
Answer (B) is incorrect because $20,000 equals decisions, and determination of profitability.
normal spoilage for the month. However, an allocation of costs does not enable a
company to determine why the sales of a particular
Answer (C) is incorrect because $70,000 is the sum product have increased. Many factors affect
of normal spoilage and abnormal spoilage. consumer demand, such as advertising, consumer
confidence, availability of substitutes, and changes in
Answer (D) is correct. Normal spoilage is an tastes. Cost allocation is an internal matter that does
inventoriable cost of production that is charged to not affect demand except to the extent it results in a
cost of goods sold when the units are sold. change in price.
Abnormal
spoilage is a period cost recognized when incurred. Answer (C) is incorrect because cost allocation
The $50,000 of abnormal spoilage is therefore permits a company to determine the profitability of a
expensed during May. In addition, 50% of the product line and to decide whether to discontinue that
normal line.
spoilage is debited to cost of goods sold because
50% (25,000 ・50,000) of the units completed were Answer (D) is incorrect because make-or-buy
sold during the period. No spoilage is allocated to decisions depend on cost analyses.
work-in-process because inspection occurs after
completion. Thus, the normal spoilage expensed
during the month is $10,000 (50% x $20,000). Total [402] Source: CMA 0693 3-4
spoilage charged against revenue is $60,000
($50,000 + $10,000). Answer (A) is incorrect because a cost accountant's
salary cannot be directly associated with a single
product. Cost accountants work with many different
[400] Source: CMA 1290 3-2 products during a pay period.

Answer (A) is incorrect because costs can be Answer (B) is incorrect because warehouse rent is
controlled by the service departments without not directly traceable to the Purchasing Department.
Other departments have influence over the level of Answer (C) is incorrect because a sunk cost is the
inventories stored. result of a past irrevocable action; it is not important
to future decisions.
Answer (C) is correct. A direct cost is one that can
be specifically associated with a single cost Answer (D) is incorrect because a target cost is the
objective maximum allowable cost of a product and is
in an economically feasible way. Thus, a production calculated before the product is designed or
supervisor's salary can be directly associated with produced.
the
department (s)he supervises.
[405] Source: CIA 0593 IV-6
Answer (D) is incorrect because directors' fees
cannot be directly associated with the Marketing Answer (A) is incorrect because assigning spoilage
Department. Directors provide benefits to all costs to finished goods is an appropriate method of
departments within a corporation. accounting for normal spoilage traceable to a job or
process.

[403] Source: CMA 1293 3-2 Answer (B) is incorrect because allocating spoilage
costs between finished goods and work-in-process is
an appropriate method of accounting for normal
Answer (A) is correct. Service department costs spoilage traceable to a job or process, provided the
should be allocated to the production departments units in process have passed the inspection point.
that use the services. When service departments
also Answer (C) is incorrect because charging spoilage
render services to each other, their costs are costs to manufacturing overhead is an appropriate
usually method of accounting for normal spoilage, assuming
allocated to each other before allocation to the allowance for normal spoilage is incorporated into
production departments. A basis reflecting cause the predetermined overhead rate.
and
effect should be used to allocate service Answer (D) is correct. Abnormal spoilage should be
department written-off to a special account that is separately
costs. Because of interactions among service reported in the income statement. Costs associated
departments, the best allocation method is the with abnormal spoilage are not inventoried and are
reciprocal (simultaneous-equations) method. It is therefore treated as a loss in the period of detection.
superior to the direct method because the latter
allocates no service costs to other service
departments. It is also superior to the step (or [406] Source: CIA 1194 III-46
step-down) method because this method allocates
no Answer (A) is incorrect because waste is input
service cost back to service departments, the costs material that is either lost in the production process or
of has no sales value.
which have been allocated in a preceding step.
Answer (B) is incorrect because scrap is input
Answer (B) is incorrect because the step method is material that has a relatively minor sales value at the
less sophisticated than the reciprocal method. end of the production process.

Answer (C) is incorrect because the direct method Answer (C) is correct. Rejected units that are
allocates service costs directly without discarded are classified as spoilage. Spoilage is
consideration separated into abnormal or normal spoilage. Normal
of interactions with other service departments; the spoilage is an inherent result of the normal production
direct method is the least sophisticated method. process. Abnormal spoilage is spoilage that is not
expected to occur under normal, efficient operating
Answer (D) is incorrect because accretion method conditions.
is
a nonsense term. Answer (D) is incorrect because rework costs are
incurred to make unacceptable units appropriate for
sale or use.
[404] Source: CMA 1295 3-27

Answer (A) is correct. A cost that bears an [407] Source: CIA 1193 IV-6
observable and known relationship to a quantifiable
activity base is known as an engineered cost. Answer (A) is incorrect because factory overhead
Engineered costs have a clear relationship to should be charged for direct materials, supplies,
output. direct labor, and applied overhead incurred for
Direct materials would be an example of an rework.
engineered cost.
Answer (B) is incorrect because factory overhead
Answer (B) is incorrect because an indirect cost should be charged for direct materials, supplies,
does direct labor, and applied overhead incurred for
not have a clear relationship to output. rework.
Answer (C) is incorrect because $19,300 excludes establishes standards and accountability, it motivates
the predetermined manufacturing overhead. good performance by highlighting the work of
effective managers. Moreover, the nature of the
Answer (D) is correct. The rework charge for direct budgeting process fosters communication of goals to
materials, indirect materials (supplies), direct labor, company subunits and coordination of their efforts.
and overhead applied on the basis of direct labor Budgeting activities by entities within the company
cost is $40,300 [$5,000 + $300 + $14,000 + (1.5 x must be coordinated because they are
$14,000)]. If an allowance for rework is included in interdependent. Thus, the sales budget is a necessary
a input to the formulation of the production budget. In
company's manufacturing overhead budget, rework turn, production requirements must be known before
of defective units is spread over all jobs or batches purchases and expense budgets can be developed,
as and all other budgets must be completed before
part of the predetermined overhead application preparation of the cash budget.
rate.
Hence, the debit is to overhead control. Answer (D) is incorrect because responsibility
centers are determined prior to budgeting, budgets
do not fix blame but rather measure performance,
[408] Source: CIA 1196 III-93 and goal congruence is promoted but not ensured by
budgets.
Answer (A) is incorrect because $1,440 (360 x
$4.00) ignores the manufacturing overhead.
[410] Source: CIA 1193 IV-27
Answer (B) is correct. Normal spoilage equals 1,140
units (4% x 28,500 good units), so abnormal Answer (A) is incorrect because a production budget
spoilage does not include revenues.
equals 360 units (1,500 total spoiled units - 1,140
units of normal spoilage). Given that .25 DLH is Answer (B) is incorrect because a cash budget does
needed to rework a spoiled unit, the loss from not include non-cash items.
abnormal spoilage is $4,140 {360 units x [(.25 x
$16) direct labor + (.25 x $30) manufacturing Answer (C) is incorrect because a capital budget
overhead]}. does not include revenues and costs.

Answer (C) is incorrect because $3,450 [300 x Answer (D) is correct. A flexible budget includes
($4.00 + $7.50)] uses the wrong amount for different cost levels for different levels of activity, for
abnormal spoilage. example, sales. Hence, it is actually a series of
budgets. Thus, a budget based on the behavior of
Answer (D) is incorrect because a loss should be costs and revenues over a range of sales is a flexible
charged for abnormal spoilage. Total spoilage budget.
exceeded the 4% normal rate.

[411] Source: CIA 1196 III-80


[409] Source: CIA 1194 III-54
Answer (A) is incorrect because $7,500 unfavorable
Answer (A) is incorrect because responsibility equals the difference between actual and budgeted
centers are determined prior to budgeting, budgets direct materials, multiplied by the standard price per
do not fix blame but rather measure performance, pound of direct materials.
and goal congruence is promoted but not ensured
by Answer (B) is incorrect because zero equals the
budgets. difference between the budgeted direct materials
purchases at standard cost and the actual cost of the
Answer (B) is incorrect because responsibility direct materials.
centers are determined prior to budgeting, budgets
do not fix blame but rather measure performance, Answer (C) is correct. The direct materials price
and goal congruence is promoted but not ensured variance measures the difference between what was
by actually paid for the goods purchased and the
budgets. standard price allowed for the goods purchased.
Thus, it equals the difference between actual price
Answer (C) is correct. A budget is a realistic plan for and standard price, multiplied by the actual quantity
the future expressed in quantitative terms. The purchased. The direct materials price variance is
process of budgeting forces a company to establish $5,000 favorable {[$2.50 - ($120,000 ・50,000
goals, determine the resources necessary to units)] x 50,000 units}.
achieve
those goals, and anticipate future difficulties in Answer (D) is incorrect because $5,100 unfavorable
their equals the difference between the standard and actual
achievement. A budget is also a control tool costs of direct materials per pound, multiplied by the
because actual pounds used instead of the actual pounds
it establishes standards and facilitates comparison purchased.
of
actual and budgeted performance. Because a
budget [412] Source: CIA 1196 III-81
Answer (A) is correct. The direct materials compared over time among different factories or with
efficiency benchmarks. A partial productivity measure
variance measures the difference between the comparing results over time determines whether the
actual actual relationship between inputs and outputs has
use of inputs and the budgeted quantity of inputs improved or deteriorated.
allowed for the activity level achieved. The direct
materials efficiency variance equals the standard Answer (D) is incorrect because scrap is neither an
unit input nor a good output.
price times the difference between inputs actually
used and standard inputs. The direct materials
efficiency variance is $500 favorable {[(32,000 x [415] Source: CIA 1192 IV-17
1.60) - 51,000] x $2.50}.
Answer (A) is incorrect because Product A does
Answer (B) is incorrect because $3,000 favorable have the greatest contribution margin ratio (53%), but
uses the amount of direct materials purchased when resources are limited, maximum profits are
instead achieved by maximizing dollar contribution margin per
of the actual amount used. limited or constraining factor, which in this case is
machine hours.
Answer (C) is incorrect because $7,500 unfavorable
equals the difference between the standard amount Answer (B) is correct. When resources are limited,
allowed for the budgeted units to be produced and maximum profits are achieved by maximizing dollar
the amount of direct materials used, multiplied by contribution margin per limited or constraining factor.
the In this situation, machine hours are the constraining
standard price per pound. factor. Product B has a contribution margin per
machine hour of $28 [4 x ($18 - $11)], which is
Answer (D) is incorrect because $8,000 unfavorable greater than that of Product A [3 x ($15 - $7) =
equals the difference between the budgeted $24], Product C [2 x ($20 - $10) = $20], or
finished Product D [3 x ($25 - $16) = $27].
units and the actual finished units, multiplied by
the Answer (C) is incorrect because Product C does
standard cost of a finished unit. have the greatest dollar unit contribution margin
($10), but when resources are limited, maximum
profits are achieved by maximizing dollar contribution
[413] Source: CIA 1195 III-84 margin per limited or constraining factor, which in this
case is machine hours.
Answer (A) is correct. Before redesign, productivity
equaled 4 units per hour (2,000 units ・500 hours). Answer (D) is incorrect because Product D does
After redesign, productivity equaled 4.2 units per have the greatest selling price per unit ($25), but
hour (2,520 units ・ 600 hours). Thus, the when resources are limited, maximum profits are
percentage achieved by maximizing dollar contribution margin per
change in productivity was 5% [(4.2 - 4.0) ・4.0]. limited or constraining factor, which in this case is
machine hours.
Answer (B) is incorrect because a 10% change
requires output of 2,640 units.
[416] Source: CIA 0592 IV-18
Answer (C) is incorrect because 20% is the
percentage change in labor hours per day. Answer (A) is incorrect because $100 F ($48,500 -
$48,600) is the direct labor price (rate) variance.
Answer (D) is incorrect because 26% is the
percentage change in units of output per day. Answer (B) is correct. The total flexible budget direct
labor variance equals the difference between cost at
actual hours and actual wages and the cost at
[414] Source: CIA 0596 III-90 standard hours and standard wages, or $1,900 U
($48,500 - $46,600).
Answer (A) is incorrect because inventory shrinkage
measures the effectiveness of internal control. Answer (C) is incorrect because the flexible budget
direct labor variance is unfavorable, not favorable.
Answer (B) is incorrect because inventory turnover Total actual cost exceeds the total flexible budget
measures the efficiency of asset usage. amount.

Answer (C) is correct. A partial productivity Answer (D) is incorrect because $2,000 U ($48,600
measure - $46,600) is the direct labor efficiency (usage)
may be stated as the ratio of output to the quantity variance.
of
a single factor of production (e.g., materials, labor,
or [417] Source: CIA 1195 III-96
capital). Partial productivity measures, for example,
the number of finished units per direct labor hour Answer (A) is incorrect because a transfer at full cost
or means that the selling division will not make a profit.
per pound of direct materials, are useful when In addition, the selling division may be forgoing profits
that could be obtained by selling to outside
customers. Thus, full-cost transfer prices can lead
to
suboptimal decisions. [419] Source: CIA 0595 III-95

Answer (B) is incorrect because a transfer at full Answer (A) is correct. Segment reporting is an
cost aspect of responsibility accounting. It facilitates
plus markup results in no incentive for the selling evaluation of company management and of the quality
division to control its costs. Hence, a sustained of the economic investment in particular segments.
level
of management effort may not be maintained. Answer (B) is incorrect because interdependence of
segments is not affected by reporting methods.
Answer (C) is incorrect because a transfer at
variable Answer (C) is incorrect because masking the effects
cost plus markup has the same weaknesses as full of intersegment transfers is a disadvantage of segment
cost plus markup. reporting.

Answer (D) is correct. A market price transfer price Answer (D) is incorrect because providing
promotes goal congruence and a sustained level of information to competitors is a disadvantage of
management effort. It is also consistent with segment reporting.
divisional
autonomy. A market transfer price is most
appropriate when the market is competitive, [420] Source: CIA 1196 III-97
interdivisional dependency is low, and buying in the
market involves no marginal costs or benefits. Answer (A) is incorrect because rent is not
controllable at the product-line level.

[418] Source: CIA 1196 III-94 Answer (B) is incorrect because advertising is not
controllable at the product-line level.
Answer (A) is correct. The outlay costs represent
cash outflows related to the production and Answer (C) is correct. Commissions, cost of sales,
transfer and salaries are all traceable to each of the product
of goods/services. The opportunity costs are the lines and are therefore controllable. Administrative
maximum contribution forgone by the supplying expenses are common or corporate-level expenses,
division if the goods/services are sold internally. An advertising does not promote a specific product line,
opportunity cost will exist if the supplier has no idle and rent is paid on a building shared by all product
capacity and an external market exists. Thus, this lines.
guideline should promote goal congruence (actions
of Answer (D) is incorrect because administration,
the divisional manager benefit the company and advertising, and rent are not controllable at the
the product-line level.
division), a sustained high level of managerial effort
(exertion toward a goal), and subunit autonomy
(freedom in decision making). The guideline will
vary
depending on whether an external market exists [422] Source: CIA 1196 III-99
and
whether the supplier has idle capacity. Answer (A) is incorrect because an increase of
$50,000 assumes the revenue will be lost and all of
Answer (B) is incorrect because the opportunity its costs will be avoided.
cost
of the buying division is irrelevant. Answer (B) is incorrect because a decrease of
$94,000 results from treating rent as an avoidable
Answer (C) is incorrect because the full cost may cost.
not
represent the outlay cost incurred to the point of Answer (C) is correct. The operating income will
transfer, and a markup is an arbitrary percentage. decrease. Product Line 2 income will be lost, but
The result may be suboptimization. For example, only the traceable costs of commissions, cost of
the sales, and salaries will be avoided. Accordingly, the
buyer may purchase at a lower price from an decrease will be $234,000 [-$700,000 + ($14,000
outside + 420,000 + 32,000)]. The other shared costs will
supplier even though the price exceeds the have to be absorbed by the two remaining product
company's lines.
outlay cost.
Answer (D) is incorrect because an increase of
Answer (D) is incorrect because, if the supplying $416,000 subtracts the costs that will not be avoided
division recovers only its variable production costs, if Product Line 2 is dropped from the lost sales
it revenue.
becomes a cost center for internal transfers. The
supplying division will earn no return on internal
sales, [423] Source: CIA 1196 III-100
which could lead to suboptimization.
Answer (A) is correct. Product Line 1 needs to incurred. If production is increased without increasing
cover its variable out-of-pocket costs as a minimum sales, inventories will rise. However, all fixed costs
on this special-order product; therefore, any selling associated with production will be an expense of the
price greater than the variable cost will contribute period under variable costing. Thus, this action will
towards profits. Thus, the minimum selling price of not artificially increase profits and improve the
the special-order product is the variable cost manager's review.
divided
by 1 minus the commission rate, or $15 [$14.70 ・ Answer (D) is incorrect because, under variable
(1.0 - .02)]. costing, operating profit is a function of sales. Under
absorption costing, it is a function of sales and
Answer (B) is incorrect because $17.30 includes the production.
average cost of salaries (at the new volume level of
24,000 units) as a cost that needs to be covered
when determining the minimum selling price. [426] Source: CIA 0596 III-86

Answer (C) is incorrect because $27.50 is Answer (A) is incorrect because $57,600 equals
calculated 10,000 units times $5.76 per unit (total budgeted
based on a full cost approach. fixed manufacturing overhead ・500,000 units).

Answer (D) is incorrect because $30.20 adds all Answer (B) is correct. The difference between
costs and expenses (except cost of sales) and variable costing and absorption costing is that the
divides former treats fixed manufacturing overhead as a
them by the original volume level of 20,000 units to period cost. The latter method treats it as a product
determine the average operating costs. The new cost. Given that sales exceeded production, both
cost methods expense all fixed manufacturing overhead
of sales is added to the average operating costs to incurred during the year. However, 10,000 units
determine the minimum selling price. (510,000 sales - 500,000 production) manufactured
in a prior period were also sold. These units
presumably were recorded at $10 under variable
[424] Source: CIA 1194 III-42 costing and $16 under absorption costing.
Consequently, absorption costing operating income is
Answer (A) is incorrect because fixed $60,000 (10,000 units x $6) less than that under
manufacturing variable costing.
overhead costs are neither direct nor period costs.
Answer (C) is incorrect because $90,000 is the
Answer (B) is incorrect because fixed difference between planned sales (495,000 units) and
manufacturing actual sales (510,000 units), times the fixed
overhead costs are not period costs. manufacturing overhead per unit ($6).

Answer (C) is incorrect because fixed Answer (D) is incorrect because $120,000 is the
manufacturing volume variance under absorption costing.
overhead costs are not direct costs.

Answer (D) is correct. Using absorption costing, [427] Source: CIA 0596 III-80
fixed manufacturing overhead is included in
inventoriable (product) costs. Fixed manufacturing Answer (A) is incorrect because $(120,000)
overhead costs are indirect costs because they considers only the production costs of the good units
cannot be directly traced to specific units produced. sold. Moreover, it includes fixed overhead, a cost
that is not affected by the choice of materials.

[425] Source: CIA 0594 III-46 Answer (B) is incorrect because $120,000 considers
only the variable costs of the good units produced.
Answer (A) is incorrect because increasing
inventories increases absorption costing profit as a Answer (C) is correct. If a different direct material is
result of capitalizing fixed factory overhead. used, incremental revenue will be $1,500,000
{[(12% defect rate - 2%) x 300,000 units] x $50}.
Answer (B) is incorrect because when sales volume Incremental cost will be $750,000 ($2.50 x 300,000
exceeds production, inventories decline. Thus, fixed units). Thus, the net benefit will be $750,000
factory overhead expensed will be greater under ($1,500,000 - $750,000).
absorption costing.
Answer (D) is incorrect because $1,425,000
Answer (C) is correct. Absorption (full) costing is the includes only the incremental direct materials cost of
accounting method that considers all the increase in the number of good units produced.
manufacturing
costs as product costs. These costs include variable
and fixed manufacturing costs whether direct or [428] Source: CIA 0596 III-95
indirect. Variable (direct) costing considers only
variable manufacturing costs to be product costs, Answer (A) is incorrect because $61,300 is the
i.e., savings for 6 months.
inventoriable. Fixed manufacturing costs are
considered period costs and are expensed as Answer (B) is correct. The assumption is that a third
of the costs can be eliminated if the error rate is
cut [431] Source: Publisher
by a third. Moreover, the study covered only a
6-month period, but annual savings are requested. Answer (A) is incorrect because managerial
Thus, the savings for 6 months equals $61,300 accounting is future oriented.
{[(.03
- .02) x 33,000 invoices x $110 per invoice] + Answer (B) is correct. Financial accounting is
($75,000 ・3) lost contribution margins}, and the primarily concerned with historical accounting, i.e.,
projected annual savings is $122,600 (2 x traditional financial statements, and with external
$61,300). financial reporting to creditors and shareholders.
Managerial accounting applies primarily to the
Answer (C) is incorrect because $222,600 assumes planning and control of organizational operations,
the full amount of lost contribution margins can be considers nonquantitative information, and is usually
saved. less precise.

Answer (D) is incorrect because $267,800 assumes Answer (C) is incorrect because financial accounting
the error rate will be reduced to 0%. is primarily concerned with quantitative information.

Answer (D) is incorrect because decision analysis


[429] Source: CIA 1195 III-91 and implementation are characteristics of managerial
accounting.
Answer (A) is incorrect because $9.70 omits all of
the fixed costs.
[432] Source: Publisher
Answer (B) is incorrect because $11.05 bases the
cost per unit on 600,000 units. Answer (A) is incorrect because it is economically
irrational to implement a system in which the cost
Answer (C) is incorrect because $11.50 uses a fixed exceeds the benefits.
cost per unit based on 600,000 units.
Answer (B) is incorrect because cost-benefit analysis
Answer (D) is correct. The company expects to should not be ignored. An accounting system should
incur be selected on an objective basis.
variable costs of $3,500,000 ($7 x 500,000 lbs.)
and Answer (C) is incorrect because dictation of a
fixed costs of $1,080,000 ($810,000 + $270,000). change by the board of directors implies a system
To earn a 20% return on invested capital (20% x selection on an other than objective basis.
$6,750,000 = $1,350,000), the company must
charge a price of $11.86 [($3,500,000 + Answer (D) is correct. Changing to a different and/or
$1,080,000 + $1,350,000) ・500,000]. more expensive accounting system requires
cost-benefit analysis. Changes should be undertaken
only if the benefits of the proposed change exceed its
[430] Source: CIA 1195 III-92 cost.

Answer (A) is correct. The minimum selling price


equals the incremental costs of the special order [433] Source: Publisher
(variable manufacturing costs, variable packaging
costs, distribution costs, and setup costs) divided Answer (A) is correct. Users, i.e., managers, must be
by both willing and able to use new accounting systems.
the units ordered. The fixed costs do not change Accordingly, they should be encouraged to
because the manufacturer has excess capacity. participate in the design and implementation of the
Total new system. If they do not, they may not understand
variable manufacturing costs are $190,000 or want to use the new information.
[$40,000
x ($4.85 - $.45 + $.35)], distribution costs are Answer (B) is incorrect because the costs of
$32,000, and setup costs are $60,000. Thus, the gathering and analyzing data are more direct costs of
minimum unit price is $7.05 [($190,000 + $32,000 changing managerial accounting systems and are
+ usually considered.
$60,000) ・$40,000].
Answer (C) is incorrect because training costs for
Answer (B) is incorrect because $8.85 adds the staff are usually considered.
additional distribution costs to the original
distribution Answer (D) is incorrect because report preparation is
costs [$1.80 + .80]. explicitly considered in systems development.

Answer (C) is incorrect because $9.05 adds an


amount for the fixed costs to the relevant costs. [434] Source: Publisher

Answer (D) is incorrect because $9.55 adds an Answer (A) is incorrect because the preparation of
amount for the target return on investment to the tax returns is a typical controller's responsibility.
relevant costs.
Answer (B) is incorrect because external reporting is
a function of the controller. "ability to bear" criterion for allocating service and
administrative costs. See SMA 4B, Allocation of
Answer (C) is incorrect because the accounting Service and Administrative Costs.
system helps to safeguard assets.
Answer (B) is incorrect because it is superior to the
Answer (D) is correct. Controllers are usually in "ability to bear" criterion for allocating service and
charge of budgets, accounting, accounting reports, administrative costs. See SMA 4B, Allocation of
and related controls. Treasurers are most often Service and Administrative Costs.
involved with control over cash, receivables,
short-term investments, financing, and insurance. Answer (C) is incorrect because it is superior to the
Thus, treasurers rather than controllers are "ability to bear" criterion for allocating service and
concerned administrative costs. See SMA 4B, Allocation of
with investor relations. Service and Administrative Costs.

Answer (D) is correct. Ability to bear, measured in


[435] Source: Publisher terms of the cost object's profitability, is not an
acceptable method because it has a dysfunctional
Answer (A) is correct. Treasurers are usually effect on management behavior. It penalizes high
concerned with investing cash and near-cash performance instead of rewarding profitability.
assets,
the provision of capital, investor relations,
insurance, [438] Source: Publisher
etc. Controllers, on the other hand, are responsible
for the reporting and accounting activities of an Answer (A) is incorrect because, in the growth stage,
organization, including financial reporting. growth is rapid, and net income and cash from
operations are positive. Investment requirements later
Answer (B) is incorrect because short-term slacken, and positive operating cash flows result.
financing ROI improves.
lies within the normal range of a treasurer's
functions. Answer (B) is incorrect because, in the maturity
stage, revenues slow, net income remains positive,
Answer (C) is incorrect because the treasurer has investment of capital is low, and a high return is
custody of assets. earned on their assets employed.

Answer (D) is incorrect because credit operations Answer (C) is incorrect because, in the decline stage,
are often within the treasurer's purview. revenues decline and operations remain profitable,
but cash flows increase because of a reduction in
working capital.
[436] Source: Publisher
Answer (D) is correct. Revenue growth, negligible
Answer (A) is correct. Management accounting is profits, negative cash flows, and negative return on
used by management for (1) control, (2) assurance assets are common during an organization's start-up
of stage. This is in contrast to the growth stage, maturity
accountability, (3) planning, (4) evaluation, and (5) stage, and decline stage. See SMA 4D, Measuring
reporting. Management accounting includes the Entity Performance.
following processes: (1) measurement, (2)
identification, (3) accumulation, (4) preparation, (5)
interpretation, (6) communication, and (7) analysis. [439] Source: CMA 0697 3-13
Marketing is supported by accounting data, but
management does not use management Answer (A) is incorrect because both tactical and
accounting strategic goals should be attainable.
for marketing, per se. See SMA 1A, Objectives of
Management Accounting. Answer (B) is incorrect because top management is
responsible for both types of goals.
Answer (B) is incorrect because it is an explicit use
of Answer (C) is incorrect because both types of goals
management accounting by an organization's may or may not address profitability.
management.
Answer (D) is correct. Tactical goals are short term
Answer (C) is incorrect because it is an explicit use and strategic goals are long range. Tactical goals
of management accounting by an organization's concern operational matters such as production,
management. materials procurement, and routine operating
expenses. Strategic goals may have a duration of 10
Answer (D) is incorrect because it is an explicit use years or more. They concern such matters as new
of management accounting by an organization's product development, capital budgeting, major
management. financing, and business combinations.

[437] Source: Publisher [440] Source: Publisher

Answer (A) is incorrect because it is superior to the Answer (A) is correct. According to SMA 5D,
Developing Comprehensive Competitor unit cost direct materials only if the rate of discount
Intelligence, exceeds reasonable interest rates, i.e., in effect are
objectives include early warning of opportunities trade discounts or rebates. See SMA 4A, Definition
and of Measurement of Direct Material Cost.
threats, such as new acquisitions, alliances,
products, Answer (C) is incorrect because purchasing,
and services. receiving, inspection, and storage costs are not a
direct material cost, per se.
Answer (B) is incorrect because SMA 5D concerns
broad-scope competitive intelligence, not business Answer (D) is incorrect because cash discounts
intelligence issues such as environmental should not be reflected whether the discounts are
scannings. taken or not. If they are taken, they should be
recorded as an interest saving, not as a reduction in
Answer (C) is incorrect because SMA 5D concerns the cost of materials.
broad-scope competitive intelligence, not business
intelligence issues such as market research.
[443] Source: Publisher
Answer (D) is incorrect because SMA 5D concerns
broad-scope competitive intelligence, not business Answer (A) is incorrect because it is an attribute of
intelligence issues. performance indicators.

Answer (B) is incorrect because it is an attribute of


[441] Source: Publisher performance indicators.

Answer (A) is incorrect because effective capacity Answer (C) is incorrect because it is an attribute of
management maximizes value delivered to performance indicators.
customers.
Answer (D) is correct. A broad-based performance
Answer (B) is incorrect because effective capacity indicator system must be forward looking, focus on
management minimizes required future significant external as well as internal relationships,
investment. and track nonfinancial as well as financial indicators.
Accordingly, they cannot rely on traditional,
Answer (C) is correct. According to SMA 4Y, historical, internal financial measures. See SMA 4U,
Measuring the Cost of Capacity, maximizing the Developing Comprehensive Indicators.
value
created within an organization starts with
understanding the nature and capabilities of all of [444] Source: Publisher
the
company's resources. Capacity is defined from Answer (A) is incorrect because it ignores the tax
several different perspectives. Managing capacity deductibility of interest payments.
cost starts when a product or process is first
envisioned. It continues through the subsequent Answer (B) is correct. The cost of debt capital is
disposal of resources downstream. Effective simply the debt interest rate times (1 - the firm's tax
capacity rate). Thus, if the tax rate is 40%, the effective cost of
cost management requires supporting effective debt capital is 60% times the interest rate because the
matching of a firm's resources with current and interest is tax deductible. See SMA 4A, Cost of
future Capital.
market opportunities.
Answer (C) is incorrect because it ignores the tax
Answer (D) is incorrect because effective capacity deductibility of interest payments.
management minimizes waste in the short,
intermediate, and long run. Answer (D) is incorrect because the capital asset
pricing model is one means of determining the cost of
common equity.
[442] Source: Publisher

Answer (A) is incorrect because estimates are used [445] Source: CIA 1195 III-67
in the calculation of direct material costs only if
they Answer (A) is incorrect because ROI is certain to
are sufficiently accurate. increase only if revenue increases and costs and
investment decrease.
Answer (B) is correct. Direct material costs consist
of quantities of materials that can be specifically Answer (B) is incorrect because ROI is certain to
identified with the cost object in an economically increase only if revenue increases and costs and
feasible manner priced at the unit price of direct investment decrease.
material. It includes sales taxes, customs duties,
cost Answer (C) is incorrect because ROI is certain to
of delivery, and is net of trade discounts, refunds, increase only if revenue increases and costs and
and investment decrease.
rebates. Cash discounts should be deducted from
the Answer (D) is correct. An increase in revenue and a
decrease in costs will increase the ROI numerator. A average.
decrease in investment will decrease the
denominator. Answer (B) is incorrect because 0.0820 is the cost of
The ROI must increase in this situation. common stock.

Answer (C) is incorrect because 0.0660 is the cost


[446] Source: Publisher of debt.

Answer (A) is incorrect because cost of capital is Answer (D) is correct. According to SMA 4A, the
used to make capital investment decisions so that cost of capital is defined as the composite cost of
each investment returns more than the cost of various sources of funds included in a firm's capital
capital. structure. It is the minimum rate of return that must be
earned on new investments that will not dilute the
Answer (B) is incorrect because the cost of interests of the shareholder. In this problem,
maintaining working capital is based on the cost of
capital. Company X has three sources of funds: debt,
common stock, and preferred stock. The cost of
Answer (C) is correct. The cost of capital is the debt capital is the after-tax cost of debt to the firm.
minimum, not maximum, rate of return that must Given that the firm is taxed at 40% and interest is a
be tax deduction, the cost to the firm is only 0.066 [(1.0
earned on new investments so as not to dilute - 0.4) x (.11)]. The cost of preferred stock is the
shareholder interest. If new investments have a annual dividend requirement divided by the current
rate of price per share or the proceeds from issuance per
return less than the cost of capital, a loss will be share. Thus, the component cost of preferred stock
incurred on those investments. Thus, the cost of for Company X is 0.075 ($2.25 ・30). The cost of
common stock is difficult to determine accurately, but
capital must be the minimum rate of return. See one of the most common estimations involves using
SMA the capital asset pricing model to determine the cost.
4A, Cost of Capital. Thus, the cost of common stock equals the risk-free
rate plus the product of beta for Company X and the
Answer (D) is incorrect because the performance of market risk premium. The component cost of
individual investments, investment managers, and common stock for Company X is 0.082 [.04 + (.7 x
others can be related to the cost of capital. .06)]. To determine the weighted-average cost of
capital, each component cost is multiplied by its
respective weight in the company capital structure,
[447] Source: Publisher and these values are summed to arrive at 0.0733
[(.066 x .5) + (.082 x .4) + (.075 x .1)].
Answer (A) is correct. Responsibility costs are
designed to motivate managers of a responsibility
center to act in the best interest of the [449] Source: CMA 1296 3-30
organization.
Therefore, the costs should be allocated only if they Answer (A) is incorrect because $1,200,000
(1) can be influenced by the actions of the center's assumes that the by-product is charged with a portion
management, (2) are helpful in measuring support of the net joint cost.
given to the responsibility center, (3) improve
comparability, or (4) are used in product pricing. Answer (B) is incorrect because $1,260,000
Whether the costs are from staff, line, or other assumes that the by-product is charged with a portion
services has no bearing on whether they should be of the gross joint cost.
allocated. Furthermore, some organizations
encourage the use of services such as consulting or Answer (C) is correct. The joint cost to be allocated
internal audit by not charging their costs to is $2,400,000 [$2,520,000 total joint cost - ($2 x
responsibility centers. See SMA 4B, Allocation of 60,000 pounds of the by-product)]. Accordingly, the
Service and Administrative Cost. joint cost to be allocated to the Second Main
Product on a physical-volume basis is $1,500,000
Answer (B) is incorrect because it provides {$2,400,000 x [150,000 pounds ・(90,000 pounds
justification for allocating cost to responsibility + 150,000 pounds)]}.
centers.
Answer (D) is incorrect because $1,575,000 does
Answer (C) is incorrect because it provides not deduct by-product NRV from the joint cost.
justification for allocating cost to responsibility
centers.
[450] Source: Publisher
Answer (D) is incorrect because it provides
justification for allocating cost to responsibility Answer (A) is incorrect because this standard is
centers. violated by a financial manager/management
accountant who fails to act upon discovering unethical
conduct.
[448] Source: Publisher
Answer (B) is incorrect because this standard is
Answer (A) is incorrect because 0.0743 is the violated by a financial manager/management
simple accountant who fails to act upon discovering unethical
conduct.
influence users be fully disclosed.
Answer (C) is incorrect because this standard is
violated by a financial manager/management Answer (D) is incorrect because the confidentiality
accountant who fails to act upon discovering standard concerns the financial manager/management
unethical accountant's responsibility not to disclose or use the
conduct. firm's confidential information.

Answer (D) is correct. A financial


manager/management accountant displays his/her
competence and objectivity and maintains integrity [453] Source: Publisher
by
taking the appropriate action within the Answer (A) is correct. One of the responsibilities of
organization the financial manager/management accountant under
to resolve an ethical problem. Failure to act would the integrity standard is to "recognize and
condone wrongful acts, breach the duty to convey communicate professional limitations or other
unfavorable as well as favorable information, constraints that would preclude responsible judgment
undermine the organization's legitimate aims, or successful performance of an activity."
discredit
the profession, and violate the duty of objectivity Answer (B) is incorrect because the objectivity
owed to users of the subordinate's work product. standard requires the financial manager/management
accountant to "disclose fully all relevant information
that could reasonably be expected to influence an
[451] Source: Publisher intended user's understanding of the reports,
comments, and recommendations presented."
Answer (A) is incorrect because the code does not
address these matters. Answer (C) is incorrect because the confidentiality
standard requires the financial manager/management
Answer (B) is incorrect because the code does not accountant to "refrain from disclosing confidential
address these matters. information acquired in the course of his/her work
except when authorized, unless legally obligated to do
Answer (C) is correct. Financial so."
managers/management accountants may not dis
close Answer (D) is incorrect because the integrity
confidential information acquired in the course of standard requires the financial manager/ management
their accountant to "refuse any gift, favor, or hospitality
work unless authorized or legally obligated to do that would influence or would appear to influence
so. his/her actions."
They must inform subordinates about the
confidentiality of information and monitor their
activities to maintain that confidentiality. Moreover, [454] Source: Publisher
financial managers/management accountants
should Answer (A) is incorrect because the integrity
avoid even the appearance of using confidential standard requires the financial manager/management
information to their unethical or illegal advantage. accountant to "communicate unfavorable as well as
favorable information and professional judgments or
opinions."
Answer (D) is incorrect because other employment
may be accepted unless it constitutes a conflict of Answer (B) is correct. One of the responsibilities of
interest. the financial manager/management accountant under
the competence standard is to "maintain an
appropriate level of professional competence by
[452] Source: Publisher ongoing development of his/her knowledge and
skills."
Answer (A) is incorrect because the competence
standard pertains to the financial Answer (C) is incorrect because one of the
manager/management accountant's responsibility suggestions from the "Resolution of Ethical Conflict"
to paragraph is to "clarify relevant ethical issues by
maintain his/her professional skills and knowledge. confidential discussion with an objective advisor (e.g.,
It IMA Ethics Counseling Service) to obtain a better
also pertains to the performance of activities in a understanding of possible courses of action."
professional manner.
Answer (D) is incorrect because the confidentiality
Answer (B) is incorrect because legality is not standard requires the financial manager/management
addressed in the IMA Code of Ethics. accountant to "inform subordinates as appropriate
regarding the confidentiality of information acquired in
Answer (C) is correct. Objectivity is the fourth part the course of their work and monitor their activities to
of the IMA Code of Ethics. It requires that assure the maintenance of that confidentiality."
information be communicated "fairly and
objectively,"
and that all information that could reasonably [455] Source: CMA 1
the competence standard is to "maintain an
Answer (A) is incorrect because the competence appropriate level of professional competence by
standard pertains to the financial ongoing development of his/her knowledge and
manager/management accountant's responsibility skills." (S)he must also "perform professional duties in
to accordance with relevant laws, regulations, and
maintain his/her professional skills and knowledge. technical standards." The third requirement under this
It standard is to "prepare complete and clear reports
also pertains to the performance of activities in a and recommendations after appropriate analyses of
professional manner. relevant and reliable information."

Answer (B) is incorrect because the confidentiality Answer (B) is incorrect because the confidentiality
standard concerns the financial standard concerns the financial manager/management
manager/management accountant's responsibility not to disclose or use the
accountant's responsibility not to disclose or use firm's confidential information.
the
firm's confidential information. Answer (C) is incorrect because the integrity
standard pertains to conflicts of interest, refusal of
Answer (C) is correct. One of the responsibilities of gifts, professional limitations, professional
the financial manager/management accountant communications, avoidance of acts discreditable to
under the profession, and refraining from activities that
the integrity standard is to "recognize and prejudice the ability to carry out duties ethically.
communicate professional limitations or other
constraints that would preclude responsible Answer (D) is incorrect because objectivity is the
judgment fourth part of the IMA Code of Ethics. It requires
or successful performance of an activity."
that information be communicated "fairly and
Answer (D) is incorrect because objectivity is the objectively," and that all information that could
fourth part of the IMA Code of Ethics. It requires reasonably influence users be fully disclosed.
that information be communicated "fairly and
objectively," and that all information that could
reasonably influence users be fully disclosed.

[456] Source: CMA 2

Answer (A) is incorrect because the competence


standard pertains to the financial
manager/management accountant's responsibility
to
maintain his/her professional skills and knowledge.
It
also pertains to the performance of activities in a
professional manner.

Answer (B) is incorrect because the confidentiality


standard concerns the financial
manager/management
accountant's responsibility not to disclose or use
the
firm's confidential information.

Answer (C) is correct. The integrity standard


requires
the financial manager/management accountant to
"refuse any gift, favor, or hospitality that would
influence or would appear to influence his/her
actions.

Answer (D) is incorrect because objectivity is the


fourth part of the IMA Code of Ethics. It requires
that information be communicated "fairly and
objectively," and that all information that could
reasonably influence users be fully disclosed.

[457] Source: CMA 3

Answer (A) is correct. One of the responsibilities of


the financial manager/management accountant
under

You might also like